Paediatrics Flashcards
Define appendicitis
Acute appendicitis is an acute inflammation of the vermiform appendix, most likely due to obstruction of the lumen of the appendix (by faecolith, normal stool, infective agents, or lymphoid hyperplasia).
Explain RFs for appendicitis
<6m breast feeding
Low fibre
Improved hygeine?
Passive smoking
Aetiology of appendicitis
Obstruction of the lumen of the appendix is the main cause of acute appendicitis. Faecolith (a hard mass of faecal matter), normal stool, or lymphoid hyperplasia are the main causes for obstruction. Faecolith alone causes simple appendicitis in 40%, gangrenous non-perforated appendicitis in 65%, and perforated appendicitis in 90% of cases.
There is evidence suggesting a neuroimmune aetiology in some cases, but this is still being investigated.
Sx of appendicitis
Abdominal pain
- Mid -> RIF
- Worse on movement
Anorexia
RIF tenderness N+V Fever Dec bowel sounds Tachycardia Fetor
ROSVINGs
- Pressing on LIF illicits pain in RIF
PSOAS
- Extending the right thigh on left lateral position elicits pain in right lower quadrant.
OBTURATOR
- Pain is elicited at right lower quadrant of abdomen by internal rotation of the flexed right thigh.
What are the relevant Ix for appendicitis
FBC -> inc WCC
ABDO/PELV CT - abnormal appendix (diameter >6 mm) identified or calcified appendicolith seen in association with peri-appendiceal inflammation
URINE PREG TEST NEG
Abdo USS - aperistaltic or non-compressible structure with outer diameter >6 mm
NEGATIVE URINALYSIS
Management of appendicitis
Open or LAP Appendectomy
Supportive care - IV fluids
Given for 24 hours for uncomplicated appendicitis: IV ABx - CEFOXITIN 1-2g prior to surgery, 1-2g 8hrly post surgery
PERFORATION/ABSCESS:
- Maintenance of BP/pulse
- Begin ABx immediately
- Abscess - may need drainage first THEN INTERVAL appendectomy
Complications of appendicitis
Perforation Peritonitis Appendicular mass Appendicular abscess Surgical wound infection
Prognosis of appendicitis
If patients are treated in a timely fashion, the prognosis is good. Wound infection and intra-abdominal abscess are potential complications associated with appendectomy. Laparascopic appendectomy has been shown to decrease the incidence of overall complications.
Define ADHD
ADHD is a triad of inattention, hyperactivity, and impulsivity.
A key element of the definition is functional impairment across TWO OR MORE domains, most often in school and at home.
ADHD can limit academic, interpersonal, and occupational success and can also lead to greater risk-taking and accidents.
In addition, patients with ADHD are more likely to have co-existing psychiatric disorders such as oppositional defiant disorder (ODD), conduct disorder, substance abuse, and possibly mood disorders, such as depression and mania.
Aetiology ADHD
Genetic predisposition: there is substantial evidence for a genetic contribution to ADHD, with the mean heritability for ADHD shown to be 76% based on twin studies.
Environmental factors: these account for 12% to 40% of the variance in twin ADHD scores. Low birth weight and maternal smoking have the strongest evidence for association with ADHD. Other risk factors include poverty, lead exposure, iron deficiency, maternal alcohol drinking during pregnancy, and psychosocial adversity.
RFs ADHD
STRONG
FHx Male LBW (<2500) Epilepsy Maternal nicotine use during pregnancy
WEAK
Maternal alcohol preg Stress during preg Psychosocial adversity - severe marital discord, low social class, large family size, paternal criminality, maternal mental disorder, and foster placement Lead exposure Traumatic brain injury Severe early deprivation Iron deficiency
Sx of ADHD
Careless mistakes/missing detail Attention deficit Listening deficit Instructions deficit Organisational difficulties Reluctance to engage in long activities Loses things necessary for tasks Easily distracted Forgetful Fidgeting Failure to remain seated Excessive talking Blurts out answers Cannot wait turn Interrupts others Mild mood symptoms Difficult peer interactions Low self-esteem Working memory impairment Processing speed impairment
Ix of ADHD
Clinical diagnosis
BUT
Behavioural rating scales EG:
The ADHD Rating Scale: an 18-item scale [DSM]
The Vanderbilt Scale: a 55-item scale, which assesses ADHD, comorbid conditions, and performance.
SNAP-IV is included in many research trials, including the Multimodal Treatment Study of AD/HD (MTA). It is a 90-item scale that screens for ADHD and other diagnoses.
Neuropsychological testing by child psychiatrist
Rx of ADHD
Methylphenidate -> can be immediate release / extended release / transdermal
Dexamfetamine (immediate/extended/TD)
Behavioural therapy
- parent training
- teacher training
2nd line: change stimulant
3rd: Atomoxetine
- CONSIDER CHILD PSYCH REFERRAL
3rd: Guanfacine or Clonidine
- NB also antihypertensives: SE hypo/bradyC/rebound HTN
4th: Nortriptyline / Bupropion / Imipramine
AKA ANTIDEPRESSANTS
NB if child has tick:
- Guanfacine or clonifidine or atomoxetine used
Complications of ADHD
SEs of meds:
Anorexia Insomnia Cardiac Mood lability Headache Psychotic symptoms Tics Substance abuse Growth delay
Prognosis of ADHD
Between 60% to 85% of patients with ADHD will continue to meet criteria in adolescence, and significant functional impairment often persists into adulthood.
Over time, symptoms of hyperactivity tend to remit, while impairments in attention persist. In fact, patients with the predominantly inattentive type of ADHD often present later (e.g., middle school, high school) because their lack of hyperactivity and impulsivity makes them less disruptive in primary school than children with combined type.
Adolescents and adults with symptoms of ADHD have higher risk for academic and professional difficulties, development of conduct disorder and antisocial behaviours, maladaptive relationships, increased injuries and car accidents, and teen pregnancies.
Define bronchiolitis
Acute viral infection of the lower respiratory tract.
Although it can affect individuals of any age, the term is most often used to refer to infection in infancy. It is characterised by epithelial cell destruction, cellular oedema, and airway obstruction by inflammatory debris and mucus.
The clinical manifestations include cough, wheeze, and laboured breathing.
Respiratory syncytial virus (RSV) accounts for the majority of cases, although rhinovirus, human metapneumovirus, influenza, parainfluenza, and adenovirus can all cause bronchiolitis as well.
Aetiology of bronchiolitis
The most common cause is respiratory syncytial virus (RSV). In one cohort study of bronchiolitis, RSV was responsible for 76%, rhinovirus for 18%, influenza virus for 10%, coronavirus for 2%, and human metapneumovirus for 3%, and 1% had parainfluenza. Bronchiolitis caused by RSV and other respiratory viruses begins as an upper respiratory tract infection, which then spreads to the lower respiratory tract in 1 to 3 days.
RSV infection occurs in almost all infants by 3 years of age, but only a minority develop bronchiolitis. This observation has led to the hypothesis that host and possibly environmental factors play a role in disease pathogenesis. Birth cohort studies have shown that diminished lung function at birth is a risk factor for wheezing in early infancy, but this mechanism cannot completely explain the variability of clinical manifestations of RSV infection. Environmental tobacco smoke exposure may also contribute to disease severity. Another area of focus has been the role of the host immune response in determining the effects of RSV infection.
RFs bronchiolitis
STRONG <3yo Nov-May Prematurity Bronchopulmonary dysplasia Passive smoking Impaired airway clearance/function eg PCD Congenital HD Immunodeficiency
Epidemiology of bronchiolitis
Bronchiolitis is one of the most common acute illnesses in infancy and the leading cause of hospitalisation in this age group. Approximately 1 in every 30 infants will be diagnosed with bronchiolitis in their first year of life.
Bronchiolitis is almost exclusively an infantile disease, and by 3 years of age essentially all children have serological evidence of having been infected with RSV. However, primary infection with RSV in infants does not confer protective immunity, so repeat infections are common. Although in most infants the disease is mild and self-limited, severe disease can occur, especially in infants under 6 months of age. Infants with underlying risk factors for severe infection, such as prematurity, congenital heart disease, or chronic lung disease, have a greater risk of hospitalisation, but the majority of hospitalisations are in infants with no underlying risk factors.
In addition to the acute effects of bronchiolitis, studies have demonstrated that a significant proportion of infants with RSV bronchiolitis go on to develop recurrent wheezing; rhinovirus has been increasingly studied and shown to have an association with recurrent wheezing and a diagnosis of asthma.
Risk factors such as family history of asthma increase the risk of a future asthma diagnosis.
Sx of bronchiolitis
Cough - dry/wet/croupy Tachypnoea Wheezing Retractions, grunting, flaring Rhinitis Fever <40deg Apnoea
A hallmark of bronchiolitis is fluctuating clinical findings, often within short time periods.
Ix bronchiolitis
Hypoxaemia (<90%)
CONSIDER
Elisa rapid antigen detection
RT-PCR
CxR - ONLY PERFORMED IF V SEVERE - hyperinflation, interstitial inflammation, atelectasis
Rx bronchiolitis
Preventative - palivizumab
Supportive - maintain O2
Mechanical ventilation is indicated for respiratory failure.
RIBAVIRIN
? SOME STUDIES SHOW corticosteroids MAY benefit if Hx of wheeze
Complications bronchiolitis
Bacterial Pneumonia
Recurrent wheeze
Paed asthma
Prognosis bronchiolitis
In general, viral bronchiolitis is a mild condition that is self-limiting and resolves within a few days.
Define Cerebral Palsy
Cerebral palsy (CP) is an umbrella term referring to a non-progressive disease of the brain originating during the antenatal, neonatal, or early postnatal period when brain neuronal connections are still evolving.
Secondary effects of spasticity on growth may, however, be progressive. There may be additional disturbances of sensation, perception, cognition, communication, and behaviour; neurogenical bladder/bowel; GORD; sialorrhoea (excessive secretion of saliva); feeding and swallowing difficulties; and/or epilepsy. CP is also known as Little’s disease.
What is the aetiology of Cerebral Palsy
Antenatal:
Prematurity Multiple birth TORCH (toxo, rubella, CMV, Herpes) Factor Va leiden Chorioamniotnitis Teratogen exposure Genetic/metabolic disorders Fetal brain malformation
Perinatal:
<10% of cases - birth asphyxia due to instrumental delivery, non-vertex presentation, birth trauma, placental abruption, rupture of the uterus or prolonged/obstructed labour, and postmaturity
Postnatal:
Hyperbilirubinaemia, neonatal sepsis, respiratory distress, early-onset meningitis, intraventricular haemorrhage, and head injuries prior to 3 years (including child abuse and shaken baby syndrome).
Around 25% of infants who survive neonatal seizures have CP
Discuss epidemiology of Cerebral Palsy
Most common cause of childhood disability affecting 2.5 per 1000 individuals in the industrialised world.
RF for Cerebral Palsy
STRONG Prem Fetal birth asphyxia MultiGest Maternal illness Fetal brain malformation Major birth defects Familial metabolic disorder Neonatal complications Maternal teratogen exposure Low socio
WEAK
Non-vertex presentation
Postmaturity
Head injury
Sx for Cerebral Palsy
All patients present with motor impairment; 80% have spasticity. Other movement disorders observed are dystonia, athetosis, chorea, and ataxia.
Other common problems include feeding difficulties, speech impairment, intellectual deficits, urinary incontinence, and a variable sensory/proprioceptive loss.
DELAY IN MOTOR/SPEECH?COGNITIVE
Retention of primitive reflexes Lack of age appropriate reflexes Spasticity/clonus Toe walking/knee hyperextension Scissoring Crouched gait Contractures Weakness Joint instability Dystonia Chorea Athetosis (changing writhing contorting movements) Ataxia Neonatal hypotonia Scoliosis
Ix for Cerebral Palsy
MRI BRAIN
- periventricular leukomalacia, congenital malformation, stroke or haemorrhage, cystic lesions
CONSIDER - ie for causes Coagulation studies ?5a Leiden Genetic testing Metabolic screen Xray of joint
Rx for Cerebral Palsy
Management is multidisciplinary and includes occupational, physical, and speech therapy; neurology and neurosurgery; psychiatry; urology; ophthalmology; and orthopaedic, paediatric, dietary, and psychosocial services. Educational and vocational support is also needed.
Spasticity can be treated with oral medications, botulinum toxin and other injections; intrathecal baclofen; or selective posterior rhizotomy.
Physical, occupational, and speech therapy address motor function, communication, and activities of daily living to prevent deformity and optimise independence and quality of life. ADAPTIVE EQUIPMENT
Orthopaedic interventions address contractures, scoliosis, subluxing hips, and extremity deformity.
Prognosis for Cerebral Palsy
A child with CP becomes an adult with CP. Except for the most severely involved patients, people with CP survive to senior age status, though some require lifelong help and accommodation.
Adults with CP have an increased risk of non-communicable diseases such as stroke, COPD, or cardiac conditions and an increased risk of related deaths.
Reflexes and reactions in early infancy that are poor prognostic factors for the development of independent walking include:
Retention of asymmetric and symmetric tonic neck reflexes
Retention of Moro (startle) reflex
Retention of neck righting reflex
Presence of lower-extremity extensor thrust response
Lack of parachute reaction
Lack of foot placement reaction.
Complications for Cerebral Palsy
Feeding difficulties Microcephaly Hydrocephalus Drooling Osteoporosis Hearing impairment Cervical myelopathy Behaviour Aspiration Poor growth Epilepsy Visual impairment Incontinence GORD Sleep disturbance Osteopenia
An 18-month-old child with a history of prematurity (28 weeks’ gestation, 1200 grams) presents with failure to meet developmental milestones. The child sat independently at 1 year, has few words vocally, does not pull to stand, and exhibits increased deep tendon reflexes in the lower extremities and sustained clonus at both ankles. There is good upper-extremity function. A magnetic resonance imaging scan of the brain reveals periventricular leukomalacia.
The child is diagnosed with spastic diplegic CP
A 2-year-old boy, born after a normal pregnancy and delivery, presents with an asymmetric gait. Examination reveals mild spasticity of the left upper and lower extremity, hyperactive left knee and ankle deep tendon reflexes, and decreased dorsiflexion of the left ankle compared with the right. When walking, the patient walks on his left toes, and his left arm is held mildly flexed at the elbow with the palm facing the floor (pronated forearm). The left calf is smaller in girth than the right leading to the diagnosis of…
hemiplegic CP.
Define Neonatal Encephalopathy
Defined by signs and symptoms of abnormal neurological function in the first few days of life in an infant born at term:
Difficulty initiating and maintaining respirations, a subnormal level of consciousness, and associated depression of tone, reflexes, and possibly seizures.
Encephalopathy is a nonspecific response of the brain to injury which may occur via multiple methods, but is commonly caused by birth asphyxia, leading to cerebral hypoxia.
Epidemiology of Neonatal Encephalopathy
Overall, the relative incidence of neonatal encephalopathy is estimated to be between 2 and 9 per 1000 term births.
40% to 60% of affected infants die by 2 years old or have severe disabilities.
In 2013 it was estimated to have resulted in 644,000 deaths down from 874,000 deaths in 1990.
Sx of Neonatal Encephalopathy
Reduced level of consciousness
Seizures (which peak at 48 hours)
Difficulty initiating and maintaining respiration
Depression of tone and reflexes
Ix for Neonatal Encephalopathy
Cord blood gas analysis can be used to determine if there is perinatal hypoxia/asphyxia, which are potential causes of hypoxic-ischemic encephalopathy or cerebral palsy, and give insight into causes of intrapartum fetal distress.
Cord blood gas analysis is indicated for high-risk pregnancies, in cases where C-sections occurred due to fetal compromise, if there were abnormal fetal heart rate patterns, Apgar scores of 3 or lower, intrapartum fever, or multifetal gestation.
Evidence of brain injury related to the hypoxic-ischemic events that cause neonatal encephalopathy can be seen with brain MRIs, CTs, magnetic resonance spectroscopy imaging or ultrasounds.[5][6]
Rx of Neonatal Encephalopathy
Currently, neonatal encephalopathy is treated using hypothermia therapy.
This has been shown to reduce brain damage, reduce future disability, and improve survival by a 2013 Cochrane review.
Prognosis + Complications of Neonatal Encephalopathy
HIE is a major predictor of neurodevelopmental disability in term infants. 25 percent have permanent neurological deficits.
It can result in developmental delay or periventricular leukomalacia.
Define Croup
Croup, also known as laryngotracheobronchitis, is a common respiratory disease of childhood, characterised by the sudden onset of a seal-like barky cough, often accompanied by stridor, voice hoarseness, and respiratory distress. The symptoms are a result of upper-airway obstruction due to generalised inflammation of the airways, as a result of viral infection (typically parainfluenza virus types 1 or 3).
Aetiology of Croup
The illness is due to viral infection (typically parainfluenza virus types 1 or 3). Several other viral pathogens have been recognised, including influenza A and B, adenovirus, respiratory syncytial virus, metapneumovirus, coronavirus HCoV-NL63, and rarely measles.
The symptoms result from upper-airway obstruction due to generalised inflammation and oedema of the airways. At the cellular level this progresses to necrosis and shedding of the epithelium. The narrowed subglottic region is responsible for the symptoms of seal-like barky cough, stridor (from increased airflow turbulence), and sternal/intercostal indrawing. If the upper-airway obstruction worsens, respiratory failure can result, leading to asynchronous chest and abdominal wall motion, fatigue, hypoxia, and hypercarbia
RFs for Croup
STRONG
6 months - 6 years old
WEAK
Male
Prior intubation
Epidemiology of Croup
Croup is a frequent cause of acute respiratory distress in young children. Typically, it affects those between 6 months and 3 years of age, peaking in the second year of life. It has been reported to occur in infants younger than 6 months, in adolescents, and, more rarely, in adults.
M>F
A peak in clinical presentations is correlated with parainfluenza virus epidemics. These peaks typically occur in alternating years and result in a 50 % increase in the number of children admitted with croup.
Sx of Croup
COMMON Symptoms increases with agititation Seal-like bark Peak late autumn [SEP-DEC] Prodromal symptoms Abrupt onset symptoms Symptoms worse at night Hoarse voice
UNCOMMON Resp distress Persistent agitiation Lethargy synchronous chest wall / abdominal movement Fatigue Hypoxia - cyanosis/pallor Dec consciousness
Ix of Croup
CLINICAL DIAG
Could do Xray lateral neck - see STEEPLE sign
BUT DON’T NORMALLY DO!!
Rx of Croup
Orally administered corticosteroids are the mainstay for all levels of severity, combined with nebulised epinephrine (adrenaline) in moderate to severe croup to provide temporary relief of the symptoms of upper-airway obstruction.
NO STRIDOR:
Dexamethasone: 0.15 to 0.6 mg/kg orally as a single dose
Can use IM dex, or inhaled budesonide (2mg)
SUPPLEMENTAL O2 IF IMPENDING RESP FAILURE
INTUBATE IF NEEDED
Prognosis of Croup
Mild-Mod = reasonable outlook but may appear frightening
Severe: Since combination treatment with dexamethasone and nebulised epinephrine (adrenaline) became standard care, prognosis for severe croup has been excellent.
1-3% resp failure
Complications of Croup
Bacterial Tracheitis
Pneumonia
A 2-year-old boy is brought to the emergency department by his parents in the middle of the night. He has had mild symptoms of an upper respiratory infection for 48 hours, awoke with a sudden onset of seal-like barky cough, and has had inspiratory stridor when crying. The stridor disappeared at rest, but the seal-like barky cough has persisted.
Croup
A 3-year-old boy is brought to the emergency department by his parents in the late evening. He has developed a sudden onset of a seal-like barky cough, accompanied by clear nasal discharge. His parents became alarmed when he developed stridor, which persists throughout the trip to the hospital. On examination, he has a seal-like barky cough and inspiratory stridor when at rest, which worsens with agitation. Persistent sternal indrawing is also evident at rest.
Croup
Define CF
Cystic fibrosis (CF) is a severely life-shortening genetic disease resulting from abnormalities in the cystic fibrosis transmembrane conductance regulator (CFTR), a chloride channel found in cells lining the lungs, intestines, pancreatic ducts, sweat glands, and reproductive organs.
There are over 1500 known disease-causing mutations that interrupt various stages of CFTR synthesis and function.
The most common clinical manifestations are pancreatic dysfunction, resulting in calorie malabsorption; and lung disease, resulting from a cycle of mucus retention, infection, and inflammation.
Epidemiology of CF
There are about 10,400 cases in the UK, 30,000 cases in the US, and 70,000 worldwide.
The incidence among white people is about 1/3000; the incidence is lower among people of African, Hispanic, and Asian descent. It is most common among people of European descent.
One study suggests that the incidence may be decreasing since the institution of newborn screening.
Lung disease is the most common cause of morbidity and mortality.
RFs for CF
FHx
Known carrier parent
Ethnicity (white)
Approximately 1 in 28 white people is a carrier of a CFTR mutation
Aetiology of CF
CF is a genetic disease caused by mutations in the cystic fibrosis transmembrane conductance regulator (CFTR), an anion channel found in the apical membrane of epithelial cells. Patients may be either homozygous or heterozygous with respect to CFTR mutations. Carriers of one CFTR mutation and one normal CFTR allele do not demonstrate disease in most cases.
Pathophysiology:
Mutations in CFTR result in abnormal salt transport by epithelial cells, resulting in thick, sticky secretions.
In the pancreas, this leads to blockage of exocrine ducts, early activation of pancreatic enzymes, and eventual autodestruction of the exocrine pancreas. Therefore, most patients require supplemental pancreatic enzymes.
In the intestine, bulky stools can lead to intestinal blockage.
In the respiratory system, the absence of CFTR function results in mucus retention, chronic infection, and inflammation that eventuate in the destruction of lung tissue
Sx of CF
COMMON
RFs \+ve Newborn screen Failure to pass meconium - May have meconium ileus Failure to thrive Voracious appetite Wet-sounding cough Recurrent lower airway infection Chronic sinusitis Genital abnormalities in males -> lack vas deferens Malabsorptive stool with steatorrhoea Digital clubbing GORD
UNCOMMON Haemoptysis Wheeze Increased AP diameter chest Hx pancreatitis/appendicitis
Ix for CF
Sweat test - positive (sweat chloride >60 mmol/L [≥60 mEq/L]) [NB 30-60 = must do more Ix]
Immunoreactive trypsinogen test - +ve but must be followed up with an additional test
Genetic testing - presence of 2 disease-causing mutations
Rx for CF
GENERAL ONGOING Rx:
Chest physio, PEEP, Oscillatory devices: airway clearance
Salbutamol bronchodilator
Mucolytic: IE dornase alfa + Hypertonic Saline
IF PSEUDOMONAS COLONISED : Tobramycin
PROPHYLAXIS: Azithromycin 250mg TD
IBUPROFEN SLOWS LUNG DECLINE IN >6YOs
CFTR modulators:
Ivacaftor: F508del mutation.
Lumacaftor/ivacaftor OR Tezacaftor/ivacaftor : F508del mutation.
Ivacaftor increases CFTR activity
Lumacaftor moves CFTR to lumen [apical membrane]
LUNG TRANSPLANTATION
_____________________
If meconium ileus at birth:
Lactulose -> surgery + NG decompression in the meantime
_____________________
GI DISEASE
Pancreatic enzyme replacement (pancreatin)
GORD Rx
Vitamin replacement
ursodeoxycholic acid if LIVER involvement
Prognosis of CF
This is a genetic disease for which there is no cure. However, the outlook for patients with this condition has greatly improved. In the past 50 years, the mean age of survival has risen from infancy/school age to almost 40 years old.
Complications of CF
Distal intestinal OB Pneumo/haemothorax Intussusception Chronic resp acidosis Delayed puberty Acute resp failure Cor pulmonale DM metlitus Osteoporosis Depression/anxiety Chronic sinusitis Failure to thrive Nasal polyps Rectal prolapse
A 1-year-old child presents with failure to thrive. By history, the child was born at the 50th percentile for weight, but has crossed multiple percentile lines despite having a ravenous appetite. The child has more bowel movements per day than other children of the same age, and the stools often look shiny and have an unusually foul smell. In addition, the child has been treated with multiple courses of antibiotics for a persistent, wet cough. On measurement, the child is small for age, with weight and length below the third percentile
CF
Patients with pancreatic-sufficient CF may present with chronic sinusitis with involvement of all the sinuses, recurrent pancreatitis, and/or appendicitis. Older males may present with infertility due to congenital bilateral absence of the vas deferens. The pathological finding of inspissated, haematoxylin-staining material within the crypts of the appendix is pathognomonic.
CF
Define developmental dysplasia of the hip
The term developmental dysplasia of the hip (DDH) represents a spectrum of conditions affecting the proximal femur and acetabulum, ranging from acetabular immaturity to hip subluxation and frank hip dislocation. In true DDH, the femoral head has a persistently abnormal anatomical relationship with the pelvic acetabulum, which leads to abnormal bony development that can ultimately result in premature arthritis and significant disability. This is distinct from transient dysplasia, which represents acetabular immaturity in which the anatomical relationship stabilises and normalises over a period of weeks to months.
There is a high rate of spontaneous resolution of hips identified as abnormal by both clinical examination and ultrasound in the neonatal period. Frank dislocations and persistently abnormal examinations should be referred to a paediatric orthopaedist.
Epidemiology of developmental dysplasia of the hip
A recent UK review of neonatal screening for DDH suggested that the incidence of DDH is generally given as 1-2 per 1000 live births.
The reported prevalence of DDH has increased significantly since the advent of clinical and sonographic screening, suggesting the possibility of overdiagnosis.
DDH is more common in girls, with a 4-fold increased risk compared with boys.
W>B
The condition is also more common in infants who are breech at or near-term delivery.
Aetiology of developmental dysplasia of the hip
Although the aetiology of DDH has not been rigorously evaluated, it is generally believed that genetic, hormonal, and/or mechanical factors play a role. Observational studies have shown that infants with a family history of DDH have an increased relative risk of the condition; however, most cases occur in children with no family history. It is believed that those with DDH have increased ligamentous laxity, and that this is most pronounced at the time of delivery. Intrauterine confinement and positioning may play a role given that infants born breech, first-born infants, and large infants have been shown to have a higher relative risk of the condition.
RFs of developmental dysplasia of the hip
STRONG
Female
FHx
Breech
WEAK
Postural deformity eg congenital muscular torticollis
Restricted intraauterine space - oligohydramnios, macrosomia, multiple gestation
Sx of developmental dysplasia of the hip
ORTOLANI TEST
The hip is flexed to 90° and abducted, with the examiner’s fingers are placed laterally over the greater trochanter or hip joint. The examiner then uses anterior pressure over the trochanter in an attempt to identify a dislocated hip that is relocatable.
Should refer to a paediatric orthopaedist if frank instability is appreciated.
BARLOW TEST
The hip is flexed to 90° and adducted, the examiner’s hand is placed on the knee, and posterior pressure is placed through the hip in an attempt to identify dislocatable hips.
Should refer to a paediatric orthopaedist if frank instability is appreciated.
Limited hip abduction
UNCOMMON
Abnormal positioning of leg or delayed crawling
Toe walking
Ix of developmental dysplasia of the hip
<6 months:
US hips - subluxation on provocative testing; abnormal relationship between femoral head and acetabulum
> 6 months:
Beyond 6 months of age, hip radiographs are typically preferred over ultrasound.
- abnormal relationship between femoral head and acetabulum (assessed by acetabular index, Shenton’s line, ossification of femoral head)
Rx of developmental dysplasia of the hip
Hip dysplasia/subluxation
1) - MILD DYSPLASIA OBSERVE WITH MONTHLY SCANS AS MANY IMPROVE
- SUBLUX for 3w IN A NEONATE = warrants Rx as dislocated hip
2) If dysplasia persists or worsens, consideration is given to use of a Pavlik harness to enhance optimum hip development.
DISLOCATION:
1) CLOSED reduction with SPICA casting
2) OPEN reduction with SPICA casting
Open reduction is where the fracture fragments are exposed surgically by dissecting the tissues
Closed reduction is the manipulation of the bone fragments without surgical exposure of the fragments.
> 6YO
- will need Salvage osteotomies
Prognosis of developmental dysplasia of the hip
Prognosis is dependent upon the age at presentation, the extent of treatment needed, and the occurrence of complications. The success of closed or open reduction of a hip dislocation depends upon the remodelling potential of a child’s femoral head and acetabulum to achieve an adequately spherical shape with adequate coverage. This remodelling potential is more reliable in younger children, and within the first 12 to 18 months after reduction is achieved. The remodelling potential remains but is likely reduced during the subsequent years of growth.
For patients presenting as infants and using the Pavlik harness, success rates approach 90% and complication rates are considered low. Rates of avascular necrosis range between 1% to 3% for the majority of hips treated. Femoral nerve palsy has an incidence of 2.5% and typically is transient, resolving once the harness is removed
Complications of developmental dysplasia of the hip
Rx related avascular necrosis Residual dysplasia Degenerative joint disease Pavlik Harness disease Limb length inequality Knee valgus Back pain Rx related nerve palsy
Define congenital adrenal hyperplasia
AKA no cortisol OR aldosterone
Congenital adrenal hyperplasia (CAH) is a family of inherited enzyme deficiencies that impair normal corticosteroid synthesis by the adrenal cortex. The most common enzyme deficiency is 21-hydroxylase deficiency, which accounts for over 90% of cases. CAH due to 21-hydroxylase deficiency can be classified as either classical (simple virilising or salt-wasting types) or non-classical.
CAH is suspected in females who are virilised at birth, who become virilised postnatally, or who have precocious puberty or adrenarche.
Males with virilisation in childhood and infants of either sex with a salt-wasting crisis in the first 4 weeks of life are likely to be affected with CAH.
Serum concentrations of delta-androstenedione and progesterone are increased in males and females with 21-hydroxylase-deficient CAH.
Serum concentrations of testosterone and adrenal androgen precursors are increased in affected females and prepubertal males.
What is the epidemiology of congenital adrenal hyperplasia?
Screening studies indicate a worldwide incidence of classical 21-hydroxylase-deficient CAH as 1 in 13,000 to 1 in 54,000 live births.
The prevalence frequency of non-classical 21-hydroxylase-deficient CAH is considerably higher, at 1 in 1000, in white populations, with an even higher frequency among selected ethnic groups - most notably, Ashkenazi Jews.
What is the aetiology of congenital adrenal hyperplasia?
CAH is an autosomal recessive disorder, and the gene encoding 21-hydroxylase enzyme, CYP21A2, is mapped to the short arm of chromosome 6 (6p21.3). To date, more than 100 mutations have been described, including point mutations, small deletions, small insertions, and complex rearrangements of the gene.
The production of cortisol occurs in the zona fasciculata of the adrenal cortex through 5 enzymatic steps.
RFs for congenital adrenal hyperplasia
Genetic predisposition
Sx of congenital adrenal hyperplasia
COMMON
WL Failure to thrive Vomiting Hypotension Ambiguous genitalia - Prader Score: degree of virilisation of the external genitalia of females as proposed by Prader. Hyperpigmentation Poor feeding Irregular menses Infertility Male pattern baldness in females Short stature Precocious puberty PCOS Hirstuism
UNCOMMON
Severe cystic acne
Ix for congenital adrenal hyperplasia
serum 17-hydroxyprogesterone - HIGH if 21OH classical
serum 11-deoxycortisol - HIGH if 11-beta-hydroxylase deficiency
serum chemistry - hyponatraemic, hyperkalaemic, metabolic acidosis; azotaemia
microfilter paper radioimmunoassay for 17-hydroxyprogesterone - H if 21-OH def
Genetic analysis - mutation and autosomal recessive inheritance pattern
ACTH stimulation testing
Renin high in salt wasting
Pelvic USS not usually indicated but does show enlarged glands with classical CAH
Rx for congenital adrenal hyperplasia
Mother of potentially affected foetus: antenatal Dexamethasone
DURING ANY ACUTE STRESS EG SURGERY / ILLNESS ECT
MUST GIVE ADDITIONAL glucocorticoid
NORMAL DAY-DAY
Hydrocortison 10-15mg/square meter body
SALT WASTING FORM
May need fludrocortisone + NaCl
IF GROWTH AFFECTED:
Somatotropin
Genital surgery
Prognosis of congenital adrenal hyperplasia
Improved care has resulted in a good prognosis and normal life expectancy. The main concerns in children are the preservation of fertility and healthy sexual function, and maintenance of general wellbeing. This includes bone health and the assessment and management of cardiovascular disease risk.
Changes in childhood play behaviour correlate with reduced female gender satisfaction and reduced heterosexual interest in adulthood. The rates of bisexual and homosexual orientation were increased in women with all forms of 21-hydroxylase-deficient CAH. They were found to correlate with the degree of antenatal androgenisation. Affected adult females are more likely to have gender dysphoria, and experience less heterosexual interest and reduced satisfaction with the assignment to the female sex. Males affected with CAH do not show a general alteration in childhood play behaviour, core gender identity, and sexual orientation.
Complications of congenital adrenal hyperplasia
Adrenal crisis
Testicular adrenal rests
Short stature
A 7-day-old newborn boy is brought into hospital with vomiting and poor feeding. The infant is dehydrated and tachycardic. Electrolytes reveal hyponatraemia and hyperkalaemia. The patient demonstrates signs of shock. On genital exam, a phallic structure is noted with hyperpigmentation of the scrotum, but no testes are palpated.
Congenital adrenal hyperplasia
Under-virilisation of newborn males can be seen in 3-beta-hydroxysteroid dehydrogenase deficiency and 17-alpha-hydroxylase deficiency.
A 16-year-old, previously healthy female presents with acne, hirsutism, and irregular menses. Her pubertal history reveals breast development at 9 years of age and pubic hair development at 7 years of age, and she reported 1 episode of vaginal spotting at approximately 11.5 years of age. A family history indicates some female relatives with symptoms of infertility, irregular menses, polycystic ovary syndrome, or alopecia. She is significantly shorter than her target height.
Congenital adrenal hyperplasia
Define febrile seizure
The American Academy of Pediatrics announced a standard definition of febrile seizures as a seizure occurring in febrile children between the ages of 6 and 60 months who do not have an intracranial infection, metabolic disturbance, or history of afebrile seizures.
The first occurrence is usually before 3 years of age, but is infrequent in children younger than 6 months. Febrile seizures have a peak incidence at 18 months of age and are most common between 6 months and 5 years. Most febrile seizures are simple with approximately 20% to 30% being complex.
Febrile seizures are usually self-limiting; an increased risk of developing epilepsy is low except in a small number of cases.
Most febrile seizures resolve spontaneously and quickly, and do not require acute or long-term anticonvulsant treatment.
Epidemiology of febrile seizure
2-5% cumulative incidence
1.6M:1F
Aetiology of febrile seizure
Viral infections triggering fever are most common, with bacteraemia as an infrequent cause.
(HHV)-6 infection
Influenza A virus is the most common cause in Asian countries
Febrile seizures are dependent upon a threshold temperature and this seems to vary from one individual to another.
The American Academy of Pediatrics considers fever to be any increase of the body’s external temperature over 38ºC
RFs for febrile seizure
STRONG Fever Young age FHx febrile seizure Viral/bacterial infection outside the central nervous system
WEAK Male Vaccinations Antenatal exposure to nicotine Iron deficiency Complications of pregnancy, labour + delivery
Sx of febrile seizure
RFs Febrile illness >38.3 Seizure: - LOC - Generalised tonic-clonic - Consciousness usually recovered quickly within 30mins - No sequelae - Less commonly, some seizures are prolonged, focal or multiple, and recovery of consciousness is delayed.
NORMAL post-ictal exam
Ix for febrile seizure
Diagnosis is clinical, however alternative causes must be ruled out:
Lumbar puncture: normal cells, protein, glucose Viral studies: may be +ve Blood culture: may be +ve FBC Serum glucose: usually N
Rx for febrile seizure
ANTIPYRETIC
- paracetamol 10-15mg/kg 4-6hrly
- ibuprofen 5-10 mg/kg orally every 6-8 hours
Anticonvulsant:
diazepam, midazolam, lorazepam, fosphenytoin
Prognosis of febrile seizure
Prognosis is generally favourable.
Febrile seizures recur in approximately 30% of children during subsequent febrile illnesses.
Most recurrences occur within 2 years. The risk of non-febrile seizures and epilepsy developing after simple febrile seizures is 5% or less. However, after complex febrile seizures, the risk of developing epilepsy is 10% to 20%.
Complications of febrile seizure
Todd's paralysis (temporary hemiparesis Non-febrile seizure Epilepsy Focal epilepsy Mesial Temporal sclerosis Behaviour and cognitive disorders
A previously healthy and developmentally normal 18-month-old boy presents to the emergency department by ambulance after his parents witnessed a seizure. The parents report the boy had a febrile illness with mild upper respiratory symptoms and they treated him with paracetamol and ibuprofen at home. The child then began to have frequent jerking movements of all limbs. The rectal temperature was 39.5°C (103.1°F). The parents called the emergency services, and a paracetamol suppository was administered during transport to the emergency department. The jerking stopped after approximately 5 minutes. Afterwards, the child was sleepy but responsive to verbal stimulation. Examination revealed a diffuse erythematous maculopapular rash and a normal mental and neurological status.
Febrile seizure
A 10-month-old girl is brought to the emergency department with a history of recurrent right arm and leg jerking followed by prolonged sleepiness. The parents report a 2-day history of fever with chest congestion and irritability. The child is admitted to hospital for neurological evaluation.
Febrile seizure
Febrile seizures before 6 months of age in a child with a relatively low fever are atypical, and require a full investigation to exclude acute bacterial meningitis or other CNS pathology. Another atypical presentation is a child with a prolonged focal (complex) febrile seizure who has Todd’s paralysis (transient hemiparesis) on recovery of consciousness. In one prospective series of 95 patients with febrile seizure, 2% had Todd’s paralysis. [7] Febrile status epilepticus is a prolonged or recurring seizure with fever and without recovery of consciousness between episodes. Classically, the duration is 30 minutes or more; some include seizures of shorter duration (10 minutes or more). Tonic-clonic generalised status is most common. In a large prospective, controlled study, [8] patients with febrile seizure status were more likely to have neurological abnormalities, a history of neonatal seizures, and a family history of epilepsy. A prospective multi-centre study of children with febrile status epilepticus (FEBSTAT study) found a median age of 1.3 years, and seizures that were most often focal, partial and long, lasting a median of 68 minutes. [9] Febrile status epilepticus was frequently the first febrile seizure, and status was unrecognised in the emergency department.
Febrile seizure
Define viral gastroenteritis in children
Viral gastroenteritis is an infection of the GI tract by a virus, usually rotavirus. It is usually self-limiting, but if untreated may result in morbidity and mortality secondary to dehydration, electrolyte imbalance, and metabolic acidosis.
Associated with substantial morbidity in developed countries. Continues to be a cause of significant mortality in developing countries.
Presents with vomiting and diarrhoea (with or without fever, nausea, or abdominal pain).
Dehydration, which may be associated with electrolyte imbalance and metabolic acidosis, is the most frequent and dangerous complication.
Oral rehydration therapy is the mainstay of treatment.
Self-limiting disease with excellent prognosis if correctly treated.
What is the aetiology of viral gastroenteritis in children
Viruses account for approximately 70% to 87% of episodes of acute gastroenteritis in children, with Rotavirus being the most common identifiable cause.
Other viral pathogens include caliciviruses, astroviruses, and adenoviruses.
Other viruses such as coronaviruses, parvoviruses, and picobirnaviruses have also been linked to human gastroenteritis.
What is the epidemiology of viral gastroenteritis in children
Worldwide, 3 billion to 5 billion cases of acute gastroenteritis occur each year in children <5 years, resulting in 1.5 million to 2.5 million deaths.
The average child <5 years of age experiences 2.2 diarrhoeal episodes per year in developed countries, and this rate in developing countries is significantly higher.
Viral gastroenteritis is spread mainly by the faecal-oral route; this may occur through contaminated hands, utensils, food, and drink. Gastroenteritis is associated with poverty and poor environmental or personal hygiene.
Spread among family members is common, and outbreaks may occur in day-care centres and hospitals. Several epidemiological studies have suggested that rotavirus infection can be transmitted by respiratory droplets.
In temperate regions, rotavirus gastroenteritis has a marked seasonality, peaking during the cold months.
List RFs for viral gastroenteritis in children
Age <5 Poor personal hygiene Exposure Day-care/nursery Winter months Poverty Lack of immunisation Lack of breast feeding Immunodeficiency
Sx of viral gastroenteritis in children
COMMON RFs Vomiting Non-bloody diarrhoea Hyperactive bowel sounds Low grade fever Abdominal pain Evidence of dehydration Decreased bodyweight Non-distended abdomen Abdominal tenderness
UNCOMMON
Mucus in stool
Ix for viral gastroenteritis in children
CLINICAL DIAGNOSIS
Measurements of serum electrolytes, urea, and creatinine are usually not necessary to assess children with acute gastroenteritis, as the results do not change the therapeutic strategy.
CONSIDER U+E - N unless severe deydration FBC - if worried re sepsis Stool culture - Negative Enzyme immunoassay for detection of viral antigen - CAN detect offending pathogen but is pointless rly
Rx for viral gastroenteritis in children
Oral rehydration solutions
Ondansetron - if vomiting prevents ORT
IV fluids
Prognosis of viral gastroenteritis in children
With proper treatment, the prognosis is excellent. Viral gastroenteritis is self-limited; the typical episode lasts 3 to 7 days. Prolonged diarrhoea may indicate secondary disaccharidase deficiency.
Worldwide, acute gastroenteritis accounts for 1.5 to 2.5 million deaths annually. Most deaths occur in developing countries.
Complications of viral gastroenteritis in children
Volume depletion U+E imbalance Metabolic acidosis - Often complicates acute diarrhoea, as a result of faecal losses of bicarbonate and impaired renal excretion of hydrogen ions. Hypoglycaemia Viral antigenaemia Secondary bacteraemia Afebrile seizure Shock
A 6-month-old boy presents with a history of low-grade fever of 37.7°C (100°F) and non-bilious vomiting (6 episodes per day) for 2 days. This is followed by diarrhoea (15 stools per day) 24 hours later. Stools are watery and do not contain blood or mucus. His mother is unsure of the urine output but the infant has lost 0.7 kg (1.5 lb) of weight in the past few days. His mother also reports that the infant is more irritable and has decreased activity level since the onset of the illness. The infant has never been breast-fed and has not had rotavirus vaccination. He attends a day-care centre. His 2-year-old brother who attends the same day-care centre has just got over an ‘upset tummy’.
Viral gasteroenteritis
Define hirschsprung’s disease
A congenital condition characterised by partial or complete colonic functional obstruction associated with the absence of ganglion cells.
Because of the aganglionosis, the lumen is tonically contracted, causing a functional obstruction. The aganglionic portion of the colon is always located distally, but the length of the segment varies. This determines the varied manifestations of the disease.
The vast majority of patients present in the newborn period up to 1 year of age. Diagnosis later in life occurs rarely.
Presents with vomiting, abdominal distension, and/or enterocolitis.
May be associated with Down’s syndrome and multiple endocrine neoplasia type IIA.
Definitive diagnosis is with a rectal biopsy.
Initial treatment is bowel irrigation, followed by a definitive surgical treatment, either transanally alone or with laparoscopic assistance. Rarely, colostomy or ileostomy is required to manage severe enterocolitis, with definitive pull-through delayed.
Irrigations often do not work for patients with total colonic Hirschsprung’s disease. Total colectomy with ileo-procto anastomosis and a protective ileostomy is recommended at the time of diagnosis. The ileostomy is closed after the child is toilet trained for urine and can accept rectal irrigations.
Aetiology of hirschsprung’s disease
There have been new developments in determining associated genetic defects. Hirschsprung’s disease is thought to be due to a deletion in the long arm of chromosome 10. The location of this mutation is between 10q11.2 and 10q21.22.
This deletion seems to overlap the region of the RET (rearranged during transfection) proto-oncogene. Multiple endocrine neoplasia type IIA is also associated with a deletion of this proto-oncogene.
The absence of ganglion cells and the presence of hypertrophic nerves, as well as an increase in the enzyme acetylcholinesterase, are the keys to pathological diagnosis of the dysfunctional bowel segment.
RFs of hirschsprung’s disease
STRONG
Downs syndrome
WEAK
MEN2A
Male
Gene mutations 10/21
Epidemiology of hirschsprung’s disease
1 in 5k births
White >
M>F
Approximately 5% to 32% of all individuals affected with the disease have an associated congenital anomaly.
Sx of hirschsprung’s disease
COMMON Vomiting Explosive passage of liquid and foul stools Abdo distension Delayed passageof meconium Fever Failure to thrive
UNCOMMON
Septic shock
Ix of hirschsprung’s disease
Plain Xray - air-fluid levels present; dilated colon
Contrast enema - contracted distal bowel and dilated proximal bowel, with demonstration of the location of the transition zone in between
Rectal biopsy - absence of ganglion cells and presence of thickened (i.e., >40 microns), non-myelinated nerves; increased activity of acetylcholinesterase; absence of nicotinamide adenine dinucleotide phosphate diaphorase-containing neurons; increase in the amount of acetylcholinesterase-containing nerve bundles
Rx of hirschsprung’s disease
Bowel irrigation
IV fluids/ABx
Decompression by colostomy or ileostomy
Definitive surgery
Total colonic aganglionosis -> cannot have bowel irrigation: only ileostomy
Complications of hirschsprung’s disease
Ischaemic enterocolitis Post-op enteroC Post op constipation/diarrhoea Duhamel rectal pouch Faecal incontinence post surgery Pseudo-incontinence Fistula/pelvic abscess
Prognosis of hirschsprung’s disease
The outcome for typical (rectosigmoid) disease is good. Enterocolitis can occur postoperatively but usually disappears after the first year of life. Soiling is usually treatable with the correct medical management.
True faecal incontinence is avoidable with proper technique preserving the sphincters and anal canal.
Patients who have had pull-through and suffer from incapacity to empty the colon may have a retained piece of colon with transition zone, or a twisted pull-through.
A 4-day-old baby presents with bilious vomiting and significant abdominal distension.
Hirschsprungs
A 1-month-old baby boy presents with feeding intolerance, abdominal distension, and copious diarrhoea.
Hirschsprungs
Diarrhoea with vomiting, fever, and abdominal distension (enterocolitis) are common presenting symptoms. Older children may have persistent abdominal distension and/or failure to thrive as a manifestation of chronic moderate enterocolitis. When the abdomen is distended, the infant can become quite ill from sepsis, hypovolaemia, and endotoxin-related shock due to bacterial translocation.
Hirschsprungs
Define Henoch-Schonlein purpura
Henoch-Schonlein purpura (HSP) is the most common vasculitis of childhood and affects the small vessels. HSP is characterised by the classic tetrad of rash, abdominal pain, arthritis/arthralgia, and glomerulonephritis.
Most common vasculitis of childhood.
Rash of palpable purpura is present in all cases.
Most cases resolve with symptomatic treatment.
Complications can occur, with renal failure being the most common cause of death.
Epidemiology of Henoch-Schonlein purpura
Normally 3-15yo 50% of cases = before 3YO 22.1 cases per 100,000 population Asian>W>B HSP can follow a URTI, and as a result it mainly occurs in the autumn, winter, and spring.
Aetiology of Henoch-Schonlein purpura
The underlying cause of HSP remains unknown. It is an immune-mediated vasculitis, with a variety of infectious and chemical triggers having been proposed as a cause. Many cases of HSP occur after a URTI, especially streptococcal infections.
HSP is a small-vessel leukocytoclastic vasculitis characterised by the tissue deposition of IgA-containing immune complexes within affected organs. Skin biopsies of lesions show neutrophils and monocytes as the predominant cell types. Fluorescence microscopy shows IgA, C3, and fibrin deposition within the small vessels.
The pathogenesis of this disease is similar to that of IgA nephropathy.
RFs for Henoch-Schonlein purpura
M
3-15
Hx URTI
Some medicines: (e.g., penicillin, cefaclor, minocycline, hydralazine, and phenytoin)
Sx of Henoch-Schonlein purpura
COMMON
RFs
Arthralgias
Abdominal pain
Rash - erythematous, non-blanching, usually lower extremities
Sx renal disease: can vary from mild (i.e., asymptomatic haematuria and/or proteinuria) to severe (i.e., rapidly progressive nephritis, nephrotic syndrome, and renal failure). It can produce microscopic haematuria, proteinuria, and RBC casts.
UNCOMMON Headaches Seizures Pulmonary haemorrhage Scrotal pain/swelling
Ix of Henoch-Schonlein purpura
Urinlaysis: assess renal involvement - may show RBCs, proteinuria, or casts
coagulation studies - N
Rx for Henoch-Schonlein purpura
No nephrotic syndrome:
Analgesics - ibu/para
Prednisolone if severe oedema / pain
IF NEPHROTIC RANGE PROTEIN
- Corticosteroid + immunosuppressant (azathioprine/ciclophosphamide)
May need plasmapheresis/renal dialysis/transplantation if very severe
Prognosis of Henoch-Schonlein purpura
The outcome for patients with HSP is usually excellent. Complete recovery occurs in 94% of children and 89% of adults.
One third of patients may have a recurrence within 4 months, but the subsequent episode is generally milder; recurrences are more common in patients with nephritis.
Complications of Henoch-Schonlein purpura
Pulmonary haemorrhage GI haemorrhage/intussusception Renal impairment ESRD CNS complications Uveitis
A 5-year-old boy is brought in by his mother. He presents with a 4-day history of a rash on his lower extremities, mild abdominal cramping, and diffuse joint pain. His mother reports that he was recently treated for a URTI.
Henoch-Schonlein purpura
While the rash occurs in all patients, the other classic symptoms do not always manifest. Scrotal pain and swelling may occur in about 13% of boys with HSP.
Other organ systems may be involved, including the CNS (patients may present with headaches or seizures) and pulmonary system (patients may present with pulmonary haemorrhage).
HSP
Define Kawasaki disease
Kawasaki disease (KD) is an acute, febrile, self-limiting, systemic vasculitis of unknown origin that almost exclusively affects young children. In an immunogenetically pre-disposed host, one or more infectious agents may play a role in triggering the clinical manifestations of the disease. Clinically, it is characterised by fever, polymorphic rash, conjunctivitis, mucosal erythema with strawberry tongue, induration of the hands and feet, and unilateral cervical lymphadenopathy. Morbidity and mortality depend on coronary aneurysms that develop in 20% to 25% of untreated patients. KD is the leading cause of acquired heart disease in children under 5 years of age in the US and other developed countries.
Acute febrile illness lasting 5 or more days.
Typical signs include fever, polymorphic rash, injected eyes, and mucosal erythema with strawberry tongue.
Swelling and erythema of the hands and feet occur in the acute stage, followed by desquamation in the second week.
Unilateral non-purulent cervical lymphadenopathy is present in about 40% of cases.
Coronary aneurysms develop in 20% to 25% of untreated patients.
Standard treatment includes intravenous immunoglobulin and/or aspirin. In resistant cases, corticosteroids or a tumour necrosis factor (TNF)-alpha inhibitor may be necessary.
Epidemiology of Kawasaki disease
13-24month usually Rare in 1st 6month 80% occur before 5 Asian > black > white origin M>F
Aetiology of Kawasaki disease
The cause of KD remains unknown. However, the following observations suggest that this disease is triggered by an unknown infectious agent.
Clinical picture: KD overlaps with infectious diseases such as scarlet fever and adenoviral infection.
Seasonal occurrence: in the US and other geographic areas, the peak occurrence of KD is in the winter/spring, similar to that seen in numerous viral diseases.
Epidemics with clear epicentre: temporal clusters have been reported in the US, Japan, and worldwide. Moreover, in Japan, outbreaks have been observed to start in one area and spread throughout the country over a period of 3 months.
Age at onset: peak incidence is in the toddler age group; 80% of the cases are in infants under 5 years old, and the rarity of cases under 3 months of age suggests protective transplacental antibodies
RFs for Kawasaki disease
Asian ancestry
3month - 4yo
Male
Sx of Kawasaki disease
COMMON Polymorphous rash - diffuse, maculopapular erythematous rash. Occasionally scarlatiniform- or erythema multiforme-type rash with target lesions on the arm and trunk. Groin erythema or desquamation and fine pustules over extensor surfaces of extremities can occur. Conjunctival injection - aka RED eyes Mucositis - dry, erythematous, fissured lips that bleed easily, erythema of the oral and pharyngeal mucosa, and strawberry tongue with prominent papillae and erythema Skin changes on extremities Unilateral enlarged cervical LNs Coronary artery aneurysms Fever + EXTREME irritability
UNCOMMON Pericarditis with effusion Congestive Heart Failure Joint pain/oedema Neurological manifestations: aseptic meningitis/facial nerve palsy
VERY RARE
GI manifestations
Urological manifestations
Other derm manifestations
Ix of Kawasaki disease
FBC - Normochromic Anaemia, leukocytosis, thrombocytosis (plts most important diagnostically)
ESR - Raised
CRP - raised
Echo - coronary artery dilatations or aneurysms
CONSIDER Urinalysis - sterile pyruria LFTs - raised LFTS, low alb ECG - rule out MI CxR - may see cardiomegaly LP - if Sx meningitis (may have aseptic men)
Rx of Kawasaki disease
1st: IVIG \+ High dose aspirin 25mg dose QTD 2nd: Prednisolone/methylpred + aspirin 2nd: Infliximab 5mg/kg single dose + aspirin 3rd: Ciclosporin + aspirin
RISK FOR MI
Low risk - CV assessment
High risk - Aspirin + LMWH + adjunct: clopidogrel + BB
Prognosis of Kawasaki disease
Kawasaki disease (KD) is an acute, self-limiting illness. The immediate outcome has improved dramatically, with a decrease in the frequency of coronary artery aneurysms to less than 3% following the introduction of intravenous immunoglobulin (IVIG) therapy. Overall, the mortality rate is less than 0.5%.
However, in untreated patients it is associated with significant morbidity and mortality. Delayed diagnosis, particularly of incomplete KD, and KD in very young children, poses a challenge as these patients have a high risk of aneurysms. The long-term prognosis of children with giant aneurysms remains a concern due to their associated risk of ischaemia or thrombosis. Management of such patients includes thromboprophylaxis and careful identification of evolving stenoses. Selected patients may need invasive revascularisation procedures.
Complications of Kawasaki disease
Myocarditis Pericarditis Coronary aneurysms Valvulitis Arthritis Mild hepatic dys Gallbladder disease Pneumonitis Aseptic meningitis Coronary thrombosis ACS Systemic artery aneurysms Extremity gangrene Bowel ischaemia and necrosis
A previously healthy 1-year-old girl was admitted to a children’s hospital with a 7-day history of spiking fever up to 39.5°C (103°F). Three days after the onset of fever she developed left-sided neck swelling and nappy rash, and became progressively fussy and irritable. She was seen in an emergency department, diagnosed with cervical adenitis, and sent home on oral antibiotics. The mother noted continued irritability, high fever, and decreased oral intake. On subsequent admission she was extremely irritable, with a temperature of 38.9°C (102°F), heart rate of 140 beats per minute, respiratory rate of 40 breaths per minute, and blood pressure 110/54 mmHg. There were no signs of nuchal rigidity. Both palpebral and bulbar conjunctivae were deep red and injected, lips were dry and crusted, the oropharynx hyperaemic with some areas of ulcerated mucosa, and the tongue papillae were enlarged and red (strawberry appearance). Examination of the neck revealed a mildly tender left unilateral mass, measuring 4 cm. The skin showed a generalised polymorphous, erythematous, macular, blanching rash, in addition to severely red and desquamated perineal region. Her extremities, especially palms and soles, were swollen, red, and mildly tender.
Kawaski Disease
Some cases do not fulfil well-accepted criteria and are called incomplete/atypical Kawasaki disease (KD). This presentation is more common among children <1 year of age, who are at higher risk for the development of coronary artery aneurysms (CAAs) if untreated. In these cases of insufficient clinical criteria, presence of coronary abnormalities or CAAs must be shown on echocardiogram. A patient can present with prolonged fever (>5 days) and 2 or 3 of the classic criteria such as generalised polymorphous skin rash and red injected eyes. Infantile periarteritis nodosa is part of the spectrum of KD. The coronary artery aneurysmal lesions are clinically and pathologically indistinguishable from those seen in KD. Acute febrile mucocutaneous lymph node syndrome was initially described before KD was recognised. It is now part of the spectrum of KD.
Kawaski Disease
Define intussusception
Intussusception is a common cause of intestinal obstruction in young children and is defined as the prolapse of one part of intestine into the lumen of the adjoining distal part.
This condition most often occurs in the ileocaecal region. The lead point of the intussusception is most often an enlarged lymph node (Peyer’s patch) in the terminal ileum. Occasionally, the lead point is an anatomic abnormality of the intestine (a ‘pathological lead point’). Conditions resulting in pathological lead points include luminal polyps, malignant tumours (including lymphoma), and benign mass lesions such as lipomata, Meckel’s diverticulum, Henoch-Schonlein purpura, and enteric duplication cysts.
Intussusception is clinically important. It results in venous obstruction and bowel-wall oedema that can progress to bowel necrosis, perforation, and, rarely, death.
Most commonly occurs in infants aged between 3 and 12 months, with a peak at the age of approximately 9 months.
Presentation often includes colicky abdominal pain, flexing of the legs, fever, lethargy, and vomiting, with blood in the stool in some cases.
When there is clinical suspicion, imaging has a valuable role. Plain abdominal x-ray may reveal an intestinal obstruction and paucity of wind in the right lower quadrant. The presence or absence of free gas will influence subsequent management; abdominal ultrasound undertaken by an experienced radiologist will usually establish the diagnosis; contrast enema (most often air but may be liquid contrast) is the most specific and sensitive test for diagnosis. As well as being used therapeutically, it can also be used diagnostically where doubt remains.
The pathological location is typically ileocaecal.
Treatment involves reduction by contrast enema. Air is likely to be more effective and safer than liquid; in cases where this is unsuccessful or where peritonitis exits, surgery is required. Open reduction is then performed in uncomplicated cases, and intestinal resection for cases complicated by bowel necrosis and perforation.
Epidemiology of intussusception
18-56 per 100k
5-7m normally
75% <12
Aetiology of intussusception
The aetiology of most cases of intussusception is unclear but is likely to be related to hyperplasia of Peyer’s patches and lymphoid tissue in the intestinal wall resulting from antecedent viral infection.
These enlarged lymph nodes have been suggested as the lead point in idiopathic intussusception. This review will focus on idiopathic intussusception in infants.
Pathological lead points are anatomical abnormalities of the intestine, such as luminal polyps, malignant tumours (including lymphoma), and benign mass lesions (e.g., lipomata, Meckel’s diverticulum, Henoch-Schonlein purpura, and enteric duplication cysts).
Intussusception is the telescoping of one portion of the intestine (the intussusceptum) into the lumen of the intestine immediately distal to it (the intussuscipiens). The mesentery is dragged alongside the proximal bowel wall into the distal lumen resulting in obstruction of venous return. Oedema, mucosal bleeding, and increased pressure result. If arterial flow becomes compromised, ischaemia, necrosis, and perforation can occur.
RFs of intussusception
STRONG
Male
6-12m
WEAK
Antecedent viral illness
1st gen rotavirus vaccination
Sx of intussusception
COMMON
RFs
Male
6-12m
Vomiting (may be bilious/non bilious)
Lethargy / irritability IN BETWEEN WAVES of pain ( can be the only Sx)
Blood per rectum -> “redcurrent jelly stool”
Pallor
Palpable abdominal mass (usually RUQ/EPIgastrum)
UNCOMMON
Hypovolaemic shock
Poor feeding
Abdo distention
Ix of intussusception
Abdo plain Xray (always initial investigation if perforation or obstruction is suspected) - may appear normal; visible abdominal mass; abnormal wind pattern; air-fluid levels; dilated bowel loops; empty right lower quadrant; ‘target sign’; free intra-abdominal air (may be indicative of intestinal perforation as a complication of intussusception)
USS (higher sensitivitt/spec) - tissue mass; ‘target sign’; doughnut sign; multiple concentric ring sign; crescent-in-doughnut sign; pseudokidney sign; sandwich sign; abnormal Doppler flow
Can do diagnostic enema - meniscus sign; coiled spring sign
Rx of intussusception
STABLE
Fluid resus and contrast enema reduction
BS ABx - ie gentamycin/vanc/piptaz
UNSTABLE - shock/perforation/peritonitis/bowel wall necrosis
Fluid resus + urgent surgical reduction
Prognosis of intussusception
The recurrence rate of intussusception after contrast enema (air or contrast reagent) reduction is approximately 10% and does not significantly differ based on the type of contrast reduction performed.
Surgical reduction has been associated with a recurrence rate of 2% to 5%.
Mortality in intussusception is related to delay in presentation, septic shock, and inadequate fluid resuscitation.
Complications of intussusception
Intestinal ischaemia, necrosis and perforation
A 9-month-old boy presents to the emergency department with a 24-hour history of colicky abdominal pain, anorexia, fever, and progressive lethargy. Episodes of pain last 1 to 2 minutes marked by crying and drawing his knees to his chest, alternating with 20-minute pain-free periods where he behaves normally. The infant has vomited yellow material several times, which became green just prior to coming to hospital. Also, just prior to presentation, he had a bowel movement that appeared red, similar to redcurrant jelly. An abdominal examination reveals a distended, tender abdomen with pain out of proportion to examination, and a palpable right-sided abdominal mass. This could only be felt when he was settled between waves of pain, and was easiest to feel when he was lying on his side on his mother’s lap.
intussusception
The classic triad of intussusception includes colicky abdominal pain, redcurrant jelly stool, and a palpable abdominal mass. This triad is present in 20% to 40% of patients. In 60% of patients, at least 2 of these features are present at the time of presentation. [3] Lethargy has also been identified as a common presenting symptom of intussusception. [5] Symptoms that have been identified as independently predictive of intussusception include rectal bleeding, history of vomiting, and male sex. [6] Ileo-ileal intussusception can occur as a postoperative event, typically following abdominal surgery, especially resection of Wilm’s tumour or neuroblastoma, pull-through for Hirschsprung’s disease, and correction of malrotation.
intussusception
Define laryngomalacia
Laryngomalacia (LM) is a congenital abnormality that predisposes to dynamic supraglottic collapse during the inspiratory phase of respiration, resulting in intermittent upper airway obstruction and stridor. LM is the most common source of stridor in infants and the commonest congenital laryngeal anomaly.
Congenital abnormality of the larynx cartilage that predisposes to dynamic supraglottic collapse during the inspiratory phase of respiration, resulting in intermittent upper airway obstruction and stridor.
Most common laryngeal anomaly and most frequent congenital cause of stridor in infants.
Natural history is presentation in early infancy with maximal symptoms at 6 to 8 months before gradual improvement and spontaneous resolution within 12 to 24 months.
Presents with inspiratory stridor. Some patients have upper airway obstruction with associated feeding difficulties. Frequently associated with GORD.
Diagnosis is made by the clinical features and with flexible laryngeal endoscopy. Direct rigid laryngoscopy under anaesthesia may also be required. Possibility of additional airway lesions should be considered.
Treatment depends on disease severity; observation alone (with treatment of associated GORD) is appropriate for most cases. Endoscopic supraglottoplasty may be required for more severe disease. Tracheostomy and pressure-assisted ventilation are other possible therapies.
Epidemiology of laryngomalacia
60-70% of congenital laryneal anomalies
M:F 2:1
Usually presents in first few weeks of life
Aetiology of laryngomalacia
The aetiology of laryngomalacia (LM) is still not fully understood, although several theories have been proposed.
- Immaturity of cartilage
- Neuromuscular incoordination
- Abnormalities in supraglottic anatomy
RFs for laryngomalacia
STONG GORD Neurological abnormalities Laryngeal anatomical abnormalities Male Genetic syndrome disorder
Sx of laryngomalacia
Onset within two weeks of birth
Stridor
Resp distress - Clinical features include nasal flaring; suprasternal, intercostal, or subcostal recession; abdominal respiration; and tracheal tug. Sternal recession and/or pectus excavatum may be present.
Resolution of symptoms by age 2
NORMAL CRY - This serves to differentiate from other laryngeal conditions where abnormal or absent cry is a feature (vocal cord palsy, laryngeal web).
Feeding difficulties
UNCOMMON
WL
Hypotonia
Dysmorphic features
Ix for laryngomalacia
Flexible laryngoscopy - dynamic collapse of the supraglottic tissues on inspiration; visible narrowing and obstruction of the supraglottic airway; anatomical anomalies; evidence of GORD
Rx of laryngomalacia
Observation (typically resolves by 2yo)
GORD Rx - ranitidine/omeprazole
BIPAP can be used
SEVERE disease:
SURGICAL Rx - supraglottoplasty
Complications of laryngomalacia
GORD Life threatening airway OB Failure to thrive Aspiration Tracheostomy-related comps Pulmonary HTN
Prognosis of laryngomalacia
For most infants with laryngomalacia (LM) the prognosis is excellent. The natural course typically involves gradual worsening after onset, with subsequent spontaneous resolution. LM is most severe when the child is 6 to 8 months of age, before gradually improving spontaneously by 12 to 24 months of age.
A 3-month-old boy is referred by his paediatrician with intermittent inspiratory stridor and progressive feeding difficulties resulting in failure to thrive. The stridor has been present since he was 1 week old. His parents report that his noisy breathing is present more frequently and is particularly prominent when he is lying down, feeding, or crying. He has difficulty feeding and chokes frequently but has a normal cry. His mother reports that he feeds slowly and often has to stop several times during the feed to ‘gasp for breath’. He has recently been diagnosed with GORD.
laryngomalacia
A 6-week-old boy presents to the emergency department with inspiratory stridor, present since a few days after birth. The stridor is loud with associated mild suprasternal indrawing (tracheal tug). Otherwise, the infant is resting comfortably and is thriving (he is on the 75th centile for weight since birth).
laryngomalacia
Laryngomalacia (LM) is often detected in patients with concurrent neurological conditions or syndromes. Children with reduced tone of the larynx due to underlying neurological pathology often present with concurrent LM. These patients can be more difficult to treat due to their underlying hypotonia. A late-onset variant has also been described that gives feeding problems in toddlers, sleep apnoea in children, and intolerance of exercise in teenagers.
laryngomalacia
Define meconium aspiration syndrome
Meconium aspiration syndrome (MAS) is defined as respiratory distress in the newborn due to the presence of meconium in the trachea. It occurs exclusively in the immediate neonatal period. It also covers respiratory distress in an infant, born through meconium-stained amniotic fluid, that cannot be explained otherwise.
Defined as respiratory distress in the newborn due to the presence of meconium in the trachea.
Infants born through meconium-stained amniotic fluid are at risk of developing meconium aspiration syndrome, particularly in the presence of maternal and fetal risk factors.
Soon after birth, infants present with respiratory distress (tachypnoea, chest retractions, and hypoxia). Some may be asymptomatic and apparently vigorous at birth, and develop severe respiratory distress hours later. Signs of post-maturity, a strong risk factor, include green/yellow-coloured skin; long, stained nails; and dry, scaling skin.
Diagnosis is confirmed by chest x-ray.
Management is largely supportive. Infants should be monitored closely to ensure adequate oxygenation and ventilation.
In the presence of persistent pulmonary hypertension, inhaled nitric oxide and extracorporeal membrane oxygenation therapy may be required.
Epidemiology of meconium aspiration syndrome
MAS is a serious complication associated with meconium-stained amniotic fluid (MSAF). In general, MSAF is found in 7% to 22% of term deliveries, with higher rates (22% to 44%) in post-term deliveries (>42 weeks).
> MAS with gestational age
Induction of labour reduces chances
Aetiology of meconium aspiration syndrome
Meconium is the first stool of the newborn. It is a collection of secretions from the fetal intestine and is a sterile, thick, black-green, viscous material containing pancreatic and biliary secretions, swallowed fetal cells, hair, and vernix. If aspirated into the lungs it causes local inflammation of the airways and release of several cytokines. Presence of meconium in the amniotic fluid and aspiration of meconium-stained amniotic fluid (MSAF) are the main causes of MAS. Passage of meconium may be a physiological maturation phenomenon during fetal development, and MSAF is uncommon between 20 weeks’ and 34 weeks’ gestation.
MSAF may have pathological causes: fetal distress and intrapartum hypoxia secondary to placental insufficiency, cord compression, cord around the neck, oligohydramnios, maternal hypertension, pre-eclampsia, maternal diabetes, maternal drug abuse, or smoking.
These factors can lead to gasping and aspiration of meconium by the fetus and the newborn. Certain infections, such as group B streptococcal infections and listeriosis, may be associated with MAS. However, one study found no evidence of maternal or neonatal listeriosis in MAS cases and concluded that MSAF is not a useful indicator for listeriosis in mother or baby.
Maternal chorioamnionitis may also contribute to stress and fetal passage of meconium. During delivery, about 5% of neonates with meconium passage aspirate the meconium, triggering lung injury and respiratory distress.
RFs for meconium aspiration syndrome
STRONG >42w Maternal: HTN, pre-eclampsia, eclampsia, smoking, substance abuse Fetal distress Oligohydramnios Thick meconium Apgar score <7 Chorioamnionitis CS
WEAK
Black/east asian
Male
Sx of meconium aspiration syndrome
RFs Sx of postmaturity: green/yellow-coloured skin and umbilical cord; long, stained nails; dry and peeling skin; post-mature appearance; and loss of subcutaneous tissue Tachypnoea Cyanosis Systolic murmur Chest wall asymmetry (tension pneumo) Dec AE Barrel shaped chest Grunting Chest retractions Rales Rhonchi Tachycardia Hypotension
Ix of meconium aspiration syndrome
CXR - irregular pattern, patchy infiltrations, atelectasis, hyperexpanded lung fields, consolidation; may show pleural effusion, pneumothorax, or pneumomediastinum; cardiomegaly may be seen
FBC - rule out infection
CRP - rule out infection
Blood culture - Normal
Rx of meconium aspiration syndrome
O2 ABx - gentamicin CPAP ventilation Vasopressor if low BP Surfactant may be given through ET
Severe resp distress: Morphine sulfate + intubation
Infants who are refractory to mechanical ventilation with high oxygen inspiration and surfactant treatment invariably have concomitant persistent pulmonary hypertension (PPHN). iNO should be given with a starting concentration of 20 ppm, along with conventional or high-frequency ventilation
ECMO - last line
Prognosis of meconium aspiration syndrome
The course of MAS may vary from mild to moderate disease to a severe form leading to death, unless prompt appropriate intervention is provided. Data from a large population of term babies with the diagnosis of MAS corroborate previous observations in smaller population studies.
In most infants (80%), the course of illness is short, lasting 3 to 4 days, after which they are discharged home.
Nine percent of patients are transferred to level III+ intensive care units. Most of these infants spend 1 to 2 weeks under respiratory support and are discharged home, 5% require oxygen support at discharge (at 28 days of life), and 4.9% developed seizures
Long-term follow-up studies suggest that MAS infants may have several neurological developmental disabilities, particularly if they have experienced in utero hypoxia.
Complications of meconium aspiration syndrome
Pneumothorax Persistent pulmonary HTN Pneumomediastinum Chronic lung disease Cerebral palsy Seizure Reactive airway disease Infections
A term, male newborn weighing 2968 g (6 lb, 9 oz) has been delivered at a tertiary hospital at 39 weeks’ gestation by emergency caesarean section because of fetal tachycardia and meconium-stained amniotic fluid (MSAF). The mother is a 17-year-old primigravida who had group B beta-streptococcal-positive culture at 35 weeks’ gestation. At delivery, the infant is limp and blue with poor respiratory effort. He has an Apgar score of 4 at 1 minute. The infant’s skin and umbilical cord are stained green, and he continues to demonstrate tachypnoea, with a respiratory rate of 70/minute, mild retractions, and grunting.
Meconium aspiration syndrome
A female infant is delivered post-term (at 42 weeks, 6 days) by emergency caesarean section because of fetal distress and thick MSAF. The birth weight is 3000 g (6 lb 10 oz). She presents with moderate retractions and grunting with an Apgar score of 3 at 1 minute.
Meconium aspiration syndrome
Atypical presentations include an initial normal presentation, with deterioration later if meconium-stained amniotic fluid and fetal distress are not recognised at birth. In some term births in which the history of meconium aspiration is not obvious, cyanotic heart disease may be incorrectly diagnosed.
Although MAS is common in term and post-mature babies, it can also be seen in preterm infants, though less commonly. In preterm infants, MAS may be misdiagnosed as respiratory distress syndrome resulting from surfactant deficiency.
Meconium aspiration syndrome
Define Meckel’s diverticulum
The most common congenital malformation of the small bowel, Meckel’s diverticulum (MD) is present in 1% to 3% of the population.
It is a true diverticulum that results from the failure of the vitelline duct to obliterate during the fifth week of fetal development. Patients are often asymptomatic. However, this embryological remnant may cause bleeding, obstruction, inflammation, or perforation.
Most common congenital abnormality of the small bowel.
Majority of patients remain asymptomatic for their entire life.
Majority of symptomatic patients present before the age of 2 years.
GI bleeding is a common presenting symptom in children and adults. The most useful diagnostic tool in cases with bleeding is the technetium-99m pertechnetate scan (Meckel’s scan).
Obstructive symptoms are another common presenting symptom and may be caused by several mechanisms, including intussusception.
Meckel’s diverticulitis occurs in 20% of patients who become symptomatic and is clinically indistinguishable from appendicitis.
Aetiology of Meckel’s diverticulum
Meckel’s diverticulum (MD) is a congenital abnormality of unknown cause. It results from an incomplete obliteration of the omphalomesenteric (vitelline) duct. In the 3-week-old embryo, the yolk sac communicates with the gut through a wide vitelline duct, which receives its blood supply from paired vitelline arteries. During week 8, the duct is normally obliterated when the placenta replaces the yolk sac as the source of fetal nutrition. The left vitelline artery usually involutes and the right one forms the superior mesenteric artery.
Epidemiology of Meckel’s diverticulum
Most common congenital malformation of the small bowel
1% to 3% of the population.
Most asymptomatic
Males 2-3x more likely to have symptomatic complications
RFs for Meckel’s diverticulum
Age <2
Sx of Meckel’s diverticulum
Age <2 Haematochezia Intractable constipation N+V (SBO) Abdominal cramps RLQ abdominal pain
UNCOMMON
Palpable abdominal mass
Hypotension/tachycardia (GI bleed)
Ix for Meckel’s diverticulum
FBC - leukocytosis / low Hb
Technetium 99 - ectopic focus
Plain abdo X-ray - dilation/pneumoperitoneum/intussusception
CT abdo/pelvis - blind-ending fluid-filled and/or gas-filled structure in continuity with distal ileum
US abdomen / contrast enema - IF intussusception is suspected
Rx of Meckel’s diverticulum
ASYMPTOMATIC
- no Rx / could excise
SYMPTOMATIC
Bleeding - excision + ?blood transfusion
Perforation - give ABx: cefotaxime + clindamycin/metronidazole
Prognosis of Meckel’s diverticulum
If symptomatic patients are treated in a timely fashion, the prognosis for Meckel’s diverticulum (MD) is excellent.
The risk of developing a postoperative complication ranges from 2% to 7%.
The most common post-surgical complication is adhesive bowel obstruction.
Complications of Meckel’s diverticulum
PostOP:
- wound infection
- intestinal leakage
- BO
A 20-month-old boy is admitted to hospital with a history of having passed a bloody stool 8 hours before presentation. He has previously been well. On examination, he is pale and distressed but has no abdominal mass or tenderness. A contrast enema proves negative for intussusception.
Meckel’s diverticulum
A 68-year-old man presents to the accident and emergency department with a 24-hour history of colicky central abdominal pain associated with anorexia and intractable constipation (obstipation). The pain is associated with nausea and vomiting. He has had no previous abdominal surgery. On examination, he is dehydrated with no clinical signs of sepsis. His abdomen is distended and diffusely tender with no rebound tenderness or guarding. He has no hernias. Laboratory studies reveal an elevated white blood cell count. The abdominal plain x-rays show grossly dilated small bowel loops with paucity of gas in the colon. However, after 24 hours of treatment for small bowel obstruction, his abdominal pain worsens.
Meckel’s diverticulum
Diverticulitis occurs in 20% to 30% of patients with Meckel’s diverticulum (MD) who become symptomatic and is clinically indistinguishable from appendicitis, with periumbilical pain that radiates to the RLQ.
Diverticular obstruction can lead to distal inflammation, necrosis, and perforation, resulting in an abscess, peritonitis, or, rarely, haemoperitoneum.
Meckel’s diverticulum
Define Neonatal jaundice
Neonatal jaundice is the yellowing discoloration of the skin and sclera of a neonate, which is caused by increased levels of bilirubin in the blood. A neonate refers to an infant in the first 28 days of life
Usually noted clinically when serum bilirubin is >85.5 micromol/L (5 mg/dL). Occurs in 50% to 70% of term neonates. Most cases physiological.
Jaundice in the first 24 hours of life is considered pathological.
Treatment for severe hyperbilirubinaemia includes phototherapy and/or exchange transfusion.
The major complication of unconjugated hyperbilirubinaemia is kernicterus.
Epidemiology of Neonatal jaundice
Most common condition in newborns requiring medical attention
50-70% of term babies
80% of preterm
Usually appears 2-4 days old -> resolves 1-2 weeks later without the need for Rx
Hyperbilirubinaemia <8% births
RFs for Neonatal jaundice
STRONG East asian American indian Maternal diabetes LBW <37weeks Decreased caloric intake/WL Breast feeding
WEAK
Oxytocin in labour
Delayed cord clamping
Aetiology for Neonatal jaundice
Physiological jaundice can be a result of:
Increased bilirubin load secondary to increased red blood cell (RBC) volume, decreased RBC life span, or increased enterohepatic circulation
Decreased uptake by the liver because of decreased ligandins or binding of ligandins to other anions
Decreased conjugation in the liver because of decreased uridine diphosphoglucuronyl transferase (UDPGT) activity. UGT1A1 gene polymorphisms of Gly71Arg and TATA promoter, which decrease UDPGT enzymatic activity, have been noted to be significant risk factors associated with neonatal hyperbilirubinaemia
Decreased excretion into bile.
Pathological jaundice with unconjugated hyperbilirubinaemia can be a result of:
Haemolytic anaemias: these result in increased destruction of RBCs, with resultant increased haem, which is converted to excess unconjugated bilirubin; the immature liver is unable to handle the excess load. They can be the result of blood group incompatibility (rhesus, ABO), RBC enzyme defects (glucose-6-phosphate dehydrogenase deficiency; pyruvate kinase deficiency), RBC membrane defects (e.g., hereditary spherocytosis, infantile pyknocytosis), thalassaemia, drug-induced (by vitamin K, sulphonamides, nitrofurantoin, anti-malarials, penicillin), or sepsis
Extravasation of blood: sequestration of blood in cavities result in increased bilirubin load. Examples include cephalhaematoma; intracranial, pulmonary, or gastrointestinal haemorrhage; large haemangiomas; excessive ecchymoses; or petechiae
Polycythaemia: increased number of RBCs leads to increased production of bilirubin
Increased enterohepatic circulation: delayed gastrointestinal transit increases bilirubin levels. Examples include intestinal atresia/stenosis, pyloric stenosis, Hirschsprung’s disease, meconium ileus/plug syndrome
Defective conjugation: congenital deficiencies of UDPGT enzyme include Crigler-Najjar syndrome; UDPGT enzyme inhibition can be the result of drugs (e.g., novobiocin), or Lucey-Driscoll syndrome
Metabolic conditions (galactosaemia, hypothyroidism, tyrosinosis, hypermethioninaemia, maternal diabetes)
Breastfeeding (including failure to establish lactation)
Decreased binding of bilirubin to albumin: increased availability of the free (unconjugated) bilirubin to cross the blood-brain barrier. This can be caused by drugs (sulphonamides, penicillin, gentamicin), acidosis, asphyxia, hypothermia, increased osmolality, or hypoglycaemia.
Pathological jaundice with conjugated hyperbilirubinaemia (direct bilirubin is >34.2 micromol/L (2.0 mg/dL)) can be a result of:
Hepatocellular disease:
Metabolic or genetic defects. Examples include alpha1-antitrypsin deficiency, cystic fibrosis, Zellweger’s syndrome, Dubin-Johnson syndrome (absence of multidrug resistance-associated protein 2 from the canalicular membrane of hepatocytes), Rotor’s syndrome (organic-anion-transporting polypeptide [OATP]1B1 and OATP1B3 are absent at the sinusoidal membrane of hepatocytes), and galactosaemia
Infections. Examples include rubella, cytomegalovirus, herpes, syphilis, hepatitis A and B, toxoplasmosis, and urinary tract infection with Escherichia coli
Total parenteral nutrition [10]
Neonatal haemochromatosis
Idiopathic neonatal hepatitis
Shock.
Intrahepatic biliary disease due to Alagille syndrome (arteriohepatic dysplasia), or inspissated bile syndrome
Extrahepatic biliary disease due to biliary atresia, choledochal cyst, bile duct stenosis, cholelithiasis.
Sx of Neonatal jaundice
Jaundice (first appearing on face - spreads caudally)
Male
FHx anaemia/splenectomy/jaundice
Maternal exposure to sulphonamides/antimalarials
BILIRUBIN ENCEPHALOPATHY Sx:
Hypertonia
High pitched cry
Retrocollis (cervical dystonia (CD) that produces patterned, repetitive muscle contractions that result in neck extension)
Opisthotonus (spasm of the muscles causing backward arching of the head, neck, and spine)
Ix of Neonatal jaundice
Transcutaneous bilirubinometer - screening test: elevated
Serum bilirubin - Increased
Coombs - to diagnose ABO/RH iso-immunisation
FBC - potential sepsis
Reticulocytes - pot sepsis
Peripheral blood smear - haemolysis causes
Rx of Neonatal jaundice
Physiological = no Rx
Jaundice is physiological if it occurs in the second postnatal day and resolves in 7-10 days and transcutaneous measurement is normal.
Acute bilirubin encephalopathy
- Immediate exchange transfusion
- Phototherapy
- Hydration
- IVIG
Treat underlying cause
Prognosis of Neonatal jaundice
Most neonates with neonatal unconjugated hyperbilirubinaemia do well after phototherapy and/or exchange transfusion. Kernicterus should be preventable if recommendations for hyperbilirubinaemia management are carried out in a timely manner.
Complications of Neonatal jaundice
Neurological damage
Acute bilirubin encephalopathy
Exchange transfusion complications (electrolyte disturbances, bleeding, infection, cardiac arrhythmias, thrombosis with embolisation, necrotising enterocolitis, and graft-versus-host disease)
Phototherapy complications (insensible water loss, loose stools, skin rash, and potential retinal damage)
Kernicterus (chronic bilirubin encephalopathy) - diagnosed pathologically by gross yellow staining and necroses of neurons in the basal ganglia, hippocampal area, and cerebellum. If the neonate survives, the clinical features include chorio-athetoid cerebral palsy, paralysis of upward gaze, sensorineural hearing loss, dental dysplasia, and intellectual deficits (less often in the mental retardation range).
A baby boy of approximately 36 weeks gestational age is born to a primigravida mother. Pregnancy and delivery are uncomplicated, with Apgar scores of 9 at 1 and 5 minutes. Mother’s and baby’s blood groups are both O+. Mother chooses to exclusively breastfeed the baby. At 24 hours of life, the baby is noted to be jaundiced and the total serum bilirubin is noted to be 119.7 micromol/L (7 mg/dL). He is discharged home later the same day with an appointment for follow-up with the paediatrician at 1 week of age. However, 48 hours later, the baby is brought to the emergency department. History from the mother reveals that the baby has progressively become more jaundiced, is not breastfeeding well and is lethargic. Examination also reveals evidence of moderate volume depletion and significant jaundice (including the soles). The neurological examination is normal and total serum bilirubin is 342.1 micromol/L (20 mg/dL).
Neonatal jaundice
A term baby is born to a mother who had a previous baby with a history of jaundice in the newborn period, not requiring hospitalisation. Pregnancy and delivery are uncomplicated, with Apgar scores of 8 and 9 at 1 and 5 minutes, respectively. Mother’s and baby’s blood groups are O+ and B+, respectively. At 12 hours of life, the baby is noted to be jaundiced and the total serum bilirubin is 85.5 micromol/L (5 mg/dL). Tests reveal direct Coombs’ test to be positive and presence of microspherocytes on the peripheral smear.
Neonatal jaundice
The neonate may present with clinical signs of bilirubin encephalopathy. These include irritability with a high-pitched cry, possibly fever and increased muscle tone (usually involving the extensor group of muscles), and characteristically intermittent backwards arching of the neck (retrocollis) and trunk (opisthotonus). Decreased tone and abnormal Moro reflex are possible manifestations.
Neonatal jaundice
Define constipation
Constipation in childhood is the infrequent passage of stools leading to 1 or more of the following: painful defecation; overflow faecal incontinence; rectal faecal impaction; or active defecation avoidance behaviour.
Childhood constipation is typically characterised by infrequent bowel evacuations, large stools, and difficult or painful defecation.
Symptoms may result from low fibre, poor nutrient, and/or insufficient water intake, which leads to excessive hardening of the stool. This often starts as an acute problem but can progress to faecal impaction and chronic constipation.
Particularly in young children, any cause of painful defecation may provoke active withholding. Withholding may worsen the constipation and lead to a vicious cycle.
Duration of constipation and amount of stool burden may depend on capacity of the child’s rectum, the degree of megarectum, and other factors including rectal sensory function.
After faecal disimpaction, maintenance stool softeners are essential and are often required for many months or years to reduce relapse risk.
Aetiology of constipation
In most children, no aetiological factors can be found. Childhood constipation results from a combination of painful defecation (from any cause) and a rectum of sufficient capacity to allow stools to be withheld.
The likely sequence of events probably follows initial hard stools, often due to low fibre, poor nutrient, and/or insufficient water intake or, less commonly, due to delayed colonic motility. The excessive colonic time for stool transport leads to high levels of colonic reabsorption of water and further hardening of the stool. This often starts as an acute problem, which can then progress to:
Pain on passing the stool, which leads to involuntary and later learned sphincter contraction to avoid discomfort
Increasing delays between episodes of defecation, leading to enlargement of residual stool volume in the rectum
Further increase in the size of the loaded rectum and more effective withholding tactics
Increase in pain and fear as delayed stool is incrementally larger and thus even more painful
Eventually, overflow faecal incontinence occurs when the stool stimulates rectal contractions and the internal sphincter temporarily relaxes, with passage of softer stool going around retained hardened fragments.
RFs of constipation
STRONG Low fibre Poor nutrient Genetic predisposition Infection Stress Obesity Low birth weight Psychiatric Hx
WEAK
Immune dysregulation
Low fluid intake
Trauma
Epidemiology of constipation
3% of consultations in an average paediatric clinic
Prevalence - 0.7% to 26.9%
Sx constipation
COMMON Difficult/painful defecation Long interval between stools Faecal incontinence Small volume, soft incontinent tool Faecal incontinence Palpable faecal mass per abdomen Otherwise healthy child Abdominal pain
UNCOMMON Anal tissue Associated bladder problems Abnormal anal appearance Abnormal neuro examination
Ix constipation
1st investigations:
NONE - history and examination findings are often sufficient to diagnose the condition.
CONSIDER
X-ray - if stool non-palpable
USS - may see rectal mass
Rectal mucosal biopsy - usually only done if abdominal distension and no passing of meconium: excludes Hirschsprung’s
Rx constipation
NO IMPACTION <1yo
Dietary modification
Osmotic laxative: Lactulose 1ml/kg OD/BD
NO IMPACTION >1yo
Could add Senna <1 tablet daily
With impaction <3yo
Usually need a laxative
- lactulose / glycerol rectal / polyethene glycol
With impaction >4yo
Could do all of above
AND phosphate enema
IE overall:
Osmotic laxative + stimulant laxative + faecal softener
Faecal softener: liquid paraffin / docusate sodium
Prognosis of constipation
1/3rd continues through puberty
Long-term prognosis depends on the degree of megarectum and the presence of other illnesses, particularly psychological disorders such as autism, or major psychosocial problems.
Complications of constipation
Anal fissure Incontinence Abdominal pain Dehydration Megarectum Aspiration pneumonia
A 5-month-old baby boy presents with difficulty and delay in passing hard stools. His mother reports that he strains for several hours and may even miss a day, before passing stool with screaming and occasional spots of fresh blood on the stool or nappy. He has recently been weaned from breastfeeding to cows’ milk formula, which he had been reluctant to drink initially. The child is thriving and now feeding normally. There was no neonatal delay in defecation and no history of excessive vomiting or abdominal distension.
Constipation
A 14-year-old girl, concerned about body image, altered her diet and decreased her oral intake hoping to lose weight. Additionally, she avoided toilets at school due to their lack of cleanliness. She presented to her paediatrician with the complaint of abdominal pain, distension, bloating, and difficult, painful defecation.
Constipation
In 90% to 95% of children with constipation, the problem is functional. [1] However, organic causes should be considered in making this diagnosis. Chronic constipation may present after any cause of painful defecation such as an anal fissure, perianal streptococcal infection, food intolerance (particularly cows’ milk allergy), lichen sclerosis et atrophicus, and penetrative child sexual abuse. Refusal to defecate may also be a presentation of complex psychological problems, such as communication disorders that fall within the autistic spectrum or children with attention deficit disorder. [2] [3] Faecal incontinence in later childhood may occur after poor sensory or behavioural response to faecal loading of the rectum secondary to longstanding megarectum. Rarer causes of constipation include those presenting in the first weeks of life, such as Hirschsprung’s disease or anorectal anomalies. In teenage years, abnormal diets as well as eating disorders may lead to constipation.
Constipation
Define faecal incontinence in children
The involuntary passage of stool from the rectum through the anus resulting in episodes of stooling at inappropriate times, occurring at an age after which toilet training is typically complete (in most, 4 years of age or older).
Involuntary defecation at inappropriate times, occurring after the typical age of completed toilet training (generally 4 years).
In most cases, caused by overflow soiling associated with chronic constipation and severe faecal impaction in an otherwise healthy child without underlying anatomical abnormalities.
May have a significant negative impact on the quality of life for the child and the family.
A thorough history and physical examination are the crux of diagnosis. Laboratory and radiological studies may be used on a case-by-case basis to determine the underlying aetiology.
The goal of therapy is for patient to stool 1 to 3 times per day in the toilet with no episodes of soiling.
Treatment involves complete colon disimpaction, long-term treatment with laxatives and stool softeners, and bowel re-education. Any underlying causes should also be addressed.
Aetiology of faecal incontinence in children
Functional faecal incontinence associated with chronic constipation and severe faecal impaction
The child has no underlying anatomical abnormality, but the severe faecal impaction leads to dilation of the rectum.
Chronic dilation leads to decreased rectal sensation, so the child has less urge to defecate.
Softer stool forms above the hard impaction, and the softer stool leaks around the hard stool, known as ‘overflow’.
Functional non-retentive faecal incontinence
The exact aetiology is unknown, but possible contributing factors include psychological, behavioural, or emotional disturbances, genetics, and decreased motor function and sensation in the gastrointestinal tract.
Aetiology is likely to be multi-factorial. [10]
Organic faecal incontinence
Anatomical abnormalities of the gastrointestinal tract or neurological denervation of the gastrointestinal tract leads to inability to control bowel movements. [11]
The child has either an underlying anatomical abnormality (e.g., anorectal malformations, spinal and neurological abnormalities, or post-surgical abnormality) that causes the lack of bowel control, or a medical condition (e.g., hypothyroidism, coeliac disease, or cystic fibrosis) that may result in faecal impaction with overflow incontinence.
Medication overuse
Rarely, faecal incontinence may be due to overuse of medication (i.e., laxatives or orlistat) used mainly by adolescents and adults.
Epidemiology of faecal incontinence in children
The overall prevalence of faecal incontinence was found to be 3%, with more boys (3.7%) than girls (2.4%) being affected.
Anorectal malformations occur in 1 in 5000 live births, and 25% of these children experience faecal incontinence that requires long-term treatment.
RFs of faecal incontinence in children
STRONG Chronic constipation Male sex Age 5-6yo Diet lacking in fibre Inadequate fluid intake Delayed or inadequate toilet training Anorectal Malformations Hirschprung's (50% post surgery) Spinal abnormalities ie bifida
WEAK
Psychological or behavioural problems due to stressful family events
Sx of faecal incontinence in children
COMMON Hx surgery Faecal soiling or diarrhoea Constipation Perianal skin irritation Behaviour problems Abdo cramping Posturing Enuresis Anorectal malformation Anormal abdo examination
UNCOMMON
Abnormal rectal examination
Spinal deformities
Weakness/absent reflexes
Ix faecal incontinence in children
1st:
Abdo Xray - severe stool retention in faecal incontinence associated with chronic constipation; large, distended bowel loops (bottleneck appearance) may be noted in Hirschsprung’s disease
OTHER
Consider barium enema if suspecting hirschsprungs (no poo in >72hrs)
If hypo thyroid Sx - TSH/T4
Sweat chloride if suspect CF
Manometry studies - abnormal sphincter
Rx of faecal incontinence in children
With faecal impaction:
- Oral laxative - liq paraffin, magnesium citrate or polyethylene glycol
- Enema - sodium phosphate or liquid paraffin
- BOTH enema and laxative
With organic cause unresponsive to Rx - surgery
Bowel retraining - get child to strain for 5-10 mins per day 3x
Loperamide
Prognosis of faecal incontinence in children
Functional non-retentive
In one study, only 29% of the patients with functional non-retentive faecal incontinence were successfully treated after 2 years of intensive therapy. After 18 years of age, 15% continued to have faecal incontinence despite treatment.
Anorectal malformations
Patients with vestibular fistula, perineal fistula, rectal atresia, recto-urethral bulbar fistula, or imperforate anus without fistula will probably have voluntary bowel movements by 3 years of age.
Complications of faecal incontinence in children
Poor QoL
Irritant dermatitis of peri-anal area / buttocks / perineum
Worsening soiling
Enuresis
Concerned parents of an embarrassed 7-year-old boy present with a complaint of frequent faecal soiling of his underwear for the past 6 months. The boy also experiences frequent bedwetting and has a large, painful bowel movement in the toilet only twice a week. The boy is otherwise well and active, and admits to not wanting to use the toilet at school or to stop playing video games to use the toilet at home.
Faecal incontinence in children
Functional
Define enuresis
Enuresis is defined as normal micturition that occurs at an inappropriate or socially unacceptable time or place. As recommended by International Children’s Continence Society, in this monograph ‘enuresis’ is reserved for micturition during sleep, or bedwetting. Daytime wetting is called ‘incontinence’.
Primarily nocturnal symptoms in children older than 5 years of age.
Differentials include diabetes, medications, emotional problems, UTI, spina bifida, seizure disorder, and neurogenic bladder.
Treatment is commonly involves behavioural changes, alarm therapy, or desmopressin.
Emotional support and encouragement is vital to management.
Epidemiology of enuresis
Prevalence at 5yo = 5-10%
A report from the UK states that between 2% and 3% of 12- to 14-year-olds and 1% to 2% of people aged 15 years and older wet the bed twice a week on average.
In otherwise healthy adults aged 18 to 64 years, studies show a 0.5% prevalence of enuresis.
Aetiology of enuresis
Disorders of sleep arousal
Decreased functional bladder capacity
Nocturnal polyuria
- Poorly concentrated urine as assessed by urinalysis can also be indicative of nocturnal polyuria.
Nocturnal overactive bladder
- Children who do not respond to interventions with alarm therapy or desmopressin may suffer from overactive bladder. Often daytime symptoms are masked by moderating fluid intake to minimise daytime urination.
RFs of enuresis
STRONG
Genetic predisposition
Upper airway ob/sleep disordered breathing
WEAK
Constipation
ADHD
Psychological disorders
Sx of enuresis
RFs Increased fluid intake at night Urinary frequency - is variable and can be too frequent (>8 voids per day) or too infrequent (<3 voids per day). Constipation Caffeine
UNCOMMON
Urinary urgency
Ix of enuresis
Urinalysis - N
To exclude infection, signs of renal disease, poor urine concentrating ability, or glycosuria indicative of diabetes mellitus. In monosymptomatic nocturnal enuresis, U/A should be normal.
Urinary tract US - N
Rx of enuresis
<7yo - reassurance
- Rx only begins after this
Bladder training
Alarm therapy
Desmopressin - 0.2 to 0.4 mg orally once daily at bedtime, dose may be increased up to 0.6 mg/day
SOME PTs
Detrusor relaxing drugs: oxybutynin 5mg orally
Tolterodine 1mg orally
Complications of enuresis
Psychological disorders
Prognosis of enuresis
While short-term recurrence is the rule, especially with pharmacological interventions, long-term resolution is almost inevitable as only 0.5% of adults have nocturnal enuresis
A 7-year-old boy is brought to the clinic by his parents, who complain that he continues to wet the bed at night, 2 to 3 times a week. Further investigation determines that both the mother and the father had nocturnal enuresis but report that they just grew out of it. The child and family are clearly distressed about this and are willing to do whatever it takes to improve the situation.
Enuresis
The most common form of enuresis is monosymptomatic nocturnal enuresis, which is usually referred to as bedwetting. Other types of enuresis are associated with baseline voiding dysfunction. All other conditions in which voiding dysfunction manifests itself with the loss of urine are more appropriately defined as incontinence. Monosymptomatic enuresis occurring only at night is dealt with separately from incontinence, both diagnostically and therapeutically.
Parents of young children who have enuresis rarely bring them to a physician, as such behaviour is socially appropriate. The age at which this becomes unacceptable varies from culture to culture. The DSM-5 and the medical community generally recognise 5 years old as the cut-off. [2] [3] Additionally, the consensus of the International Children’s Continence Society is that the number of acceptable wet nights is between 1 and 3 per month; more than this and the child and/or parents are typically concerned enough to bring it to the attention of their physician. [4] Most of these children will present having never been consistently dry. Secondary enuresis can be associated with a stressful life event or a new medical condition, and warrants further investigation.
Enuresis
Define Rh incompatibility
Rhesus (Rh) incompatibility is caused by destruction of fetal red blood cells (RBCs) from transplacental passage of maternally derived immunoglobulin G (IgG) antibodies. IgG antibodies are produced by the maternal immune system, usually against the RhD antigen. These antibodies can freely cross the placenta, binding to and destroying RBCs. More than 50 known RBC antibodies potentially cause Rh incompatibility. The consequence is progressive fetal anaemia, which, untreated, may ultimately lead to hydrops fetalis (collection of fluid in serous compartments) and death.
Maternal antibodies produced in response to paternally derived D antigens on fetal red blood cells are the leading cause of severe haemolytic disease of the fetus and newborn (erythroblastosis fetalis).
Effective immunoprophylaxis of rhesus D-negative at-risk mothers is key to primary prevention.
Intrauterine fetal transfusion is a life-saving treatment for severely affected fetuses.
Survival rates are more than 90%.
Epidemiology of Rh incompatibility
About 15% of the white population has a RhD-negative blood type.
Seven percent of black people have this blood type.
In the UK, about 16% of the white population is RhD negative. In 2005, it was estimated that about 65,000 RhD-positive babies were born in the UK to women who were RhD-negative (accounting for 10% of all births).
Aetiology of Rh incompatibility
Rh incompatibility is caused by destruction of fetal red blood cells (RBCs) from transplacental passage of maternally derived immunoglobulin G antibodies. Passage of fetal cells into the maternal circulation and fetomaternal haemorrhage (FMH) is a frequent occurrence, detectable in 65% of pregnancies either antenatally or in the early postnatal period.
Placental trauma of varying degrees may lead to sensitising FMH. FMH increases throughout pregnancy (3% first trimester, 43% second trimester, and 64% third trimester).
FMH has been found in 1% to 6% of external cephalic versions. Small amounts of FMH (>0.1 mL) are potentially immunising and occur in 2% of patients undergoing amniocentesis. The incidence of FMH at the time of chorionic villus sampling is about 14%. The incidence is higher with transplacental cordocentesis.
RFs of Rh incompatibility
STRONG RhD neg mother FMH Invasive foetal procedures Placental trauma Abortion Multiparity Omission of Rh immunoprophylaxis
WEAK
ECV
Molar pregnancy
Ectopic pregnancy
Sx of Rh incompatibility
RFs
Ix of Rh incompatibility
1st
Maternal blood - Rh-neg
Maternal serum Rh Ab screen - +ve screen
Consider
Paternal blood typing
Fetal US - signs of fetal anaemia - may show subcutaneous oedema, ascites, pleural effusion, or pericardial effusion
Doppler of MCA - ≥1.5 MoM - fetal anaemia
Kleihauer-Betke test/flow cytometry- measures fetal blood in maternal circulation
Rx of Rh incompatibility
Anti-D immunoglobulin
- After FMH
- Amniocentesis / chorionic villus
- > 40 weeks gestation
Intravascular intrauterine blood transfusions possible
Neonates with erythroblastosis are immediately evaluated by a paediatrician to determine the need for exchange transfusion, phototherapy, or intravenous immunoglobulin.
Prognosis of Rh incompatibility
RhD alloimmunisation is a disease that will usually progress during the pregnancy and in future pregnancies. Therefore, follow-up of these patients is required through antibody titres and fetal middle cerebral artery Doppler ultrasound during subsequent pregnancies.
Complications of Rh incompatibility
Hyperbilirubinaemia and kernicterus - Encephalopathy, athetoid cerebral palsy, and/or sensorineural deafness may result from deposition of bilirubin into the basal gangli
Transfusion related fetal Brady
Transfusion related neurodevelopmental disorders
Fetal / neonatal hydrops - Hydrops fetalis is defined as abnormal accumulation of fluid in 2 or more fetal compartments.
Neonatal anaemia
A 32-year-old woman presents at 25 weeks’ gestation in her third pregnancy with a positive antibody screen. She is known to be Rh-negative with a Rh-positive sexual partner. Two previous children were born overseas: the first child was carried to term and is healthy. The second child, also born at term, underwent phototherapy in the immediate neonatal period due to jaundice. The patient did not have anti-D prophylaxis given antenatally or postnatally in the previous pregnancies. Physical examination is normal.
Rh incompatibility
A 38-year-old primigravida woman presents for routine antenatal care. Her blood type is known to be Rh-negative with a negative indirect Coombs test, and her sexual partner is Rh-positive. She has been counselled regarding the need for Rh immunoprophylaxis at 28 weeks of pregnancy and postnatally if her newborn is found to be Rh-positive.
Rh incompatibility
Manifestations of severe erythroblastosis fetalis include ultrasound evidence of significant effusions in serous cavities, organomegaly, polyhydramnios, and extensive skin oedema (anasarca). Anti-RhD antibody titres in severe disease are usually high (>1:32 dilutions). Anti-Kell antibodies may be associated with profound fetal anaemia and hydrops in the presence of low antibody titres due to suppression of erythropoiesis. Evidence suggesting severe fetal anaemia includes high peak systolic velocities on Doppler ultrasound of the middle cerebral artery, low biophysical profile scores, and a sinusoidal fetal heart rate pattern. Although these manifestations of severe fetal disease are usually not detected in a first affected pregnancy, significant fetomaternal haemorrhage from any cause may lead to a secondary immune response and hydrops fetalis, even in a primiparous patient.
Rh incompatibility
Define intestinal malrotation
Intestinal malrotation is a term used to describe an entire spectrum of rotational and fixation disturbances that can occur during embryonic development.
Abnormal caecal attachments to the right upper peritoneal cavity (i.e., Ladd’s bands) can cross the second portion of the duodenum creating an extraluminal low-grade obstruction of the duodenum, which may present with signs of partial obstruction. However, the most significant pathological concerns in malrotation are a lack of intestinal fixation to the retro-peritoneum and a narrow midgut mesenteric base that predisposes to a twisting of the small bowel in the form of midgut volvulus. This condition creates an abrupt obstruction of the duodenum, resulting in bilious vomiting.
If the twisting at the base also obstructs flow in the superior mesenteric artery, the entire small intestine and proximal colon may become acutely ischaemic and subsequently necrotic within a few hours.
Patients with malrotation are at risk for volvulus and should be identified whenever possible to allow for proper therapy.
An entire spectrum of rotational and fixation disturbances that can occur during embryonic development.
The anatomical variant that poses the highest risk of volvulus is a narrow midgut mesenteric base accompanied by lack of retro-peritoneal midgut fixation. This variant cannot be reliably determined from any radiological studies.
Malrotation predisposes patients to a risk of midgut volvulus.
Emergency surgical consultation is appropriate before obtaining any diagnostic studies if midgut volvulus is suspected. Upper GI contrast studies should be ordered in patients who are stable.
Treatment is surgical correction, the procedure of choice being the Ladd procedure.
Epidemiology of intestinal malrotation
0.2% of upper GI contrast studies
Autopsy suggests 0.5%
Aetiology of intestinal malrotation
Congenital
RFs of intestinal malrotation
Embryological abnormality
Sx of intestinal malrotation
COMMON Billous green vomiting (>90% of midgut volvulus) Abdominal pain (inconsolable state) Infant <1y Normal abdo exam - a distended abdomen is more commonly a bowel obstruction further downstream. Vomiting and abdominal pain with a flat abdomen should alert physicians to the presence of volvulus Distention Tenderness Tachycardia + HTN Tachycardia + hypotension
UNCOMMON
WL
Dark blood in nappy
Rebound tenderness and guarding
Ix for intestinal malrotation
1st Ix
Upper GI contrast - right-sided duodenum (malrotation); duodenum courses inferior or medial to normal (malrotation); bird-beak cut-off of duodenum (volvulus); corkscrew of duodenum (volvulus); a web in the duodenum (duodenal atresia)
CT Abdo with oral+IV contrast - no oral contrast beyond duodenum (volvulus); no contrast in the distal superior mesenteric artery (volvulus with ischaemia); twirling of the superior mesenteric artery and vein (volvulus); transposition of superior mesenteric artery and vein (malrotation); a transition point in bowel calibre, right-sided duodenum; duodenum courses anterior or to right of superior mesenteric artery
Plain abdo x-ray - distended stomach and proximal duodenum with paucity of bowel gas seen distally (volvulus or duodenal web); normal x-ray (malrotation without volvulus or volvulus without complete obstruction)
FBC - may have elevated WCC/polycythaemia
ABG - metabolic acidosis with respiratory alkalosis resulting in profound decrease in partial pressure of carbon dioxide
Rx of intestinal malrotation
OBSTRUCTION WITH ISCHAEMIA
Emergency surgery - open laparotomy + Ladd procedure
+ Cefoxitin IV before surgery
Obstruction without ischaemia
Urgent surgery - ladd procedure open laparotomy
IF intermittent/partial
Can do open or laparoscopic
Prognosis of intestinal malrotation
Mortality with midgut volvulus is approximately 10% and usually depends on degree of intestinal necrosis.
Survivors of volvulus are at risk for recurrent volvulus of around 10%.
Complications of intestinal malrotation
Short-gut syndrome post op
Volvulus following Ladd procedure
Ladd procedure related adhesive small bowel
A 7-day-old female infant is brought to the emergency department with acute-onset bilious vomiting of 6 hours’ duration. The patient has previously tolerated breastfeeding without emesis and has stooled normally. Physical exam is benign without significant tenderness or distention.
Intestinal malrotation
A 1-month-old male infant is brought to the emergency department with acute onset of inconsolable discomfort and bilious vomiting of 6 hours’ duration. Physical exam reveals a tight, distended abdomen with some dark blood in the nappy. He is tachycardic and tachypnoeic.
Intestinal malrotation
Define child abuse
Child abuse (including neglect) is any form of maltreatment of a child, either by inflicting harm or by failing to act to prevent harm. Children may be abused in a family, institutional, or community setting, by those close to them, such as a parent or caregiver or, more rarely, by a stranger. They may be abused by adults or by other children. There are 4 categories of child abuse: physical abuse, emotional abuse (also called psychological maltreatment), sexual abuse, and neglect. This monograph primarily addresses physical child abuse. For information on sexual child abuse, please see our detailed content on sexual abuse.
Epidemiology of child abuse
Child abuse is a world-wide phenomenon and can affect children of all ages; however, the highest incidence occurs to infants and toddlers
In industrialised countries it is estimated that 4% to 16% of children are physically abused, around 10% are neglected or emotionally abused, 5% of boys and 5% to 10% of girls are exposed to penetrative sexual abuse, and 30% are exposed to any form of sexual abuse. Around 80% of child abuse is perpetrated by carers or parents.
Aetiology/RFs for child abuse
STRONG
Domestic violence
Substance abuse/ mental health disorder in parents
Excessive crying/frequent tantrums in infancy
Immature parents/coping skills
Parent / carer abused as child
WEAK
Poor socio-economic status
Demanding parenting role
Sx of child abuse
COMMON
RFs
Inconsistent/changing Hx
Inconsistent injuries in isolation/combination IE - unexplained bruising in a pre-mobile child and other suspicious injuries that do not fit with the developmental age of the child (e.g., a child not yet independently mobile may be unlikely to fall against a particular object).
Bruising
Subdural haemorrhages in infant/young toddler
Long bone fractures in premobile child - high specific for abuse
Rib fractures in absence of major trauma
Immersion scalds
Family known to social services
Poor bonding
Faltering growth
Dental neglect
Petechaie with bruising
Extensive multilayered retinal haemorrhages extending to peripheries
UNCOMMON Small bowel perforation in child <3 Torn Frenum Apnoeas Cigarette burns Frequent accidental poisonings Contact burns Dental injuries Caustic burns
Ix for child abuse
FBC - check platelets/anaemia Clotting profile - exclusion Dilated fundoscopy Skeletal surgery CT brain - haemorrhages LFTs - raised with direct injury Serum calcium - N Serum phosphate - N Alk-phos - raised with fracture PTH - N Vit D - N VWF assay
Rx of child abuse
Management of injuries
Reporting to authorities
- Doctors may be asked to provide written reports for use in multidisciplinary meetings, police investigations, and civil or criminal courts, and may be required to appear as witnesses (of fact or as experts) in court.
Social service intervention
Mental health services - CBT/play therapy
Prognosis of child abuse
The mortality from abusive head trauma (AHT) is high. Around 20% to 30% of AHT child victims die of their injuries, and around two-thirds survive only to have significant long-term disability, including neurological, behavioural, and cognitive sequelae.
Children who have been abused manifest a variety of negative outcomes as adults, including high-risk behaviours such as binge drinking, drug use, and criminal behaviour.
Child abuse of any form (emotional, physical, or sexual abuse, neglect) is also associated with increased mental health problems in adulthood. This may include depression, post-traumatic stress disorder, substance abuse, or anti-social personality disorder.
Complications of child abuse
Kyphosis Psyhological scarring Difficulty with relationships PTSD Developmental delay Brain damage Failure to thrive Death Blindness Disfigurement
A 6-year-old boy is noticed by his teacher to have a black eye. When asked how he sustained the injury, the child states that his stepfather punched him. He is referred to social services and brought to the paediatrician for further assessment. On examination he has a bruise to the outer aspect of his left orbit. On careful inspection, there are 2 linear bruises to the left cheek extending towards the ear, a bruise to the anterior and posterior aspect of the right pinna, and 4 small bruises to the outer aspect of the upper arm. On close questioning, the boy states that his stepfather grabbed him by the arm and slapped him on the face after he had disturbed him when he was watching television. The boy also states that there are lots of arguments in the home, that his stepfather is frightening and has hit his mother, and that if he is naughty, his stepfather will grab his ear and twist it until it hurts.
Child abuse
A 5-month-old baby is brought to the accident and emergency department by ambulance. His mother states that he has not been feeding and has been irritable for the past 24 hours. She states that he vomited any food he was given and subsequently went “blue” and appeared to stop breathing, prompting her to call an ambulance. In the A&E department, the baby appears lethargic but responsive. Examination is unremarkable with the exception of a small bruise to the right cheek and a torn frenum (or frenulum). The mother is anxious about the baby. She is 19 years old and had a previous baby who died of SIDS. She had this baby with her second partner who has recently left her. The baby is admitted for observation and investigation. A modified Glasgow coma scale of 8 is recorded and the baby has a generalised seizure while on the ward. FBC, platelets, a clotting screen, a septic screen, and the biochemical profile are normal. Bi-lateral extensive retinal haemorrhages are noted on indirect ophthalmology. The skeletal survey shows healing posterior rib fractures on the left side to ribs 7 to 10. A CT brain scan shows multiple small subdural haemorrhages over the convexity and in the interhemispheric fissure, with diffuse hypoxic ischaemic injury. A diagnosis of non-accidental head injury seems most likely and a referral is made to police and social services.
Child abuse
Define osteomyelitis
Osteomyelitis is an inflammatory condition of bone caused by an infecting organism, most commonly Staphylococcus aureus . It usually involves a single bone but may rarely affect multiple sites.
Broadly, bone infection is either haematogenous (originating from bacteraemia) or contiguous focus (originating from a focus of infection adjacent to the area of osteomyelitis). Despite these different causes all forms of acute osteomyelitis may evolve and become chronic, sharing a final common pathophysiology, with a compromised soft-tissue envelope surrounding dead, infected, and reactive new bone.
Suspect osteomyelitis in those with a history of open fracture, recent orthopaedic surgery, or a discharging sinus; in the immunocompromised patient; or in the unwell child.
Plain radiographs provide a good initial imaging modality for screening acute and chronic osteomyelitis.
Magnetic resonance imaging and computed tomography (CT) may be used to determine diagnosis and treatment decisions. In some cases positron emission tomography/CT or single-photon emission computed tomography/CT may have a role.
Diagnosis must be confirmed using deep microbiological samples via radiological guided biopsy or open surgery.
In chronic osteomyelitis, surgery to remove the dead bone is the primary treatment modality. Antibiotics alone cannot achieve a cure.
Giving empirical antibiotics without microbiological sampling should be avoided, with the exception of patients who are septic and unwell. Even then, blood cultures should be taken before antibiotics are given.
Only cultures from deep sites are reliable. There is no value in surface or sinus swabs.
Epidemiology of osteomyelitis
A study in the US showed that the overall age- and sex-adjusted annual incidence of osteomyelitis was 21.8 cases per 100,000 person-years between 1969 and 2009. The annual incidence was higher for men than for women and increased with age.
Aetiology of osteomyelitis
Osteomyelitis may be caused from haematogenous spread, direct inoculation of micro-organisms into bone, or from a contiguous focus of infection. A trivial skin infection may be the source of bacteraemia, or it may have emerged as the result of a more serious infection such as acute or subacute bacterial endocarditis. Intravenous drug misuse has been linked to haematogenous osteomyelitis involving the long bones or vertebrae.
Common micro-organisms in acute haematogenous osteomyelitis
Infants:
S aureus
Group B streptococci
Aerobic gram-negative bacilli
Children up to 4 years:
S aureus
Streptococcus pyogenes
Haemophilus influenzae (in those not immunised)
Kingella kingae (increased incidence in children under 4 years)
Older children and adults:
S aureus
Older adults:
Gram-negative bacilli
Patients with intravascular devices:
S aureus
Candida species
Patients who misuse intravenous drugs:
S aureus
Pseudomonas aeruginosa
Patients with sickle cell disease; patients from developing countries:
S aureus
Salmonella species
RFs for osteomyelitis
STRONG Penetrating injury Surgical contamination IV drug use DM Periodontitis
Sx of osteomyelitis
Non-specific pain at site of infection Malaise and fatigue Local inflammation, erythema, swelling Low grade fever Wound drainage Scars/previous flaps Reduced range of movement
UNCOMMON UTI symptoms Torticollis Limb deformity Tenderness to percussion
Ix for osteomyelitis
WBC - High
ESR - usually raised
CRP - usually raised
Plain X-ray - lytic lesion/osteopenia
Consider USS MRI bone CT scan Radionucleotide scans Histology - TB/actinomycosis require histology
Rx of osteomyelitis
Acute:
High dose ABx - take blood cultures
Chronic:
May need debridement
Prognosis of osteomyelitis
Management of osteomyelitis in dedicated units, with close multidisciplinary working, is associated with high success rates and high patient-satisfaction scores. The prognosis from early intervention is usually better, particularly in the context of implant-related infection. Delay in treating these patients is likely to worsen long-term outcomes. The functional outcome for osteomyelitis of the lower extremity depends on the status of the foot and knee.
Complications of osteomyelitis
Drug reactions Flap failure Amputation Growth disturbance Joint stiffness Infection recurrence Fracture Neurological impairment secondary to bony collapse IV catheter related infections
A 40-year-old man who suffered an open tibial fracture in a motor vehicle accident 6 months ago presents with swelling and pain in his lower leg.
Osteomyelitis
A 5-year-old boy fell off his bicycle 2 weeks ago and has stopped walking and complains of non-specific pain in his leg. His mother reports that he apparently has had flu, with fever and chills.
Osteomyelitis
Symptoms of osteomyelitis vary with the duration of the disease and may have a gradual onset over several days. A patient with acute osteomyelitis typically presents as acutely unwell, with bone pain in the region affected along with tenderness, warmth, and swelling. Pain may occur with or without movement. This classic presentation does not occur in all cases. Some sites, such as the vertebrae or pelvis, may present a diagnostic challenge with a systemically unwell patient displaying signs of sepsis but without clear localising signs.
Chronic osteomyelitis generally has a longer duration of symptoms. The pain may be less severe, with minimal fever and fewer constitutional symptoms. There is often a history of a discharging sinus or signs of old healed sinuses, soft-tissue abscesses, or scars from previous surgery or injury. Patients may have lived with discharging sinuses for years, having previously been told that there is nothing that can be done for their problem.
Chronic osteomyelitis may produce long-term ill health with weight loss, malaise, fatigue, or depressed mood. Acute systemic upset is less common but pyrexia, sweating attacks, and anorexia are associated with flare-ups of the disease.
Osteomyelitis
Define phimosis/paraphimosis
Paraphimosis is a condition caused when the foreskin of the uncircumcised penis is retracted and left behind the glans penis, leading to vascular engorgement and oedema of the distal glans. Medical emergency.
Caused when the foreskin of an uncircumcised penis is retracted and left behind the glans, leading to vascular engorgement and oedema of the distal glans.
In its acute form, it is a medical emergency requiring rapid evaluation and possible surgical decompression.
Can occur at any age but is most common in children and older people.
Circumcision is preventive and is curative if the process develops.
Most cases are iatrogenic in aetiology
- Phimosis = unretractable foreskin
Epidemiology of phimosis/paraphimosis
The incidence of paraphimosis is unknown. Only isolated case reports exist in the literature. Due to the fact that most cases are iatrogenically or inadvertently induced, precise reporting of the condition may be compromised. Additionally, some patients may not report the condition or may allow it to evolve into the chronic form
Aetiology of phimosis/paraphimosis
The most common aetiology follows retraction of the foreskin of an uncircumcised male penis by a health professional during penile examination, catheterisation, or cystoscopy, who then neglects to return the foreskin back over the glans penis.
The coexistence of poor hygiene and recurrent bouts of bacterial infection (balanitis) makes this condition more likely. Additional causes of paraphimosis include sexual intercourse in an uncircumcised male with phimosis (narrow opening of the foreskin), varying sexual practices that cause constriction of the foreskin, penile piercings, parasitic infestations, lichen sclerosis, inadequate circumcision, or haemangiomas of the penis.
RFs for phimosis/paraphimosis
STRONG Lack of circumcision Urinary catheterisation Dependence on carer Tight foreskin Phimosis Poor hygiene Bacterial infection Parasitic infection Lichen sclerosis DM Penile piercing Haemoangiomas
WEAK
Peripheral vascular disease
Inadequate circumcision
Sx of phimosis/paraphimosis
COMMON Penile pain Band of retracted foreskin tissue beneath the glans Swollen glans penis Indwelling catheter
UNCOMMON
Black tissue on glans
Non-pliable glans penis
Inability to urinate freely
Ix for phimosis/paraphimosis
Clinical
Rx of phimosis/paraphimosis
Ischaemia and necrosis:
Urgent surgery
No Ischaemia/Necrosis
Manual manipulation
Surgical reduction
Prognosis of phimosis/paraphimosis
Following prompt reduction of paraphimosis, most patients will have a full recovery. Foreskin swelling will subside over subsequent days. Depending on the aetiology, appropriate education of the patient and/or carers is recommended. Some patients may not warrant circumcision. Patients who have evidence of phimosis should be counselled about the appropriateness of circumcision. In addition, those who have a repeated episode of paraphimosis or have undergone a dorsal slit procedure warrant follow-up circumcision.
Complications paraphymosis
Necrosis of glans+foreskin
A 72-year-old uncircumcised man is brought to the emergency department with concerns about penile swelling. He has a history of dementia and is in a facility for long-term care. He is having problems with incontinence and a decision was made to manage this with long-term Foley catheter drainage. The catheter was changed 2 days prior to presentation in the emergency department. On the day of presentation, the foreskin is noted to be retracted completely, with engorgement and oedema of the glans and pain on palpation.
Paraphymosis
A 6-month-old uncircumcised boy is brought to the paediatrician’s clinic with a 24-hour history of penile swelling. The parents have noted that the foreskin has been difficult to retract. During his bath the night before, the mother had finally been able to completely retract the foreskin. In the clinic, the foreskin remains retracted below the glans penis, which is oedematous and engorged. The infant appears to be in pain when the penis is manipulated
paraphymosis
Due to embarrassment, some patients may not present until necrosis of the glans penis has occurred. A chronic form may also occur, and in these patients a fibrotic ring and chronic engorgement and oedema of the glans penis are noted.
paraphymosis
Define pyloric stenosis
In infantile hypertrophic pyloric stenosis (HPS), hypertrophy of the pyloric sphincter results in narrowing of the pyloric canal. It is the most common cause of gastric outlet obstruction in the 2- to 12-week-old age group. Pyloric stenosis leads to progressive and projectile vomiting.
Recurrent projectile non-bilious vomiting, typically in a 3- to-6-week-old infant (usually male), but may occur on older infants.
Features may include a history of feeding intolerance with multiple formula changes.
Failure to thrive/weight loss may progress to increasing volume depletion. An olive-shaped mass may be palpable in the right upper abdomen.
Ultrasound shows pyloric channel length >17 mm and pyloric muscle thickness >4 mm.
Treatment is with intravenous fluid and electrolyte replacement, followed by pyloromyotomy (open or laparascopic).
Complications of surgery include wound infection, gastric or duodenal mucosal perforation, or incomplete myotomy.
Epidemiology of pyloric stenosis
2-4/1000 births
M:F 4:1
Aetiology of pyloric stenosis
Hyperacidity as a result of antral distention with feeding and hypertrophy of the pylorus from repeated contraction is believed to be a cause.
Additionally, poor pyloric muscle neuronal innervation is believed to play a role.
Nitric oxide synthase deficiency is also implicated as a biochemical cause, by decreasing smooth muscle relaxation.
Exposure to oral erythromycin, especially during the first 2 weeks of life, is associated with significant odds of developing pyloric stenosis.
RFs for pyloric stenosis
STRONG
1st born male infant
FHx
WEAK
Prematurity
Erythromycin in 1st 2 weeks life
Maternal exposure to macrolides
Sx of pyloric stenosis
COMMON RFs Non-bilious projectile vomiting 3-6 weeks old Upper abdominal mass Multiple formula changes Tachycardia (volume depletion) Decreased wet nappies Dry MM Flat/depressed fontanelles Constipation Poor weight gain Irritability
UNCOMMON
Peristaltic waves
Ix for pyloric stenosis
U+E - in severe cases, hypochloraemic alkalosis and mild hypokalaemia
USS abdomen - pyloric muscle thickness >4 mm, pyloric canal length >17 mm
Rx of pyloric stenosis
IV fluid resus - Intravenous fluid replacement should be provided at 1.5 times maintenance rate with 5% dextrose plus 0.45% saline.
Pyloromyotomy - after electrolyte balance corrected
Prognosis of pyloric stenosis
The success of surgical treatment is near 100% and complication rates are negligible.
Complications of pyloric stenosis
Post op emesis
Surgical wound infection
Surgical mucosal perforation
Incomplete myotomy
Define sepsis in children
Sepsis is a clinical syndrome resulting from a dysregulated immune response to infection. It is characterised by derangements in multiple pathobiological processes, which may lead to widespread tissue injury. It encompasses a clinical spectrum of severity, including severe sepsis, septic shock, and multi-organ failure. Sepsis is a leading cause of morbidity and mortality in children worldwide.
Initial clinical presentation may be non-specific (especially in younger age groups).
Given the time-critical nature of severe sepsis and septic shock, when sepsis is suspected on clinical grounds it is usually best to initiate sepsis investigations and treatment, including administering antibiotics and fluid resuscitation. These should continue until sepsis has been excluded.
Progression to organ failure and shock is often very rapid, so early recognition and treatment is crucial.
Empirical broad-spectrum antibiotic therapy (based on the most probable pathogens) should be administered as soon as possible, and always within the first hour following recognition.
Other treatments are primarily supportive, and should be delivered according to internationally recognised consensus-based guidelines.
Epidemiology of sepsis in children
Case fatality 8.9%
Roughly 7.7% of paediatric ICU admissions
Aetiology of sepsis in children
Early onset neonatal - think GBS, S.aureus, H influenza and coat neg staphylococci
Late onset neonatal - coagulation neg staphylococci
Infants + young children - S.pneumonia, N meningitidis, S aureus, H influenza
Fungal (e.g., Candida species, Aspergillus species) and viral (e.g., influenza, respiratory syncytial virus, human metapneumovirus, varicella, and herpes simplex virus) pathogens account for up to 5.3% and 2.9% of severe sepsis in children
RFs for sepsis in children
STRONG Immunodeficiency Comorbidities Younger age Perinatal RFs for infection - maternal fever, prolonged rupture of membranes (>18 hours), maternal carriage of group B streptococci, and chorioamnionitis; fetal factors include fetal distress and poor 5-minute Apgar score (i.e., ≤6). Healthcare associated factors: parenteral nutrition, mechanical ventilation, chronic central venous access, and use of vascular catheters (arterial, central venous, or peripheral). Recent surgery Breached skin integrity
WEAK
Male
Sx of sepsis in children
COMMON RFs Fever/low temp Tachypnoea Tachycardia Bradycardia (neonates/infants) Altered mental state Decreased perfusion Change in feeding Dec UO Mottling of skin, ashen appearance, cyanosis Low O2 sat
UNCOMMON Vasoplegia Non-blanching purpuric rash Hypotension Specific focal Sx
Ix for sepsis in children
FBC - inc WCC/thrombocytopenia Serum glucose BLOOD CULTURE - +ve Urinalysis - UTI focus Urine culture - UTI focus Blood gases - base deficit Serum lactate - >2 usually U+E - deranged Serum creatinine - elevated LFTs - abnormal Coagulation studies - abnormal CRP - elevated CXR - may reveal focus in older children
CONSIDER
LP - if suspect meningitis
HSV PCR blood/CSF
Rx of sepsis in children
SEPSIS 6:
1. Administer oxygen - keep >94%
2. Take blood cultures - think source control
3. IV BS-ABx
4. IV fluid if lactate >2mmol - 500STAT
NB lactate >4 = call critical care
5. Serial lactate - recheck after 10ml challenges
6. Measure UO + complete fluid balance chart
- Maintain airway
- Consider vasoactive-inotropic support, IE adrenaline/NA
0. 1-1microgram/kg/min
May need blood transfusion
Corticosteroid
EMPIRAL ABX:
Apicillin + Cefotaxime/Gentamicin
OR
BenPen + Gentamicin
CONSIDER
Aciclovir if severe sepsis (may be HSV1)
Nystatin - prevents candidiasis
IF neonatal - also give prostaglandins (Rx until duct-dependent cardiac lesion can be ruled out)
Prognosis of sepsis in children
Without treatment, severe sepsis carries a mortality rate in excess of 80%.
With treatment, overall mortality is approximately 10% in children up to the age of 19 years.
In children with cancer, overall mortality from sepsis is 17%. The rate increases to 30% in children who have undergone haematopoietic stem cell transplants.
Complications of sepsis in children
AKI Myocardial dysfunction DIC Hypoglycaemia Hyperglycaemia NEC MOF Persistent pulmonary hypertension of the newborn Hypocalcaemia Abdominal compartment syndrome Thyroid insufficiency Neurological sequelae
A 5-week-old, full-term male infant presents with progressive post-feeding emesis for the past 2 weeks. Initially he was diagnosed as having formula intolerance; formula type was changed several times without relief. Subsequently, he was thought to have gastro-oesophageal reflux. The parents continue to report non-bilious post-feeding emesis, which has become progressively forceful and projectile.
Pyloric stenosis
A previously well 1-year-old girl presents to the accident and emergency department with a history of lethargy and fever for 24 hours. She recently had symptoms suggestive of a viral upper respiratory tract infection. Her parents report that for a few hours prior to presentation she had become drowsy and difficult to rouse. They also report that they had noticed a rash developing on her trunk and limbs shortly before presentation. On initial assessment the following features are identified: reduced level of consciousness (response to painful stimulus only); tachycardia (heart rate 190 beats per minute); prolonged capillary refill time (>5 seconds peripherally); cold peripheries (core-toe temperature gap >10°C [>18°F]); fever (core temperature 39°C [102°F]); tachypnoea (respiratory rate 40 beats per minute) and grunting on expiration; and a widespread, non-blanching, purpuric rash on the trunk and limbs.
Sepsis
A 2-week-old preterm male neonate develops transient apnoeas and bradycardic episodes while in the neonatal intensive care unit. He had been born at 30 weeks’ gestation after spontaneous onset of preterm labour. He had required intubation and mechanical ventilation for 48 hours following birth for neonatal respiratory distress syndrome. Standard dosing of surfactant was administered during this time. He required respiratory support with continuous positive airway pressure for 1 week after his extubation, and was cycling on and off high-flow oxygen therapy at the time of this event. He had established full enteral feeding after a period of parenteral feeding via a percutaneous central venous catheter (long-line). The long-line was still in situ at the time of this event, and was planned for removal that day. In addition to the apnoeas and bradycardias, it was noted that he had temperature instability and increased capillary refill time (>3 seconds); both of these features were a change from the previous observation trends.
Sepsis
The typical presentation of sepsis varies according to the age of the child. Whereas older children often present with a focus of infection, infants and neonates usually present with non-specific symptoms and signs. For example, the early signs of sepsis in preterm infants are often apnoeas and bradycardias. In the neonatal population, including preterm infants, any change from the patient’s normal pattern of observations should raise the suspicion of sepsis.
Septic shock commonly presents as ‘cold shock’ with profound peripheral vasoconstriction and impaired myocardial contractility. However, another mode of presentation is ‘warm shock’ characterised by systemic vasoplegia (dilated peripheral vasculature and ‘flash’ capillary refill) with a high cardiac output and bounding pulses. Studies suggest that this mode of presentation is more common in hospital-acquired sepsis.
Sepsis
SIRS classification
Systemic inflammatory response syndrome (SIRS):
Generalised inflammatory response defined by the presence of 2 or more of the following criteria (abnormal temperature or white cell count must be one of the criteria):
Abnormal core temperature (<36°C or >38.5°C [<97°F or >101°F])
Abnormal heart rate (>2 standard deviations above normal for age, or <10th percentile for age if child is <1 year of age)
Raised respiratory rate (>2 standard deviations above normal for age, or mechanical ventilation for acute lung disease)
Abnormal white cell count in circulating blood (above or below normal range for age, or >10% immature white cells).
Define slipped capital femoral epiphysis
Slipped capital femoral epiphysis (SCFE) is the most common hip disorder in the adolescent age group. It occurs when weakness in the proximal femoral growth plate allows displacement of the capital femoral epiphysis. SCFE is a misnomer; it is the metaphysis that displaces anteriorly and superiorly, leading to the slipped state.
May present with an acute/insidious onset of pain and limp.
The disorder is typically seen in the adolescent age group.
Associated systemic disease is a common feature.
Obligatory external rotation on hip flexion is a key examination finding.
Recommended surgical treatment is in situ pinning; prophylactic fixation of the contralateral hip may be necessary when concomitant metabolic disease is present.
Epidemiology of slipped capital femoral epiphysis
Roughly 1 in 10,000
M>F
Average age of onset was 11.6 years for girls and 12.6 years in boys.
Aetiology of slipped capital femoral epiphysis
The aetiology of SCFE is unknown in most cases. Obesity is recognised as the most strongly associated risk factor. Obesity increases the shear stress across the physis, weakens it, and causes the characteristic displacement in SCFE. Hormonal involvement associated with the adolescent growth spurt may also provide insight into the aetiology of SCFE. The physis weakens at puberty, possibly due to the effect of circulating gonadotrophins. The weakening effect of testosterone on the physis offers a causative hypothesis for the high incidence in males. The lower incidence in female adolescents may be due to oestrogen increasing the strength of the physis and narrowing its width. SCFE is associated with endocrine disorders, but the vast majority of children with SCFE are obese. Radiotherapy is also thought to be a risk factor for the condition. There is no evidence of a genetic predisposition.
RFs for slipped capital femoral epiphysis
STRONG
Puberty
Obesity
Endocrine disorders
WEAK Male sex Ancestry Geographic region Prior radiotherapy
Sx of slipped capital femoral epiphysis
COMMON
Weight >90th centile
RFs
Gait with affected leg externally rotated
Groin/Knee referred pain
Bilateral hip pain - bilateral in 60%
Tendelenburgs gait
Restricted range of motion
UNCOMMON Weight <50th centile Symptoms of hypothyroidism/panhypopituitarism Renal failure - renal osteodystrophy Recent trauma
Ix slipped capital femoral epiphysis
Bilateral anteroposterior xray - Klein’s line does not intersect the femoral head
Frog-leg lateral x-rays - Klein’s line does not intersect the femoral head; Bloomberg’s sign positive
Rx of slipped capital femoral epiphysis
Surgical repair
Prophylactic fixation of contralateral hip
Prognosis of slipped capital femoral epiphysis
Prognosis is related to the initial severity of the slip, success of surgery, avoidance of serious complications, underlying disorders, and bilaterality. A study of stable SCFE cases treated with surgical dislocation of the hip noted that 87% of cases had some degree of labral damage and 85% of cases had some degree of cartilage damage.
Complications of slipped capital femoral epiphysis
Chondrolysis
Late deformity
Contralateral hip SCFE
Osteonecrosis
A 13-year-old boy presents with hip, groin, thigh, and medial knee pain. He is overweight and recently experienced an adolescent growth spurt. On physical examination, the affected leg is externally rotated and there is limited range of motion in the hip joint. He is unable to bear weight on the affected leg.
Slipped capital femoral epiphysis
Define sudden infant death syndrome (SIDS)
“The sudden death of an infant under 1 year of age, which remains unexplained after a thorough case investigation, including performance of a complete autopsy, examination of the death scene, and a review of the clinical history”.
Leading cause of infant death beyond the neonatal period.
Incidence roughly 1 in 2000 infants.
Peak incidence between 1 and 3 months of age, although events may occur up to 12 months of age.
Risk factors include exposure to tobacco smoke (antenatally and postnatally); prone and side sleeping; bed-sharing during sleep; a sleep environment that includes soft mattress and/or sleeping surface (including sofa or armchair), and placement of soft sleep bedding; over-bundling/over-heating; prematurity; exposure to alcohol and illicit drugs (antenatally and postnatally); and viral infection. Several risk factors may co-exist in any given patient.
Protective factors include use of a dummy during sleep, immunisation, and breastfeeding.
Careful evaluation of death by trained forensics teams is necessary to rule out other causes of death, including suffocation, asphyxia, entrapment, infection, ingestions, metabolic diseases, arrhythmia-associated cardiac channelopathies, and trauma (accidental or non-accidental).
What is the epidemiology of sudden infant death syndrome (SIDS)
SIDS remains the leading cause of post-neonatal infant death.
1 in 2000 infants
Peak incidence 1-3months
M>F
Black>white
Aetiology of sudden infant death syndrome (SIDS)
Aetiology is currently unknown. Indeed, the identification of a specific aetiology/acute process in an infant as a cause of death is sufficient to eliminate the diagnosis of SIDS.
The frequent autopsy finding of mild upper airway inflammation in suspected cases of SIDS raises the possibility that intercurrent respiratory tract infections have a role in SIDS, particularly in combination with over-bundling.
Associations with other modifiable risk factors such as sleeping position, sleeping surface, sleeping arrangements, smoke exposure, not breastfeeding, and non-use of a dummy have been well established, although direct causative roles have yet to be determined.
These 3 key risk categories/factors are:
A vulnerable infant (e.g., prematurity, low birth weight, disordered autonomic regulation)
A critical period during homeostatic control development (e.g., cardiorespiratory regulatory mechanism maturation)
An exogenous stressor (e.g., smoke exposure, sleep environment, over-heating).
RFs of sudden infant death syndrome (SIDS)
STRONG Side/prone position at last sleep Bed sharing Soft sleeping environment Maternal cigarette smoking (ante+post) Increased number smokers in house Formula feeding Non-use of dummy Premature birth
WEAK Maternal substance abuse Single mother Dec maternal age Low level antenatal care Low maternal education Low socio-economic status Black/american indian Lack of immunisations
Sx of sudden infant death syndrome (SIDS)
Major risk factors include prone position at last sleep, bed-sharing, soft sleeping surface/environment, maternal cigarette smoking, increasing number of smokers in house, smoking in same room as child, premature birth, formula feeding, non-use of pacifier, and maternal alcohol or drug use.
No factor appears to be sufficient, in itself, to initiate a SIDS event. Risk factors should be placed in the Triple-Risk hypothesis model and the aggregate of co-existing factors used to evaluate the risk of a SIDS event in any given patient.
Ix for sudden infant death syndrome (SIDS)
Everything is negative
Autopsy inconclusive
Rx of sudden infant death syndrome (SIDS)
Carer follow up + grief counselling
Prognosis of sudden infant death syndrome (SIDS)
After a SIDS event, counselling is given to the parents and other carers and surviving siblings. It may be necessary to continue counselling for a period of time, as not all maladaptive behaviours may manifest immediately after the event. Parents should be offered testing for metabolic and genetic conditions that may mimic SIDS.
For parents considering pregnancy again, reassurance can be given that the likelihood of another SIDS event is extremely low. After birth of subsequent children, healthcare providers can consider recommending more frequent follow-up, as much for parental reassurance as for monitoring the infants.
Define conversion and somatic symptom disorders
Conversion and somatic symptom disorders are psychiatric conditions that fall under the somatic symptoms and related disorders category of the DSM-5. Somatic symptom and related disorders are those with distressing physical symptoms that are not fully explained by other medical, neurological, or psychiatric disorders (also known as somatisation).
Conversion disorder is characterised by voluntary motor or sensory function deficits that suggest neurological or medical conditions but are rather associated with clinical findings that are not compatible with such conditions. Somatic symptom disorder is characterised by one or more somatic symptoms that are distressing or result in significant disruption of daily life. To meet DSM-5 criteria, these patients must have excessive thoughts, feelings, or behaviours related to the somatic symptoms or associated health concerns as manifested by at least one of the following: disproportionate and persistent thoughts about the seriousness of one’s symptoms; persistently high levels of anxiety about health or symptoms; excessive time or energy devoted to these symptoms or health concerns. Importantly, even if any one somatic symptom is not continuously present, the state of being symptomatic is persistent (typically more than 6 months).
Conversion and somatic symptom disorders are both categorised as somatic symptom and related disorders, previously termed somatoform disorders.
Somatic symptom and related disorders are psychiatric conditions where patients experience distressing physical symptoms that are not fully explained by other medical, neurological, or psychiatric disorders, as well as abnormal thoughts, feelings, and behaviours in response to these symptoms. They may result from psychological stress that is unconsciously (without awareness) expressed somatically, though the underlying cause is not fully understood.
Risk factors include being female, having a history of abuse or adverse childhood events, and having personality traits of alexithymia (difficulty expressing emotions) or neuroticism. Symptoms that persist with an external focus of control, and without awareness of the psychological and stress-related interplay, can lead to considerable functional impairment and distress.
Diagnosis is made by clinical interview, behavioural observation, physical examination suggestive of pseudoneurological causes, and tests to rule out medical or neurological causes. The diagnosis should not be made solely on the basis of medically unexplained symptoms; rather, it should be based on evidence from the clinical examination and the patient’s abnormal thoughts, feelings, and behaviours in response to the medically unexplained symptoms.
Good doctor-patient relationships and validation of the patient’s suffering are essential for effective management. Treatment includes cognitive behavioural therapy and avoiding unnecessary medicines, tests, and procedures. Diagnosis and treatment of associated comorbid psychiatric conditions benefit overall functioning and recovery.
Long-term management involves interrupting perpetuating factors, maintaining the same doctor, and providing strategies for self-efficacy, distress tolerance, coping, and modulating the interaction of anxiety, stress, and physical symptoms.
Aetiology of conversion and somatic symptom disorders
Chronic and/or acute intrapsychic (emotional/psychological) stress or conflict, in combination with either emotional processing deficits, avoidance tendencies, or social, cultural, or family taboos against emotional expression, seems to be important in the aetiology of somatisation. Societal attitudes in which people exhibiting psychological distress are less tolerated than those who are physically ill are also likely to reinforce somatisation.
Cognitively, disturbances in attention and control through dissociation, misattribution, and misinterpretation probably contribute to both conversion and somatic symptom disorder symptoms.
Epidemiology of conversion and somatic symptom disorders
75% female
Conversion:
5-22/100k
Early-mid adulthood
Somatic:
10% to 15% of primary care patients have multiple unexplained symptoms that are present for >2 years
The prevalence of somatic symptom disorder is not known but may be around 5% to 7%
RFs of conversion and somatic symptom disorders
Hx sexual/physical abuse Hx unstable childhood Hx trauma related disorders Female Alexithymia Neuroticism Previously poor doctor patient relationships
Sx of conversion and somatic symptom disorders
Recent stressors Unusual neurodeficits Give-way weakness False sensory findings Distractible symptoms Inconsistent paralysis "seizures with awareness" Bizarre movements Gait disorders Cognitive complaints Speech disturbance Swallowing disturbance
Psuedoclonus
Convergence spasm
Ix of conversion and somatic symptom disorders
Lab - N
EEG - N
Personality testing - mental illness trait, personality disorder trait, hypochondriasis, depression, hysteria, anxiety, somatisation, pain disorder, or health concern
Rx of conversion and somatic symptom disorders
Psychotherapy
CBT
Hypnosis
Biofeedback training
benzodiazepines 1-2mg lorazepam, 5-10mg diazepam
Antidepressant - sertraline 25mg, fluoxetine 10mg, citalopram 10mg
Biofeedback training
Atypical antipsychotic Aripiprazole 2mg Quetiapine 25mg lanzipine 2.5mg Risperidone 0.5mg
ECT
Prognosis of conversion and somatic symptom disorders
CONVERSION
Between 50% and 90% of patients with conversion disorder exhibit short-term resolution of symptoms after reassurance, but up to 25% of these responders relapse or develop new conversion symptoms over time.
Favourable prognostic indicators for patients with conversion disorder include acute onset of symptoms, precipitation by a well-defined stressful event, good premorbid health, and absence of psychiatric or neurological comorbidities
SOMATIC
Reported remission rates range from very low (<10%) [116] to a 50% recovery within 1 year.
Patients with many somatic symptoms, anxiety or depression, and old age or marked impairment were more likely to have persistent symptoms.
Complications of conversion and somatic symptom disorders
Depression
Anxiety
Suicidal ideation
Substance abuse
A 21-year-old woman presents to the accident and emergency department with acute onset of left-sided body twitching after a minor accident in which she hit her head. Physical examination, laboratory investigations, and imaging studies are normal. Over the next several weeks, she begins experiencing episodes of full-body movements. In the week before the initial symptom onset, her boyfriend (for whom she also worked) broke up with her, and she had a conflict with her parents. She and her boyfriend have since reconciled. There is no reported history of abuse, but there is a family dynamic of high expectations of the patient. Neurological examination and EEG during a typical episode are normal.
Conversion and somatic symptom disorders
A 35-year-old married woman seeks treatment for pelvic pain. On review of systems, she reports several years of various symptoms, including GI problems (constipation, abdominal pain, nausea, and vomiting); headaches; vulvodynia; fatigue; all-over body pains; paraesthesias; and several sensitivities to environmental factors and medications. She has visited the accident and emergency department on several occasions and has been previously admitted to hospital for persistent GI symptoms. Extensive GI work-up, laboratory studies, and laparoscopy to rule out endometriosis have been unrevealing. She reports a stressful childhood, with an alcoholic father and sexual abuse by her grandfather.
Conversion and somatic symptom disorders
Male, middle-aged, and older patients are also seen with conversion and somatic symptom and related disorders, although this is less common. Importantly, these disorders can coexist with other medical and neurological conditions. Previous health-related traumatic experiences (e.g., myocardial infarction, cancer) in a patient with high illness anxiety is associated with development of somatisation.
Conversion and somatic symptom disorders
Define acute sinusitis
Acute sinusitis (also commonly known as acute rhinosinusitis) is a symptomatic inflammation of the mucosal lining of the nasal cavity and paranasal sinuses, where clinical symptoms have been present for 4 weeks or less. It can be caused by either a viral or a bacterial infection.
Majority of cases in adults and children are of viral aetiology.
Duration of symptoms more than 10 days often indicates bacterial cause.
Imaging is not required for diagnosis unless complications are suspected.
Condition is usually self-limiting; however, symptomatic therapy is recommended.
Antibiotics are only recommended in select patient groups (e.g., immunocompromised or with severe disease).
Epidemiology of acute sinusitis
16% of adults
0.5-13% of viral -> bacterial
13% of children by age 3
F>M
School-age children on average contract 6 to 8 upper respiratory tract infections per year, and of these, 5% to 10% will be complicated by sinusitis.
Aetiology of acute sinusitis
The most common cause of acute sinusitis is a viral infection.
he three most common bacteria are Streptococcus pneumoniae (20% to 43% of cases), Haemophilus influenzae (22% to 35% of cases), and Moraxella catarrhalis (2% to 10% of cases). M catarrhalis is less common in the adult population.
RFs for acute sinusitis
STRONG
Viral URTI
Allergic Rhinitis
WEAK
GORD
- Gastroesophageal reflux into the nasopharynx can cause symptoms that may be mistaken for sinusitis. In the paediatric population, it is associated with chronic sinusitis but does not contribute to acute sinusitis.
Sx of acute sinusitis
COMMON <10 day Sx (viral) >10 days <4w (bacterial) Symptoms that worsen after an initial improvement Purulent nasal discharge Blocked nose Swollen septum Facial pain/pressure
UNCOMMON
Severe symptoms at onset
Dental pain (acute maxillary sinusitis)
Other COMMON Cough Myalgia Sore throat Hyposmia Oedematous turbinate
UNCOMMON
Fever
Rx for acute sinusitis
Suspected Viral:
Supportive
Antipyretic: paracetamol/ibuprofen
Decongestant eg pseudoephedrine
Intranasal corticosteroid possible:
mometasone nasal
Decongestant:
Ipatropium nasal
Saline nasal irrigation
Mucolytic:
Guaifenesin
BACTERIAL
- amoxicillin
- amoxicillin/clavulanate
- clindamycin
Ix for acute sinusitis
Clinical diagnosis
IF refractory to Rx/immunocompromised then nasal endoscopy possible
Sinus culture
Prognosis of acute sinusitis
In general, acute sinusitis is a self-limiting disease and generally resolves within 1 month. However, the use of antibiotics in appropriately selected patients may limit the length and severity of symptoms.
Complications of acute sinusitis
Complications are more commonly seen in the paediatric population, and occur due to direct extension of the infection into neighbouring structures. Orbital spread of infection with orbital cellulitis or orbital abscess represent the most common complications. Intracranial spread of infection resulting in meningitis or abscess is much less common.
Cavernous sinus thrombosis
A 19-year-old woman presents with a 12-day history of purulent nasal drainage and nasal congestion, and reports a history of fever, myalgia, and facial pressure. She is otherwise healthy and works as a teacher. After 5 days of illness, the patient’s symptoms started to improve; however, they have worsened in the last few days, despite the use of over-the-counter medications. Physical examination shows oedematous mucosa of the inferior turbinate. There is also thick mucus in the nasal cavity. Nasal endoscopy demonstrates purulent drainage and a small polyp in the ostiomeatal complex. The adenoids are small and erythematous.
Acute sinusitis
A 33-year-old man with a medical history of paediatric-onset asthma, atopic dermatitis, and allergic rhinitis presents with a 7-day history of facial pressure, dental pain, nasal blockage, and hyposmia. The patient developed these symptoms after recently mowing his lawn. The symptoms have not improved despite use of an intranasal corticosteroid, an antihistamine, and intranasal saline washes. Physical examination shows a septum deviated to the left side, and a large concha bullosa on the right side. There are no polyps, but there are swollen turbinates and thin, clear mucus present.
Acute
Define otitis externa
Acute otitis externa (AOE) is defined as diffuse inflammation of the external ear canal, which may also involve the pinna or tympanic membrane.
It is a form of cellulitis that involves the skin and subdermis of the external auditory canal, with acute inflammation and variable oedema.
It is most commonly caused by bacterial infection. The diagnosis of AOE requires the presence of rapid onset (generally within 48 hours) of symptoms within the past 3 weeks, coupled with signs of ear canal inflammation.
Epidemiology of otitis externa
AOE has a lifetime incidence of 10%.
Peak 7-12 yo
Aetiology of otitis externa
Most commonly caused by bacterial infections. It is often polymicrobial, but the most common pathogens are Pseudomonas aeruginosa (20%-60% prevalence) and Staphylococcus aureus (10%-70% prevalence).
Other aetiologies are idiopathic, trauma (from scratching, aggressive cleaning), chemical irritants, allergy (most commonly to antibiotic ear drops such as neomycin), high-humidity conditions, swimming, or skin disease (allergic dermatitis, atopic dermatitis, psoriasis).
RFs for otitis externa
STRONG External auditory canal obstruction High humidity Warmer temperatures Swimming Local trauma Allergy Skin diseas DM Immunocompromised Prolonged use topical antibacterials
WEAK
Chemical irritants
Sx of otitis externa
COMMON RFs Ear pain Tragal tenderness Ear canal swelling/erythema Otorrhoea Aural fullness Itching Dec hearing Erythematous timpanic membrane
UNCOMMON
Granulation tissue in ear canal - malignant otitis externa
Ix otitis externa
Pneumatic Otoscopy - Normal (unless also otitis media)
Typanometry - N (unless also otitis media)
Ear culture - if failed response
Rx of otitis externa
Antibacterial drops
ciprofloxacin, can include hydrocortisone too
or acetic acid
Refractory -> oral cipro
Fungal -> acetic acid or tolnaftate topical
Malignant/necrotising = needs debridement
Prognosis of otitis externa
Patients with uncomplicated diffuse otitis externa usually respond to treatment. Between 65% and 90% of patients have clinical resolution within 7 to 10 days, regardless of agent used.
The mortality rate of malignant otitis externa has decreased over the years from 50% to 0%-15%.
Complications of otitis externa
Contact dermatitis
Cranial nerve palsy (facial-> usually malignant/./nec otitis ext)
Osteomyelitis
A 35-year-old man presents with a 2-day history of rapid-onset severe ear pain and fullness. The patient complains of otorrhoea and mild decreased hearing. He reports that his symptoms started after swimming. No fever is reported. On physical examination the external ear canal is diffusely swollen and erythematous. He has tenderness of the tragus and pain with movement of the auricle. The tympanic membrane was partially visualised due to the swelling. The concha and the pinna look normal. Neck examination fails to reveal any lymphadenopathy.
otitis externa
Malignant or necrotising otitis externa is a form of otitis externa that is more common in older patients with uncontrolled diabetes or in patients with immunodeficiency. [1] [4] In malignant otitis externa, the infection and the inflammatory process involve not only the skin and soft tissue of the external auditory canal but the bone tissue of the temporal bone as well. [5] If left untreated, osteomyelitis of the petrous bone and/or skull base could result. [5] [6] It is most commonly caused by Pseudomonas species. [1] [5] Patients usually present with severe ear pain, otorrhoea, and fullness, and are not responding to the conventional treatment of AOE. Depending on the stage of presentation and the extent of invasion, patients may have facial weakness and other cranial nerve abnormalities. [1] On physical examination the external auditory canal is swollen, with evidence of granulation tissue in the floor of the canal. [1] The diagnosis is usually made by CT or MRI scans, which show presence of soft tissue and bone destruction. [5] Technetium-99 or gallium scans will show increased radioisotope uptake in the temporal bone and/or skull base
otitis externa
Otomycosis is fungal otitis externa. Acute fungal otitis externa is less common than acute bacterial otitis externa. [1] It is most commonly caused by Aspergillus species. [3] It presents in a similar way with ear pain, itching, aural fullness, and otorrhoea. Physical examination reveals swollen ear canal skin and discharge. The presence of black spores indicates Aspergillus niger as the causative organism. [1] [3] White filamentous hyphae can often be seen. The definitive diagnosis of otomycosis can be helped by microscopic examination and ear cultures. Otomycosis should be suspected in patients who fail treatment with antibacterial agents. [3] Secondary fungal infection of the external auditory canal is well known after prolonged treatment with topical antibacterial agents
otitis externa
Define acute otitis media
Acute otitis media (AOM) is an infection involving the middle ear space and is a common complication of viral respiratory illnesses.
May present with otalgia, irritability, decreased hearing, anorexia, vomiting, or fever, usually in the presence of an ongoing viral respiratory infection.
Physical examination will reveal a bulging, opacified tympanic membrane with decreased mobility. The membrane may be white, yellow, pink, or red.
Diagnosis is generally made with conventional otoscopy. Additional tests might include pneumatic otoscopy or tympanometry to confirm the presence of an effusion.
Treatment includes pain control with analgesics and might include antibiotics.
Complications include perforation of the tympanic membrane and, rarely, mastoiditis, seventh cranial nerve palsy, or sigmoid sinus thrombosis.
Epidemiology of acute otitis media
More than 80% of children experience at least one episode of AOM before the age of 2 years with a peak incidence between 6 and 18 months.
Children with anatomical anomalies (e.g., cleft palate, cleft uvula) or immunological deficiencies encounter more AOM than their peers. Environmental risk factors include childcare attendance, exposure to older siblings, exposure to tobacco smoke, absence of breastfeeding, bottle feeding in a supine position, and dummy use. A higher incidence among boys, children with a family history of AOM, and certain ethnic groups (Native Americans and Native Alaskans) suggests a genetic susceptibility.
RFs of acute otitis media
STRONG Nursery Older siblings Young age FHx Absence of breast feeding Supine feedings Low socio-econ Craniofacial anomaly Immunological deficiency
WEAK
Male
Dummy use
Env tobacco exposure
Aetiology of acute otitis media
Respiratory viruses account for most cases of otitis media and are self-limiting. Co-infections of the middle ear with a virus and a bacterium demonstrate the role that both play in the development of acute, suppurative otitis media or pus drum.
The most common bacteria responsible for AOM are Streptococcus pneumoniae (approximately 40%), non-typable Haemophilus influenzae (25% to 30%), and Moraxella catarrhalis (10% to 15%).
Unfortunately, an aetiological diagnosis in a clinic setting is not readily feasible.
Sx of acute otitis media
COMMON Otalgia Preceding URTI Bulging tympanic membrane Myringitis Irritability Sleep disturbance Fever
UNCOMMON
Decreased appetite
Ix for acute otitis media
Clinical
Typanometry - not required but - flat (B) curve (low compliance)
Define non-allergic rhinitis
Non-allergic rhinitis (NAR) refers to a group of chronic rhinitis sub-types that are not caused by allergy or infection. At least eight sub-types have been proposed, including vasomotor rhinitis (VMR [also known as ‘autonomic rhinitis’, ‘non-allergic rhinopathy’, and ‘idiopathic non-allergic rhinitis’]), non-allergic rhinitis with eosinophilia syndrome (NARES), atrophic rhinitis, senile rhinitis, gustatory rhinitis, drug-induced rhinitis, hormonal rhinitis, and occupational rhinitis. VMR is the most common sub-type.
To establish a definitive diagnosis of NAR, all other chronic rhinitis syndromes should be properly considered and excluded.
Environmental tobacco smoke, perfumes and fragrances, as well as temperature and barometric changes may aggravate symptoms in NAR, but specific IgE responses by skin or serological testing are all negative. The presence of eosinophils in the nasal mucosa in NARES distinguishes it from other sub-types of NAR.
It is a chronic condition that should be distinguished from a common cold, which can manifest with symptoms of NAR but is self-limiting.
Traditional triggers such as cat or dog exposure should be absent.
Symptoms and examination findings can overlap between perennial allergic rhinitis and non-allergic rhinitis (NAR), with nasal turbinates swollen and beefy red, scant mucus, cobblestoning of posterior pharynx from chronic post-nasal drainage, and retraction of tympanic membranes indicating congestion.
A diagnosis of NAR requires negative specific IgE responses by skin or serological testing.
Differentiation between non-allergic rhinitis with eosinophilia syndrome and other sub-types of NAR is determined by the presence or absence of eosinophilia in the nasal passage.
Treatment is based on symptoms, and all patients should be counselled on avoidance of triggers. Symptom control in NAR requires a balance between the control of excess secretions and over-suppression. First-line treatments include intranasal corticosteroids, intranasal antihistamines, and intranasal ipratropium.
Structural problems or other complicating conditions should be ruled out with imaging if initial therapeutic trials fail to relieve symptoms. Possibilities include osteomeatal complex obstruction that occurs as a result of chronic inflammation or recurrent infections, severe nasal septal deviation and nasal polyposis, or, less commonly, tumour or foreign body.
Aetiology of non-allergic rhinitis
It has been proposed that, in response to local mucosal injury, a compensatory response might over-amplify axonal release of neuropeptides, which then initiate the vascular and glandular responses characteristic of NAR.
The most popular mechanism postulated for vasomotor rhinitis patients has been an autonomic imbalance between the sympathetic and parasympathetic nervous system, resulting in parasympathetic hyperactivity leading to nasal congestion and drainage.
Parasympathetic responses are largely responsible for increased blood flow leading to engorgement of venous sinusoids, plasma extravasation, and increased release of secretions of mucus-secreting cells.
Otherwise, as a diagnosis based on exclusion, the aetiological factors in NAR are also largely negative risk factors. It is more likely to be the diagnosis in those with no family history of allergies, onset of symptoms after age 35, no seasonal symptoms, and no symptom exacerbation by cats. It is positively associated with triggers such as perfumes/fragrances and pot pourris.
Epidemiology of non-allergic rhinitis
Prev = 17-52%
Prevalence in children is unknown but is probably under-diagnosed due to the difficulty of children recognising and communicating signs and symptoms.
RFs for non-allergic rhinitis
STRONG
Symptom onset >35y
No exacerbation with cats
Neg FHx of allergy
WEAK No seasonal changes Fragrance exposure Smoke exposure Exposure to hairspray Exposure to pot pourris Change in temperature Barometric changes High oestrogen state BBlocker use Intranasal drug abuse
Sx of non-allergic rhinitis
COMMON Presence of RFs Nasal congestion PN-drip Rhinorrhoea Sore throat Ear plugging Sinus headaches Postural change triggers Sneezing
UNCOMMON
Purulent discharge
Bilateral symptoms
Sinus pain
Ix for non-allergic rhinitis
Allergy skin testing - Negative - exclude AR
Serological specific IgE testing - Negative
Consider
Nasal provocation test - symptoms return
Rx of non-allergic rhinitis
Trigger avoidance
Intranasal antihistamine - Azelastine BD
Saline nasal irrigation
Intranasal corticosteroid
COMBINATION
Oral antihistamines
Intranasal ipatropium
Surgery possible but rarely done - very very final option:
CO2 turbinectomy and partial laser turbinectomy
Complications of non-allergic rhinitis
Decongestant SEs: insomnia, tachycardia, increased blood pressure, and urinary retention and can aggravate underlying cardiac conditions
headache
Recurrent sinusitis
SEs nasal corticosteroids: bleeding/ulceration
SEs of atihistamines: excessive dryness and can aggravate benign prostatic hypertrophy
Eustacian tube dysfunction
Complications of non-allergic rhinitis
Decongestant SEs: insomnia, tachycardia, increased blood pressure, and urinary retention and can aggravate underlying cardiac conditions
headache
Recurrent sinusitis
SEs nasal corticosteroids: bleeding/ulceration
SEs of atihistamines: excessive dryness and can aggravate benign prostatic hypertrophy
Eustacian tube dysfunction
Define tonsillitis
Acute tonsillitis is an acute infection of the parenchyma of the palatine tonsils. This definition does not include tonsillitis as part of infectious mononucleosis, although tonsillitis may occur in isolation or as part of a generalised pharyngitis. The clinical distinction between tonsillitis and pharyngitis is unclear in the literature, and the condition is often referred to simply as “acute sore throat”.
Can be difficult to distinguish clinically from viral pharyngitis.
Most patients do not seek medical help.
Most cases resolve spontaneously and do not require antibiotics. Antibiotics are used to treat group A beta-hemolytic streptococcal infection.
There is some evidence that tonsillectomy may be effective in selected children with recurrent severe acute tonsillitis.
_______________________
Centor criteria [18]
The Centor criteria give an indication of the likelihood of a sore throat being due to bacterial infection. The criteria are:
Tonsillar exudate Tender anterior cervical adenopathy Fever over 38°C (100.5°F) by history Absence of cough. If 3 or 4 of Centor criteria are met, the positive predictive value is 40% to 60%. The absence of 3 or 4 of the Centor criteria has a fairly high negative predictive value of 80%.
Epidemiology of tonsillitis
In UK general practice, recurrent sore throat has an annual incidence of 100 per 1000 population.
Acute tonsillitis is more common in children between the ages of 5 and 15 years.
The prevalence of bacterial tonsillitis, specifically group A beta-haemolytic streptococci (GABHS), is 15% to 30% of children with sore throat and 5% to 15% of adults with sore throat.
Acute tonsillitis is most commonly seen in winter and early spring in temperate climates, although it may occur at any time of the year.
Aetiology of tonsillitis
Tonsillitis is usually viral; it is most commonly caused by the rhinovirus, followed by the coronavirus, and the adenovirus. Less commonly it is caused by the influenza virus, the parainfluenza virus, enteroviruses, or herpes viruses. In tonsillitis associated with infectious mononucleosis, the most common infective agent is the Epstein-Barr virus.
Common bacterial pathogens include beta-haemolytic and other streptococci, with the most common being group A beta-haemolytic streptococci (GABHS). GABHS is responsible for 15% to 30% of all cases of acute tonsillitis in children aged between 5 and 15 years and for 5% to 10% of all tonsillitis in adults. Group C beta-haemolytic streptococci are the cause in about 5% of patients.
Rarer bacterial causes include Mycoplasma pneumoniae and Neisseria gonorrhoeae , which can be the cause in sexually active adolescents (particularly those engaging in oral-genital sex).
RFs of tonsillitis
WEAK
Age 5-15
Contact with infected poeple
Sx of tonsillitis
COMMON RFs presence Pain on swallowing Fever >38 Exudate: particularly when it is caused by group A beta-haemolytic streptococci Sudden onset Sx Tonsillar erythema/enlargement Lymphadenopathy
Associated with infective process: Headache Abdominal pain N+V Presence cough/runny nose
Ix for tonsillitis
Throat culture - not nec routine
Rapid strep antigen test - identification of group A beta-haemolytic streptococci (GABHS) [lower sensitivity but fast]
WCC - rule out infectious mononucleosis - A raised WBC count with neutrophilia is suggestive of a bacterial infection, whereas a raised WBC count with lymphocytosis and atypical lympocytes is suggestive of infectious mononucleosis.
Heterophile antibodies - inf mono
Vaginal and cervical, or penile, and rectal cultures - if suspect N gon
Persistent/recurrent symptoms - check HIV
Rx for tonsillitis
Not due to strep - manage with analgesics para/ibu/aspirin/naproxen
If Group A strep:
Analgesia
+ ABx:
Ben-Pen, amoxicillin, azithromycin, clarithromycin
SEVERE swelling + group A
- Consider corticosteroids
Recurrent episodes:
Consider tonsillectomy
Prognosis of tonsillitis
Acute tonsillitis is an acute, self-limiting infective condition that normally resolves completely within 1 week with no sequelae.
In vulnerable people (e.g., infants, very old people, immunosuppressed or immunocompromised patients), tonsillitis may run a more severe course. Antibiotics and/or admission to hospital for a limited period of time may be advisable.
Very rarely (but more commonly in developing countries) acute tonsillitis may be associated with significant suppurative complications, such as neck abscess, and non-suppurative complications, such as rheumatic fever or acute glomerulonephritis.
Complications of tonsillitis
Scarlet fever: A diffuse erythematous rash, which is a manifestation of delayed-type skin reactivity to pyrogenic exotoxin (erythrogenic toxin, usually types A, B, or C) produced by Streptococcus species.
Characteristically the rash blanches with pressure and has multiple small papules. It generally starts on the head and neck, and is associated with circumoral pallor and a strawberry tongue. It subsequently spreads to the trunk, sparing the palms and soles, and is more marked over the skin folds.
Acute sinusitis Acute Otitis media Peritonsillar / neck abscess Acute rheumatic fever Acute post-streptococcal glomerulonephritis (manifesting with haematuria and oedema) Streptococcal toxic shock syndrome
Paediatric Autoimmune Neuropsychiatric Disorders Associated with Streptococci (PANDAS) syndrome - not on DSM but has been described
Define epiglottitis
Epiglottitis is a cellulitis of the supraglottis with the potential to cause airway compromise, and should be treated as a surgical emergency until the airway is examined and secured.
Pertinent diagnostic criteria include the classic ‘tripod’ position of the patient, drooling, high fever, and a toxic appearance.
A cellulitis of the supraglottis that may cause airway compromise. An airway emergency, especially in children, and precautionary measures must be taken.
Epiglottitis is classically described in children aged 2 to 6 years of age; however, it may manifest at any age, including in newborns. It may now be more common in older children and adults due to the Haemophilus influenzae type B (Hib) vaccine.
The most common signs are rapid onset of high fever, sore throat, inability to control secretions, classic tripod positioning, difficulty breathing, and irritability. Adults may have a more indolent presentation and may not require airway intervention (only about 20% of adults require intubation whereas most children do).
Cooperation between the otolaryngologist, paediatrician, or emergency department physician and anaesthetist is crucial.
No action should be taken that could stimulate a child with suspected epiglottitis, including examination of the oral cavity, starting intravenous lines, blood draws, or even separation from a parent. Similar caution is required in fulminant acute epiglottitis in adults.
Diagnosis is made on clinical grounds, and laboratory or other interventions should not preclude or delay timely control of the airway in a suspected case of epiglottitis.
Once the airway has been secured and antibiotics have been initiated, the condition usually resolves rapidly.
Vaccination does not preclude the possibility of epiglottitis or even the possibility of epiglottitis from Haemophilus influenzae.
Epidemiology of epiglottitis
Most 3-5yo
4.9 per 100k
Aetiology of epiglottitis
Infection of the supraglottis: classically with Haemophilus influenzae , but also from other potential pathogens, including Streptococcus pneumoniae , Staphylococcus aureus , and MRSA.
Other rare bacterial pathogens such as Pasteurella multocida have been reported.
Less commonly, viral pathogens such as parainfluenza have been implicated as aetiological agents.
Another rare aetiology of epiglottitis is fungal infection. Several reports of candidial epiglottitis have been reported.
Traumatic: a mechanical insult to the epiglottis can occur, most commonly from caustic ingestion or thermal injury, which may result in marked oedema and inflammation of the epiglottis.
RFs of epiglottitis
STRONG
No vaccination with H influenza vaccine
Immunocompromise
Middle age
Sx of epiglottitis
COMMON RFs No vaccination (Hib vacc) Sore throat Dysphagia Drooling (failure to swallow) Toxic appearance Acute distress Fever Tripod position Difficulty breathing
OTHER COMMON Decreased oral intake Muffled voice Stridor Irritability
Ix for epiglottitis
Laryngoscopy - swelling of supraglottic structures
Lateral neck radiograph - markedly enlarged epiglottis, referred to as a ‘thumbprint sign’
Can do:
Cultures of blood/supraglottis (epiglottis) - positive cultures - NB often not possible as it’s more important to secure airway
Rx of epiglottitis
- SECURE AIRWAY + O2
- IV Abx - cefotaxime/ceftriaxone
- Dexamethasone orally
- Inhaled adrenaline
- Still require ABx when stable and extubated
Complications of epiglottitis
Mediastinitis
Cellulitis of neck
Pneumonia - post intubation
Prognosis of epiglottitis
Epiglottitis is an acute condition and, though very severe (especially so in children), if the patient is treated appropriately the prognosis is excellent for a quick and complete recovery. Other than in unvaccinated or immunocompromised individuals, there is no theoretical increased risk for future episodes of this disease.
There are numerous sequelae of not treating an infectious process that causes airway obstruction. These include: death, mediastinitis, neck space infection, necrotising fasciitis, pneumonia, aspiration, and asphyxiation.
Define acute pharyngitis
Acute pharyngitis is characterised by the rapid onset of sore throat and pharyngeal inflammation (with or without exudate). Absence of cough, nasal congestion, and nasal discharge distinguishes bacterial from viral aetiologies. It can be caused by a variety of viral and bacterial pathogens, including group A Streptococcus (GAS), as well as fungal pathogens ( Candida ). Bacterial pharyngitis is more common in winter (or early spring), while enteroviral infection is more common in the summer and autumn. Generally a self-limited condition with resolution within two weeks.
Hallmarked by acute onset of sore throat; the absence of cough, nasal congestion and discharge suggests a bacterial aetiology.
Rapid antigen detection tests allow immediate point-of-care assessment of group A Streptococcus (GAS) pharyngitis.
The goal of treatment of GAS is to prevent acute rheumatic fever, reduce the severity and duration of symptoms, and prevent transmission.
Acute pharyngitis is generally a self-limited condition with resolution within two weeks. Infected individuals are not, however, immune to reinfection with most aetiological pathogens.
The only situation in which antibiotic prophylaxis to prevent GAS infections is recommended is for individuals with a history of rheumatic fever.
Epidemiology of acute pharyngitis
Bacterial pharyngitis is more common in winter (or early spring), while enteroviral infection is more common in the summer and autumn. Pharyngitis is most common in school-aged children during the winter months. Seasonal colonisation with group A Streptococcus (GAS) reaches its peak (up to 20% of children) during this season.
GAS pharyngitis is the focal point of clinical interest in pharyngitis, as the main goal of therapy is prevention of rheumatic fever associated with this organism. GAS pharyngitis, however, represents less than one third of all cases of acute pharyngitis.
Aetiology of acute pharyngitis
Acute sore throat is most commonly due to viral organisms or group A Streptococcus (GAS). Common viral causes include the Epstein-Barr virus (mononucleosis), adenoviruses, enteroviruses, influenza A and B, and parainfluenza. GAS pharyngitis is spread from person to person, especially in the winter, via the respiratory droplet route; however, it represents less than one third of all cases of acute pharyngitis. Other streptococci that can cause pharyngitis include groups B, C, and G streptococci. Other bacteria found rarely in adolescents with pharyngitis include Mycoplasma species and Arcanobacterium haemolyticum .
HIV, chlamydia, or gonorrhoea may be implicated as a cause of acute pharyngitis in sexually active adolescents or sexually abused children.
In low- and middle-income countries, diphtheria and measles are also prominent causes of acute pharyngitis.
Tularaemia may be a cause if there is a history of eating meat from undomesticated animals.
Candida infection is common as a source of sore throat in immunocompromised individuals, including those undergoing chemotherapy or oropharyngeal irradiation for cancer.
RFs for acute pharyngitis
COMMON Nasal colonisation with Group A Strep GAS infected contact Sexual activity/abuse Ingestion of non-domestic meats (Tularaemic ulceroglandular pharyngitis is acquired from the ingestion of partially cooked wild animal meat.) Immunocompromised Inhaled corticosteroids Lack of immunisation
WEAK
Irradiation
Sx of acute pharyngitis
COMMON RFs Child/adolescent Winter/spring (Bacterial) Summer/autumn (enteroviral) Rhinorrhoea, nasal congestion and cough (viral) Sore throat Pharyngeal exudate (GAS) Adenopathy [cervical] Fever >37 Headache N+V + Abdo pain
MEASLES:
Conjunctivitis
Koplik spots
Macropapular rash
UNCOMMON
Scarlatiniform rash
Pharyngeal ulceration (in tularaemia)
Ix for acute pharyngitis
Rapid antigen test for Group A strep - can be +ve/neg
IF NEG
- do throat swab culture
- test growth of chalmidya/gon if sexually active
Consider monospot for EBV heterophile ABs
Rx of acute pharyngitis
Supportive
- paracetamol
- ibuprofen
- lidocaine topical 2%
In instances when the rapid test for GAS proves negative, a reasonable approach is no antibiotics at all, with a next day follow-up if the throat culture is positive
Confirmed GAS
- ben-pen
- pheno-pen
- amox
- azithromycin
Recurrence:
Tonsillectomy
Infectious mononucleosis
- prednisolone orally
Candida
- antifungal - clotrimazole or fluconazole
Diptheria
- Diptheria antitoxin [equine]
Prognosis of acute pharyngitis
Antibiotic therapy of group A Streptococcus (GAS) pharyngitis results in a lessening of symptom intensity and duration, and prevents the long-term complication of rheumatic fever. Symptom resolution is within one to a few days. Infected individuals are not, however, immune to reinfection. Complications of viral pharyngitis are extremely uncommon. Symptoms usually go away within 7 to 10 days.
A 7-year-old girl presents with abrupt onset of fever, nausea, vomiting, and sore throat. The child denies cough, rhinorrhoea, or nasal congestion. On physical examination, oral temperature is 38.5°C (101°F), and there is an exudative pharyngitis with enlarged, tender anterior cervical lymph nodes. A rapid antigen test is positive for group A Streptococcus (GAS).
pharyngitis
A 7-year-old girl presents with acute sore throat accompanied by fever. On examination, oral temperature is 37°C (98.6°F), and there is an exudative pharyngitis without palpable cervical nodes. Both the rapid antigen test and throat culture are negative for GAS.
pharyngitis
Pharyngitis may be part of the presentation of viral upper respiratory infections that usually can be distinguished by the presence of rhinorrhoea, nasal congestion, and/or cough. Acute pharyngitis frequently occurs with acute infectious mononucleosis. It may rarely, in sexually active or sexually abused individuals, accompany acute HIV infection (with associated adenopathy, rash, fever, and splenomegaly), chlamydia, or gonorrhoea (no distinctive clinical features). Diphtheria and measles may present as pharyngitis and should always be a consideration when sore throat is encountered in low- and middle-income countries or in unvaccinated children. Tularaemia should be considered if symptoms are not responsive to penicillin treatment or if there is a history of eating meat from undomesticated animals. Candida infection is common as a source of sore throat in immunocompromised individuals, including those undergoing chemotherapy or oropharyngeal irradiation for cancer.
pharyngitis
A 4-year-old boy presents to the emergency department with complaints of dysphagia, fever, drooling, and muffled voice. Symptoms have progressively worsened over the course of the day. He is toxic-appearing, and leans forwards while sitting on his mother’s lap. He is drooling, and speaks with a muffled ‘hot potato’ voice. The parents deny trauma or evidence of foreign-body ingestion. They have no recollection of the child receiving a Haemophilus influenzae type B (Hib) vaccine.
Epiglottitis
The presentation of a patient with epiglottitis, especially in the post-Hib vaccine era, can be varied. [3] Vaccination is not 100% effective, so it is possible that patients may present with only some of the typical symptoms. Adults may have a more indolent presentation than children. [4]
A South Korean review documented a high incidence of epiglottic cysts in patients presenting with epiglottis (29%). In this series, these cysts predisposed patients to more severe airway obstruction and a higher risk of recurrence. [5]
Epiglottitis
A 6-year-old previously healthy boy presents with acute onset of fever of 39°C (102°F), severe throat pain that is exacerbated by swallowing, headache, and malaise. On examination his tonsils are symmetrically enlarged and red, with purulent exudate. He has multiple enlarged, painful anterior neck lymph nodes, but no other lymphadenopathy and no splenomegaly. He has no runny nose or cough, and no difficulty breathing.
Tonsillitis
Tonsillitis may occur in isolation or as part of a generalised pharyngitis. The clinical distinction between tonsillitis and pharyngitis is unclear in the literature, and the condition is often referred to simply as “acute sore throat”. Its severity may vary from a mild sore throat that responds well to simple analgesics to a severe sore throat that is associated with significant malaise and reduced oral intake, necessitating admission to hospital.
Tonsillitis
A 45-year-old male presents with chronic post-nasal drainage, cough, and nasal congestion that began approximately 2 years ago but has been getting progressively worse over the past few months. He snores, which results in sore throats in the morning. He experiences pressure over his forehead and behind his eyes, along with ear plugging and popping. His symptoms are aggravated by strong odours, such as perfumes/fragrances and tobacco smoke, and by weather changes. He is not bothered by cats, dogs, freshly cut grass, or dust. He never had these symptoms as a child and has no family history of allergies. These symptoms interfere with his work, as he has to make presentations to clients and is concerned that the constant clearing of his throat is diminishing his effectiveness.
Non-allergic rhinitis
Define laryngitis
Laryngitis refers to inflammation of the larynx. This can lead to oedema of the true vocal folds, resulting in hoarseness. Laryngitis can be acute or chronic, infectious or non-infectious. Accompanying signs of infectious laryngitis include odynophagia, cough, fever, and respiratory distress. The most common variant is acute viral laryngitis, which is self-limiting and usually related to an upper respiratory infection. Bacterial laryngitis can be life-threatening. Haemophilus influenzae is one of the most frequently isolated bacteria. Other causes include tuberculosis (TB), diphtheria, syphilis, and fungi. Non-infectious causes of laryngitis include reflux laryngitis, vocal strain and chronic irritant laryngitis.
Laryngitis is inflammation of the larynx, which can lead to oedema of the true vocal folds. Causes may be infectious or non-infectious (e.g., vocal strain, reflux laryngitis, chronic irritative laryngitis).
Generally clinically diagnosed.
Symptoms of acute disease, most commonly hoarseness, generally arise over a period of <7 days, are usually preceded by a viral upper respiratory infection (URI), and are ordinarily self-limiting. Patients may present with airway distress and high fever. Exudative tonsillopharyngitis with fever and anterior cervical lymphadenitis is highly suggestive of a bacterial origin.
The airway should be assessed first. Diligence and promptness are key, as they can be lifesaving.
Chronic laryngitis presents with hoarseness lasting >3 weeks. A thorough evaluation and specialist consultation should be obtained, because symptoms are similar to those of laryngeal malignancy.
Treatment for viral laryngitis consists of voice rest and hydration. For bacterial causes, antibiotics are used along with supportive measures. Vocal strain is managed with voice therapy and vocal hygiene.
In more pronounced cases, especially in children in whom the larynx is already small, oedema may lead to narrowing of the airway and airway compromise.
Epidemiology of laryngitis
Accurate figures with regard to acute laryngitis are difficult to collect, because it is generally unreported. Sore throat accounts for 1% to 2% of all patient visits to a primary care physician in the US. This accounts for approximately 7.3 million annual visits for children and 6.7 million for adults.
Viral agents tend to have annual periods of peak prevalence, such as rhinovirus infections in autumn and spring, and influenza virus infection epidemics generally from December to April.
TB laryngitis is historically a sequela of pulmonary TB, but recent cases without pulmonary involvement have been encountered. TB is the most common granulomatous disease of the larynx.
Fungal infections:
Generally caused by Candida albicans , Blastomyces dermatitis , Histoplasma capsulatum , and Cryptococcus neoformans .
Non-infectious causes of laryngitis include the following:
Irritant laryngitis (e.g., due to toxic exposure)
Allergic
Traumatic, especially due to heavy vocal use.
RFs of laryngitis
STRONG Hx URTI Incomplete/absent Haem Influenza (HIB) vaccine Incomplete/absent diptheria vaccine Contact with infected Travel to endemic TB/diptheria HIV/immunocompromise Residence in nursing home Inhaled corticosteroid/prolonged ABx use Heavy vocal use Tobacco use
Aetiology of laryngitis
Infectious laryngitis may be caused by viral, bacterial, or fungal infection.
Virus infection:
Generally the most common cause of infectious laryngitis
Rhinovirus is the most common virus that is aetiologically associated with URIs
Other causative viruses include parainfluenza virus, respiratory syncytial virus, influenza, and adenoviruses
Parainfluenza viruses type 1 and type 2, as well as influenza viruses, are the most common pathogens responsible for croup.
Bacterial infection:
Pathogens consist of Moraxella catarrhalis , Haemophilus influenzae , Streptococcus pneumoniae , Staphylococcus aureus , and Klebsiella pneumoniae
Epiglottitis is most frequently caused by Haemophilus influenzae type B
Diphtheria is caused by Corynebacterium diphtheriae . Occasional cases may be caused by Corynebacterium ulcerans
Although atypical forms of acid-fast bacilli can play a role, most TB infections are caused by Mycobacterium tuberculosis
Syphilis is a less common cause.
Sx of laryngitis
COMMON RFs presence Hoarseness - most characteristic symptom of laryngitis. Dysphagia Sore throat Odynophagia Cough Hyperaemia of oropharynx Hx heavy vocal cord use GORD
UNCOMMON
Oropharyngeal white-grey exudates - DIPTHERIA
OTHER COMMON / Concomitant Rhinitis Fatigue/malaise Fever Enlarged tonsils Enlarged/tender cer nodes PNDrip Dyspnoea
OTHER UNCOMMON / Concomitant
WL - TB
Tonsillopharyngeal exudate: bacterial
Acute resp distress - think acute epiglottitis, croup, or diphtheria
Drooling - epiglottitis
Stridor - acute epiglottitis, croup, or diphtheria.
Ix for laryngitis
1st
Laryngoscopy - mainstay of diagnosis - Performed if the patient presents initially to an otolaryngology specialist, but most primary care physicians are not experienced in the technique and diagnose most cases of viral laryngitis clinically
CONSIDER Biopsy if suspect TB Cultures - if bacterial Nasal swab - bacterial Rapid antigen - GAStrep Other TB Ix if suspected
Rx of laryngitis
IF AIRWAY COMPROMISE
- Stabilise
NO SUSPICION DIPTHERIA = give corticosteroid
SUSPICION DIPTHERIA = isolate and give BENPEN + diptheria antitoxin
ACUTE VIRAL
- Supportive
- Guaifenesin / codeine
ACUTE BACTERIAL
- Can give phenoxybenz
- Guaifenesin / codeine
TB
- referral + TB therapy
VOCAL REST
FUNGAL -> refer otolaryngologist
Complications of laryngitis
Airway compromise
Formation of true vocal folds
Largyneal stenosis (TB)
Muscle tension dysphonia
Prognosis of laryngitis
Acute infectious laryngitis
This is often a self-limiting disease. With adequate voice rest and hydration the voice will return to normal within days. Continued extensive voice use can result in injury to the true vocal folds and formation of pathologies. It may also lead to the development of compensatory behaviour and can result in muscle tension dysphonia. Therefore, the patient needs to be counselled on the importance of voice rest and hydration.
Diphtheria
Patient age and immunisation status are important factors in terms of likely prognosis. Elderly and very young patients generally have a poorer prognosis, whereas past history of immunisation usually leads to a better prognosis. Any delay in administration of diphtheria antitoxin is more likely to result in associated toxic complications. Therefore, it is important to give diphtheria antitoxin as soon as possible.
Tuberculosis
Once appropriate treatment is started, laryngeal lesions should regress. If left untreated, progressing lesions can cause fibrosis, scarring, and, as a result, laryngeal stenosis. This may necessitate tracheotomy.
A 45-year-old man presents with hoarseness for 5 days, cough, and pain on swallowing. He has no fever but complains of increased mucus in his throat and occasional difficulty breathing. He has no prior history of hoarseness, surgery to the larynx, intubation, or vocal abuse. He has slight throat pain but denies reflux symptoms. On examination, there is no acute respiratory distress. His oral cavity is within normal limits, but the oropharynx shows hyperaemia. The tonsils are slightly enlarged and erythemic. Mirror examination of the larynx reveals diffuse oedema and erythema of the laryngeal structures, with increased mucus in the glottis. The airway is patent. There are no lesions involving the true vocal folds, and they are both mobile.
Laryngitis
A 45-year-old man has hoarseness for the past 3 weeks, accompanied by painful swallowing and cough. He has no fever and states that he has lost 2.5 kg over the last 3 months. He is known to be HIV positive. Indirect laryngoscopy reveals an exophytic lesion on the left true vocal fold, with oedema of both true vocal folds.
Laryngitis
Diphtheria is encountered rarely in developed nations but can still infect children and adults who are immunocompromised or have not received vaccinations. Initial symptoms include hoarseness and sore throat. There is progressive shortness of breath as the patient becomes generally ill. On examination, the patient is toxic. Oral examination reveals white-grey exudates on the tonsils and the soft palate, extending down to the base of the tongue. Other less common causes of infectious laryngitis include syphilis and fungal infection. In patients who are using corticosteroid inhalers, the onset of hoarseness should raise the suspicion of possible laryngeal candidiasis, which is the most common type of fungal laryngitis. Patients with laryngitis due to vocal trauma will have an accompanying history of increased voice use and high vocal demands.
Laryngitis
Define pertussis / whooping cough
Pertussis (also known as whooping cough) is an upper respiratory tract infection (URTI) characterised by a severe cough. Bordetella pertussis is the typical aetiological agent.
Patients can be infectious for several weeks if it is left untreated. B pertussis will spontaneously clear from the nasopharynx within 3 to 4 weeks from the onset of the cough in about 80% to 90% of patients if untreated; however, infants who have not been vaccinated or treated can remain culture-positive for more than 6 weeks. Other Bordetella species that may rarely cause pertussis or pertussis-like cough include B parapertussis , B bronchiseptica , or B holmesii ; these species are not vaccine-preventable. In China, pertussis is known as the ‘100-day cough’.
With the resurgence of pertussis in highly vaccinated populations, the disease continues to be a public health and medical concern.
Three identifiable stages typical in childhood cases: catarrhal, paroxysmal, and convalescent.
Initial symptoms may be similar to a cold, with rhinorrhoea and lacrimation, or a dry cough followed by episodes of severe coughing. Fever may be absent or low-grade.
Inspiratory whooping is a characteristic symptom in children but may be absent in infants, adolescents, and adults.
Culture of the bacterium Bordetella pertussis from nasal secretions can confirm the diagnosis, especially early in the course of the disease. A negative culture does not exclude the diagnosis. Other diagnostic tests include polymerase chain reaction (PCR) and serology.
Macrolide antibiotics are the preferred first-line agent for treatment and prophylaxis.
Universal childhood immunisation with the acellular pertussis vaccine is advised. Booster vaccination is recommended for pregnant women. In the US, booster vaccinations are also recommended for non-pregnant adults.
Cocooning is a prevention strategy to protect newborns and infants.
Epidemiology of pertussis / whooping cough
Despite being a vaccine-preventable disease, pertussis remains a fairly common condition in developed countries that have longstanding immunisation programmes with high vaccine uptake, and it is still considered a public health concern.
Hypotheses that may explain the resurgence of pertussis include: the theoretical evolving virulence of B pertussis , waning vaccine-conferred immunity, waning herd immunity or incomplete booster vaccinations, the introduction of acellular vaccines, and increased physician awareness of pertussis in adolescents and adults.
In England in 2016, there were 5945 confirmed cases of pertussis, a rate of 11 per 100,000, an increase from previous years with 4190 cases in 2015 and 3388 cases in 2014.
However, in 2012, there were 9367 confirmed cases of pertussis in England (a rate of 18 cases per 100,000). This was considerably higher than the cases reported in 2011 (1053; rate of 2 cases per 100,000).
In the UK, US, and other developed countries, an epidemic outbreak of pertussis typically occurs every 3 to 4 years.
Aetiology of pertussis / whooping cough
B pertussis is the typical aetiological agent.
Other Bordetella species that may rarely cause pertussis or pertussis-like cough include B parapertussis , B bronchiseptica , or B holmesii ; these species are not vaccine-preventable
RFs for pertussis / whooping cough
STRONG
Age < 6months - Infants under 6 months of age are not fully vaccinated and so are at increased risk. The series of primary vaccines for pertussis starts at 2 months and ends at 6 months.
Baby born to mother infected by >34weeks
No / incomplete immunisation
School teachers / healthcare workers - As an adult, immunity can decrease, especially if the last booster vaccine has not been administered in previous 5 years.
Close contact with infected person
Sx of pertussis / whooping cough
COMMON RFs Cough (begins 1-2w post infection) Inspiratory whooping Rhinorrhoea - early stages Post-tussive vomiting / gagging
OTHER COMMON
Sneezing
Absent/low grade fever
Decreased appetite
UNCOMMON
Apnoea (baby)
Ix for pertussis / whooping cough
Culture of a nasopharyngeal aspirate or swab from the posterior nasopharynx - Definitive diagnostic test, with a specificity of 100%. May be positive for B pertussis
PCR naso-pharyngeal aspirate - Sensitivity of 94% and a specificity of 97%
Serology - IF NO VACCINE - Because a vaccination can induce antibodies (i.e., IgM, IgA, and IgG antibodies), serological assays cannot differentiate infection from vaccine response
FBC - inc WCC
Rx of pertussis / whooping cough
INFANT <1month
Azithromycin / clarithromycin
Infants aged under 1 month who receive a macrolide should be monitored for hypertrophic pyloric stenosis (IHPS) DONT GIVE ERYTHROMYCIN
> 1month / non-preg
Azithromycin / clarithromycin / erythromycin base
If given after 3 weeks of onset of cough, treatment may have limited benefit.
Prognosis of pertussis / whooping cough
Infants <1 year in age account for the majority of deaths from pertussis in the US. The mortality rate of hospitalized infants is 1%.
In older children and adults, the prognosis is generally very good. It may take a few months for the cough to entirely resolve. Most healthy older children and adults will make a full recovery. Those with comorbid conditions, however, have a higher risk of morbidity and mortality. Because infection in newborns and infants can be severe, they need careful monitoring to avoid complications. The hospitalisation rate of adults in the US is about 3%.
The rate of developing pneumonia is up to 5% and rib fracture is up to 4%.
Complications of pertussis / whooping cough
Pneumonia Seizure Apnoea/brady Rib fracture Encephalopathy Otitis media
A 40-year-old high school teacher presents with cold symptoms lasting 3 weeks. She has low-grade fever, fatigue, and paroxysms of coughing. Her cold symptoms were initially mild but gradually increased in severity, resulting in her presentation to the emergency department. OTC cold medications have not provided relief.
pertussis / whooping cough
A 12-month-old female infant presents with spasmodic cough, cyanosis around her lips and fingers during coughing, and post-tussive vomiting. Her parents report that she has had a cold for approximately 3 weeks, and her appetite has decreased. The infant’s mother reports that she herself has been coughing for 6 weeks. The infant’s immunisation records are incomplete.
pertussis / whooping cough
Define vasovagal syncope / neurally mediated reflex syncope
Syncope is a syndrome characterised by a relatively sudden, temporary, and self-terminating loss of consciousness, associated with the inability to maintain postural tone, with rapid and spontaneous recovery. The causes vary widely from patient to patient, but they have a common underlying pathophysiology. The transient loss of consciousness with syncope is the result of a temporary inadequacy of cerebral nutrient flow, most often triggered by a fall in systemic arterial pressure below the minimum needed to sustain cerebral blood flow. Neurally mediated reflex syncope (NMRS) refers to a group of related conditions or scenarios in which symptomatic hypotension occurs as a result of neural reflex vasodilation and/or bradycardia. Vasovagal syncope (VS) refers to a particular type of NMRS also known as the common faint.
Neurally mediated reflex syncope (NMRS) refers to a group of related conditions or scenarios in which symptomatic hypotension occurs as a result of neural reflex vasodilation and/or bradycardia.
Vasovagal syncope (VS) refers to a particular type of NMRS also known as the common faint.
VS has many manifestations and is generally considered to encompass faints triggered by emotional upset, fear, and pain, as well as those occurring in less well-defined circumstances.
Patient education is the foundation of treatment for most NMRS syndromes, including VS.
Patients must be informed that, although reflex faints are almost never life-threatening, they tend to recur (often in clusters), and injury can result if preventive measures are not taken seriously.
Strategies for reducing syncope recurrences in the long term comprise 1) physical techniques to improve orthostatic tolerance, 2) pharmacological interventions to prevent depletion of intravascular volume and/or enhance arterial and venous tone, 3) cardiac pacing to avert bradycardia.
Epidemiology of vasovagal syncope / neurally mediated reflex syncope
Syncope is estimated to account for approximately 1% of accident and emergency department visits in Europe.
Prevalence of syncope varies in the population: 15% of children.
Vasovagal syncope is the single most common cause of syncope. It tends to peak in frequency in young adults and then again in older people. Unfortunately, in older patients the historical features may be less clear cut, making the diagnosis more difficult to establish. Most studies suggest that women (especially younger women) are more susceptible to vasovagal faints, although reporting bias may play a role, as young men may be less inclined to report such events.
Aetiology of vasovagal syncope / neurally mediated reflex syncope
Syncope has many possible causes, and these causes are commonly grouped in 3 main groups: neurally-mediated reflex syncopal syndromes, orthostatic, or cardiovascular.
In susceptible people, vasovagal syncope may be triggered by prolonged periods of upright posture; relative dehydration; excessively warm, closed-in environments; or extreme emotions. Common places for these events are churches, restaurants, and long queues.
RFs for vasovagal syncope / neurally mediated reflex syncope
STRONG Hx syncope Prolonged standing Emotional stress Dehydration Preceding episode of nausea / vomiting Preceding episode of severe pain
Sx of vasovagal syncope / neurally mediated reflex syncope
STRONG Nausea Light headedness Pallor Diaphoresis Diminished vision/hearing Physical injury Fatigue after episode Palpitations prior Bradycardia at time
Ix for vasovagal syncope / neurally mediated reflex syncope
INITIAL
12 lead ECG - rule out cardiac causes Serum Hb - rule out anaemia BM/BG - rule out hypo BHcG - rule out preg (can cause syncope) Cardiac enzymes - rule out MI D-Dimer - rule out PE serum cortisol - rule out addisons Urea/creatinine - check dehydration
OTHER Tilt-table Standing BP Valsalva manouvre Carotid sinus masage - all to rule out alternatives if unclear
Rx of vasovagal syncope / neurally mediated reflex syncope
Education re triggers
Rehydration
Can give fludrocortisone to aid re-expansion but RAREKY used
Prognosis of vasovagal syncope / neurally mediated reflex syncope
Recurrences are common but often occur in clusters. Most therapies have not been studied adequately to ascertain effectiveness for preventing subsequent events. However, data for physical manoeuvre intervention and for the drug midodrine tend to support benefit. In the long term, the mortality risk is very low, but injury is a concern due to recurrent falls, especially in older people
Complications of vasovagal syncope / neurally mediated reflex syncope
injury/fracture
Haemorrhage 2nd trauma
Define Legg-Calvé-Perthes’ disease
A self-limiting disease of the femoral head comprising of necrosis, collapse, repair, and re-modelling that presents in the first decade of life and is more commonly seen in boys. The cause is hypothesised to be single or multiple vascular events, followed by re-vascularisation. In later life, it can lead to a painful and poorly functioning hip. The disease was first described independently by Jacques Calvé, Arthur T. Legg, and Georg Perthes in 1910
Patients commonly have a characteristic phenotype of short stature, delayed bone age, and hyperactivity.
Although several aetiologies have been proposed, a multi-factorial involvement is most likely.
Typically unilateral, though bilateral involvement is present in 10% of cases.
Clinical features include a limping gait and hip pain frequently radiating into the thigh, knees, groin, or buttocks. Pain is worse with activities.
Treatment is age-dependent. Options include mobilisation and monitoring, non-surgical or surgical containment, or salvage procedures.
Epidemiology of Legg-Calvé-Perthes’ disease
Typically, Perthes’ disease presents in 4- to 8-year-olds with a mean age of 7 years, although it can affect children between 2 and 12 years of age. It is about 4 or 5 times more common in males than females. Perthes’ disease is bilateral in about 10% to 12% of cases.
The disease appears very uncommon in black people, though there are no true studies of incidence. Studies of incidence demonstrate that South Asians have 3 times the rate of East Asians, and that Caucasians have 9 times the rate of East Asians. More northerly latitudes, independent of race, appear to have greater disease incidence.
RFs for Legg-Calvé-Perthes’ disease
STRONG Male 4-8yo Socio-economic deprivation Hypercoagulable states
WEAK Urban population Eskimos, Japanese, some central European races Transient hip synovitis Hip joint effusion Passive smoking Skeletal dysplasias Trisomy 21 Congenital defects - Genitourinary tract and inguinal anomalies have been demonstrated to have an association with Perthes' disease. Behaviour disorders - ADHD Endocrinopathy
Aetiology of Legg-Calvé-Perthes’ disease
The cause of Legg-Calvé-Perthes’ (Perthes’) disease is hypothesised to be single or multiple vascular events, followed by re-vascularisation. Although several theories have been proposed over the years, it appears that Perthes’ is most likely to be multifactorial.
Sx of Legg-Calvé-Perthes’ disease
STRONG
RFS
Limp - painless limp that can be described as a gluteus medius lurch, though may be antalgic during acutely painful episodes
Limited range motion of hip - There is a flexion deformity of the hip in the acute setting. With progression of the disease adduction in flexion, internal rotation, and abduction in extension are limited due to impingement lesions. The later stage of the disease may be characterized by a global reduction in all ranges of motion with associated pain indicative of joint arthritis.
OTHER COMMON Short stature Muscle wasting Hyperactivity Trendelenburgs Synovitis
UNCOMMON Knee pain Thigh pan Groin buttock pain Asymmetric limb length Minor trauma
Ix for Legg-Calvé-Perthes’ disease
Bilateral hip x-rays - femoral head collapse and fragmentation; subchondral
CONSIDER
FBC - N
ESR - N / reactively increased
CRP - N / reactively increased
Bone scintigraphy - cold spot in the affected hip early in the disease process
MRI possible
Rx of Legg-Calvé-Perthes’ disease
Supportive pain relief - paracetamol 75mg/kg 4-6, max / ibuprofen (10mg/kg 6-8hrly)
<5yo
- Mobilisation + monitoring
- Non-surgical containment
- A dynamic radiographic screening arthrogram allows assessment of the optimal position in which to contain the hip. This position is then maintained in an abduction cast, splint, or brace, or a combination of all 3 where a spica and adductor tenotomy is followed by a period in an abduction brace.
5-7YO
Epiphyseal involvement >50%
- SURGICAL containment
7-12YO
SAME but also:
- SALVAGE procedure
Salvage options address either the acetabulum to recreate or deepen the socket or aim to improve congruence between the weight-bearing areas by altering the femoral head orientation with a femoral valgus osteotomy.
> 12 with arthritis
Replacement arthroplasty after skeletal maturity
Prognosis of Legg-Calvé-Perthes’ disease
The prognostic variables include femoral head sphericity, congruence with the acetabulum, age at presentation and the geometric extent of the femoral head involvement. Congruence is a more important predictor of future arthritis than femoral head sphericity.
In later life, Legg-Calvé-Perthes’ (Perthes’) disease leads to a painful and poorly functioning hip. More than 50% of patients with Perthes’ disease will develop signs of osteoarthritis between their 4th and 5th decades.
When girls are affected, involvement of the femoral head is more severe compared with that in boys of the same age. Females also tend to have a more severe involvement than males at a later age.
Predictors of poor outcome include recurrent synovitis, lateral subluxation, more than 50% femoral head involvement and a collapsed lateral pillar.
Predictors of good outcome include femoral head involvement classified as Catterall group I or II, Salter Thompson group A, Herring sub-types A or B, and children under 7 years of age.
Complications of Legg-Calvé-Perthes’ disease
Osteoarthritis LT
Limb length inequality LT
Stiffness + loss of rotation V
A 5-year-old Caucasian boy is brought in to the orthopaedic clinic by his mother with complaints of a limp favouring the right side with no associated pain. This painless limp had started insidiously 3 weeks earlier and was first noticed by the school physical education teacher. The mother notes that it has recently been getting worse. He is one of 3 siblings and lives with his single mother. The other siblings include an elder sister (from his mother’s earlier marriage) and a younger brother. His mother’s current partner is a heavy smoker. His mother recalls that 1 year earlier he came from school and complained of right knee pain. This was initially overlooked for a few days, but when it persisted he was taken to the general practitioner who reassured his mother but did not arrange follow-up. The symptoms had recurred the following month when he was taken to the emergency department and blood tests and x-rays were reportedly normal.
Legg-Calvé-Perthes’ disease
A 7-year-old girl presents with a painless limp, although she has been treated for acute pain in the past. She has a free range of hip motion. Plain anteroposterior and frog lateral radiographs reveal Legg-Calvé-Perthes’ disease of the right hip. She undergoes screening of the right hip under a general anaesthetic with an arthrogram to determine the best position of right femoral head containment in the hip joint. A surgical containment by way of a varus lateral opening wedge osteotomy of the proximal femur, fixed with an AO (Arbeitgemeinschaft fur Osteosynthesefragen) 3.5 mm pre-contoured plate and held with 3.5-mm AO screws, is performed. She is maintained non-weight-bearing post-procedure. A radiograph 4 weeks later reveals a good containment of the hip joint, satisfactory alignment at the osteotomy and some early callus formation medially. A further radiograph 3-months postoperatively shows a good union at the osteotomy and a well-contained hip joint. Weight-bearing and resumption of normal activities is then allowed progressively.
Legg-Calvé-Perthes’ disease
The typical age of presentation varies between 4 and 8 years but can range from 2 to 12 years with a few reports of involvement in adolescents. The usual phenotype of a child with Legg-Calvé-Perthes’ (Perthes’) disease is a person with short stature, delayed bone age and a rather hyperactive child. A limp is the most common presentation with a large proportion being unilateral afflictions. Bilateral involvement is seen in up to 10% to 12% of the cases, however, both hips are rarely at the same stage of the disease at any given time. Pain may vary in site, intensity and radiation patterns. It may be referred to the knee, thigh, groin, or buttocks. Perthes’ disease is a non-traumatic condition, although a history of minor trauma may be noted.
Legg-Calvé-Perthes’ disease
Define allergy in a child
Allergy = a hypersensitivity reaction initiated by immunological mechanisms. This can be IgE mediated [eg peanut] or non IgE mediated [coeliac disease]
Hypersensitivity = objectively reproducible symptoms in response to a defined stimulus at a dose which is tolerated by normal people
Epidemiology of allergy in a child
40% if children have AR, asthma or eczema and up to 6% develop a food allergy
Increasing in prevalence in many countries
Commonest chronic diseases of childhood, and causes of hospitalisation and school absence
Significant morbidity and can be fatal
20 children die from asthma each year
2 from food allergy
Aetiology/RFs of allergy in a child
Polymorphisms in genes lead to susceptibility
Abnormal immune response to harmless stimuli (usually protein)
Developing immune system is “sensitised” before the allergic response develops
N.B. sensitisation can be occult, EG egg sensitisation from trace amounts in maternal breast milk
Only a few stimuli typically account for most allergic disease:
Inhalant: dust mite, plant pollen, pet dander, moulds [asthma/AR]
Ingestant: nuts, seeds, legumes, cows milk, egg, seafood, fruits
Bites/stings, drugs, latex
Some proteins may be broken down by cooking: eg apples
Early phase:
- release of histamine, LTs and PGs
- urticaria, angiooedema, sneezing, bronchospasm
Late phase 4-6 hrs later
- nasal congestion, cough, bronchospasm
NB non-IgE mediated = delayed onset of symptoms by 4-6 hrs + more varied clinical picture
Sx of allergy in a child
Mouth breathing
OSA/snoring
Allergic salute
Pale/swollen inferior turbinates
Hyperinflated chest or Harrison sulci from chronic untreated asthma
Atopic eczema affecting limb flexures
Allergic conjunctivitis: may be prominent creases ie DennieMorgan folds, blue grey discolouration below the lower eyelids
Check growth with food allergy
- also with corticosteroid Rx
Immunotherapy Rx of allergy in a child
Allergen immunotherapy
AR, conjunctivitis, asthma, anaphylaxis, insect stings
- SCIT/SLIT
- Must have good adherence, only 1 allergen
- Months of progressive low dose exposure under specialist supervision
Food allergy definition
Pathological immune response mounted against a specific food allergen. Usually IgE mediated.
NB a non-immunological hypersensitivity reaction to specific food is called food intolerance.
Aetiology of food allergy in children
Infants most common is milk, egg and peanut
Children = peanut, tree nut, fish and shellfish
Food allergy can be secondary. If so, usually because of cross-reactivity of the allergen IE similar protein structures
EG apple + birch tree = oral allergy syndrome
Clinical features of food allergy/intolerance in children
IgE - urticaria, facial swelling, anaphylaxis, 10-15 mins after food
Non IgE - diarrhoea, vomiting, abdominal pain, sometimes failure to thrive, colic/eczema can be present, can present as blood in the stools from proctitis
Intolerance - diarrhoea, vomiting, bloating, cramps, steatorrhoea
Diagnosis of food allergy/intolerance in children
IgE mediated - skin prick tests or RAST, NB both can yield false positive IE individual sensitised but not allergic, but usually a greater response = greater likelihood of allergy
Non IgE - Usually relies on clinical Hx + examination, + endoscopy/biopsy. Diagnosis is supported by presence of eosinophilic infiltrates
Both IgE/non - gold standard is exclusion of food, followed by a double blind placebo challenge
Rx of food allergy/intolerance in children
Avoidance
Allergic attack education
Adrenaline / EpiPen
Food allergy to cows milk / eggs usually resolves in early childhood, but nuts/seafood usually persists
Rx of allergic rhinoconjunctivitis
Second gen non-sedative antihistamines (top/oral)
Topical corticosteroid nasal/eye preps
Chromoglycate (mast cell stabiliser) eye drops
LTRA eg montelukast
Nasal decongestants - DO NOT USE FOR MORE THAN 7 days else risk of rebound effect
Allergen IT
Systemic corticosteroids ARE NOT USED DUE TO RISK OF ADVERSE EFFECTS
Define urticaria / angiooedema
Rash resulting from exposure to allergen or viral infection
Angiooedema = involvement of deeper tissues: swelling of the lips and soft tissues around the eyes
Chronic = >6weeks - usually non-allergic
NB physical urticarias: cold, delayed pressure, heat contact, solar, vibratory
NB other causes: water (aquagenic), sweating (cholinergic), aspirin, NSAIDs, C1 esterase inhibitor deficiency [angiooedema w/o urticaria/pruritis]
Aetiology of cows milk intolerance
- May be IgE mediated: immediate Sx on ingestion - urticaria->angio oedema-> anaphylaxis depending on severity
DIAGNOSED BY: skin prick test - Non - IgE - loose stools, failure to thrive, skin prick negative, elimination reduces symptoms, reintroduction reproduces Sx
- Non-allergic cows milk hypersensitivity - aka temporary lactose intolerance - prev well, develops D+V, vomiting settles but watery stools continue for several weeks, on stool sample - NO PATHOGENS but reducing substances (ie detection of glucose, fructose, lactose, galactose, and pentose, depicting malabsorption)
Define anal fissure
An anal fissure is a tear of the squamous epithelial mucosa of the anal canal, between the anocutaneous junction and the dentate line. They most commonly occur during passage of a firm stool. Anal fissures are common in infancy, and they represent the most common cause of bright rectal bleeding at any age.
Aetiology of anal fissure
The generally accepted proximate cause of the anal fissure is a mechanical tear resulting from the passage of hard stool. An unhealed fissure may become infected and develop into a chronic ulcer. A healed fissure may develop into a classic sentinel skin tag in the posterior midline.
Fissures have a predilection for the posterior midline (90%) but may also be located in the anterior midline or laterally. The explanation for this phenomenon is both anatomic and functional. The posterior commissure of the anoderm is less well perfused than other anodermal regions.
Epidemiology of anal fissure
Most fissures affecting the pediatric population manifest in children aged 6-24 months; however, the overall incidence of the problem is not well described.
RFs for anal fissure
Constipation
Low fibre diet
Sx of anal fissure
Cry on bowel movements
Streaks of blood on surface of faeces
Hard stool
Fresh blood on toilet paper/loo
It is important to remember that underlying systemic illness frequently manifests with anal lesions. Thus, pertinent negatives, such as fever, rash, oral or skin lesions, weight loss, diarrhea, and abdominal pain, should be excluded.
Visualisation of fissure on examination
The fissure appears as a minor laceration, usually in the midline, and is more often posterior than anterior. If the fissure is chronic, a small external skin tag (ie, sentinel tag) may be identified at the base of the laceration; this represents epithelialized granulomatous tissue secondary to chronic inflammation.
If a fissure is suspected, palpation of the abdomen is essential to check for palpable masses (stool) in the left lower quadrant.
If the examination is limited by pain and the diagnosis remains unclear, an examination under anesthesia should be pursued.
Rx of anal fissure
Will heal in 10-14 days
Surgery only required if doesn’t heal after 6-8weeks (eg, chemical or surgical sphincterotomy)
Conservative
Fibre diet, stool softeners (osmotic usually polyethylene glycol)
Chemical sphincterotomy - topical GTN 0.2%, [SE GTN headache]
Topical diltiazem 2% gel
Topical nifedipine with lidocaine
Botox injections reduce anal sphincter tone by inhibiting AcH release (normally not done in children)
Surgery:
Outpatient open lateral internal sphincterotomy
Ix for anal fissure
Clinical
Prognosis of anal fissure
Recurrence rates of open or closed lateral sphincterotomy have been reported to be 0-10%, with most of the recurrences occurring in adults and with chronic fissures. In contrast, anal dilatations have the highest rates of fistula recurrence (10-30%) and, for this reason, are not recommended in children.
Complications of anal fissure
Headache and diarrhea are the most common complications of administering topical nitrates and stool softeners, respectively. Significant hypotension with topical nitrate administration has not been reported in the literature. However, during the first office visit, children and their families should be questioned about a history of vascular headaches, and blood pressure should be taken before the initial application of topical nitrates. Incontinence has not been associated with these therapeutic regimens.
Short-term complications of operative therapy include urinary retention, hematoma formation, and incontinence. Long-term complications, such as difficulty controlling flatus, daytime soiling of underwear, and nighttime incontinence, are noted with both open and closed internal sphincterotomy.
Define asthma
Asthma is a chronic respiratory disorder characterised by variable airway inflammation, airway obstruction, and airway hyper-responsiveness. These features interact to determine the clinical symptom pattern of the individual. While the majority of asthmatic children have an intermittent symptom phenotype, the minority have persistent symptoms, reflecting the underlying chronic inflammation. In older children, as in adults, this may lead to permanent structural alterations of the airway wall (airway remodelling) and potentially a more severe asthma phenotype.
This topic covers the treatment of children up to 12 years of age. Children 12 years and older are treated the same as adults, except for emerging therapies such as bronchial thermoplasty. Please see our Asthma in adults topic for more information.
As paediatric asthma differs from adult asthma, child-specific asthma guidelines should be used and adult and adolescent guidelines should not be extrapolated to younger age groups.
The prevalence of childhood asthma appears to have plateaued in many affluent countries. However, asthma remains a significant cause of childhood morbidity and mortality, and is still a common problem managed in ambulatory and emergency care settings.
Most children with asthma have mild intermittent asthma and do not require daily therapy. Minimum doses and medications for maximal control should be used in those who require preventative therapies. Therapy should be individualised.
A number of important differential diagnoses, adherence and environmental issues should be considered when evaluating a child with suspected asthma, especially in very young children or when high doses of inhaled corticosteroids are required.
Epidemiology asthma
Paed asthma = most common chronic respiratory disease in the developed world
10% prev roughly
Prepubertal male + post pubertal females most common
Aetiology of asthma
Mixed genetic / environmental
Transient early wheezing (1 or more episodes of lower respiratory tract illness [LRTI] with wheezing in the first 3 years of life, but no wheezing at 6 years of age)
Late-onset wheezing (no history of LRTI with wheezing in the first 3 years, but wheezing at 6 years of age)
Persistent wheezing (1 or more episodes of LRTI with wheezing in the first 3 years of life, and wheezing at 6 years of age).
RFs for asthma
STRONG Allergic sensitisation/positive SPT Atopic disease Wheezing triggered by non-viral non-allergic environmental factors (change in weather, environmental tobacco smoke, exercise, and emotion) Resp virus early in life Serum eosinophilia (>4%) Fix Passive/active smoking Abnormal lung function/hyperresponsiveness
WEAK
Female
Air pollution
Obesity
Sx of asthma
COMMON Wheezing SOB/dyspnoea Features atopic disease Tachypnoea Recessions Accessory muscle use Hx of response to Rx
OTHER COMMON >3yo Dry night time cough Dyspnoea on exertion Expiratory wheezing
UNCOMMON
Chest wall deformity - Harrison sulci, hyperinflation
Ix for asthma
1st
Spirometry - dec FEV1/FVC ratio >0.8 [obstructive pattern]
>12% improvement post bronchodilator
CxR - hyperinflation/normal
CONSIDER PEFR challenge - BEWARE (limited role in paediatric asthma due to its effort dependence and lack of sensitivity) Airway challenge - Dec FEV1 Exercise challenge FBC - Eosinophilia Sweat test - NORMAL [CF] FeNO - elevated
Rx of asthma
0-4yo
- SABA PRN + education
- Low dose ICS ie Fluticasone propionate 50 micrograms/day + SABA
- Add LTRA - Montelukast 4mg oral
- Med dose ICS 125ug fluti
- Add LTRA
- High dose ICS 250ug fluti
- Add LABA
IE NO LABAs in <4 until very last line Rx
5+ 1. SABA 2. Low dose ICS 3. Add LTRA/theophylline/sodium chromoglycate 4. Med dose ICS + SABA OR Low dose ICS + LABA/LTRA/theophylline 5. High dose ICS + LABA/LTRA/theophylline 6. Immunomodulator - omalizumab 7. Oral CS / immunomodulator
Prognosis of asthma
Life expectancy equivalent
ICS
Regular use has negative effects on short-term linear growth, particularly at higher doses. A transient slowing in height velocity does occur initially, and may affect final achieved adult height.
Complications of asthma
Exacerbation - ST H Remodelling - V M Dental caries RML syndrome Oro candidiasis Adrenal suppression
Define pre-school wheeze
A wheeze present in pre-school children. Thought to be viral-associated, resulting from small airway susceptibility to narrowing/obstruction during inflammation/aberrant immune responses to viral infections.
Thus = episodic
RFs of pre-school wheeze
Maternal smoking during/after preg
Prematurity
Male
Usually resolves by 5
Aetiology of pre-school wheeze
Bronchiolitis, pneumonia Transient early wheezing Non-atopic wheeze after LRTI Atopic asthma Cardiac failure Inhaled foreign body
Children may present with episodes of recurrent cough. These episodes may be triggered by viral infections, change in weather, or exercise. The cough is typically dry in nature and only occasionally associated with an audible wheeze. Closer questioning may reveal the predominant trigger and a feeling of chest tightness and difficulty breathing accompanying the cough. A beta-2 agonist MDI typically provides relief.
Asthma
Indications for hospital admission in asthma?
Persisting dyspnoea/tachy despite reliever therapy
Exhaustion
Marked reduction in PEFR
<92% saturation on air
CxR only indicated if there are unusual Sx / Infection Sx
Rx of an asthma attack?
SEVERE O2 via face mask Try SABA 10 puffs / oral or IV pred Neb ipatropium Repeat bronchodilators every 20-30mins
Life threatening
- AKA silent chest, altered consciousness, cyanosis, O2 <92%, PEFR <33%
Neb salb+ipa bromide
IV hydrocortisone
Discuss with senior clinician or PICU
Repeat bronchodilator every 20-30 min
Rx of an asthma attack?
SEVERE O2 via face mask Try SABA 10 puffs / oral or IV pred Neb ipatropium Repeat bronchodilators every 20-30mins
Life threatening
- AKA silent chest, altered consciousness, cyanosis, O2 <92%, PEFR <33%
Neb salb+ipa bromide
IV hydrocortisone
Discuss with senior clinician or PICU
Repeat bronchodilator every 20-30 min
Define autism spectrum disorder
Autism spectrum disorder (ASD) is characterised by persistent impairments in social communication, and restricted, repetitive, and stereotyped patterns of behaviours, interests, or activities.
Abnormal development is present during early childhood, but may only become manifest later. There may be a history of language delay (single-word or phrase speech delay) and 25% of children lose previously acquired acquired language skills (regression).
Approximately 20% to 30% of children develop epilepsy and 50% have intellectual disability; others have ability in the average or above average range.
Epidemiology of autism spectrum disorder
In the last 10 years, the detected prevalence of ASD has increased substantially.
More recent data suggest that at least 1% of children have ASD; some studies report a prevalence of around 2%.
Around 50% of children with ASD have an intellectual disability.
Data suggest that the rise in ASD diagnosis has been accompanied by a fall in the prevalence of other neurodevelopmental disorders and that more accurate clinical diagnosis underlies the increase in detected prevalence.
Aetiology of autism spectrum disorder
It is well established that there are strong genetic influences in the development of ASD, with a sibling recurrence risk in the region of 10%.
In a minority of cases (approximately 10%) ASD occurs in association with cytogenetically detectable chromosomal abnormalities and recognised genetic syndromes, such as fragile X syndrome, tuberous sclerosis, and Down’s syndrome.
RFs for autism spectrum disorder
STRONG
Male 4:1
FHx
WEAK
Genetic variants
Chromosomal abnormalities - NF1/Downs/tuberous sclerosis
Sx of autism spectrum disorder
COMMON
RFs
Language delay or regression - no single words by 2yo, or phrases by 33months, lack of babble in 1st yr, echolalia, stereotyped speech, NB some may develop language very early
Verbal + non-verbal communication impairment - dec pointing, lack of social interaction, dec facial expressions, unable to depict emotion
Social impairment - uninterested in play, lack of friendships, misunderstandings
Repetitive/rigid/stereotyped behaviour or interests - repetitive language or requests, toys, play, obsessions, distressed if this is altered.
OTHER COMMON Placid or irritable as a baby Feeding difficulties Unusual posturing Motor stereotypes - repetitive snapping, clapping, spinning etc Sensory interests
UNCOMMON
Macrocephaly
Signs of fragile X syndrome, tuberous sclerosis, dysmorphism (may indicate chromosomal disorder) are weakly associated with ASD.
Ix for autism spectrum disorder
ASD screening questionnaire - eg autism screening questionnaire
Childhood autism rating scale
Childhood autism screening test
Consider audiology
Rx of autism spectrum disorder
Applied behaviour analysis Picture exchange communication systems ASD preschool programme Early start Denver model ChildsTalk
PHARMACOLOGICAL
Some psychotropics may aid those with very challenging behaviours, aggression, or irritability that does not respond to behavioural techniques
Risperidone Aripiprazole Fluoxetine Methylphenidate Atomoxetine Melatonin - for sleep disturbance Buspirone
Complications of autism spectrum disorder
Developmental regression Intellectual disability Epilepsy Anxiety Depression
Prognosis of autism spectrum disorder
any adults with ASD require lifelong full-time care. About 15% of adults with ASD will live independent lives, whereas 15% to 20% will live alone with community support.
Verbal and overall cognitive capacity seem to be the most important predictors of ability to live independently as an adult.
A 3-year-old boy presents following concerns about language development. He started using single words at age 18 months but still doesn’t use 2 words together. He stopped using words he had previously learnt between 18 and 24 months, but has now regained most of these words. He also seems uninterested in engaging with other children. He occasionally engages with his parents but less than they think he should. He doesn’t tend to look at them much and he has difficulty maintaining eye contact with them. When he wants something he pulls them to where the object is and screams; he doesn’t point like other children. His parents have also noticed that he does not play in the same way as other children of his age; he tends to line toys up, or plays with certain aspects of them, such as the car doors. He doesn’t use the toys in the imaginative way that other children do. When his toys are moved he becomes very upset. He tends to become distressed when he thinks there is change around the house. In contrast, he is not concerned when either of his parents leaves the house. He tends to flap his hands at times and his parents report him staring at the ceiling lights for 10 to 20 minutes at a time. He is a fussy eater and hates being messy.
ASD